You are on page 1of 184

Publications of

Menoufia University, Faculty of Engineering, Shebin El-Kom


Course code: BES 112 1st grade engineering
2021/2022

Mathematics (2)

Prepared by

Dr. Eng./ Muhammad Magdy

1
Faculty Vision

‫تحقيق التطوير المستمر فى التعليم الهندسى والبحث العلمى وخدمة المجتمع فى المجاالت‬
.‫الهندسية المتعددة‬

Achieve the continuous improvement in engineering


education and scientific research and community service
in various fields of engineering

Faculty Mission

‫تخريج مهندسين مؤهلين لتلبية احتياجات سوق العمل فى القطاعات الهندسية والتكنولوجية من‬
‫خالل برامج تعليمية تتوافق مع المعايير القومية االكاديمية المرجعية وبرامج للدراسات العليا فى‬
‫مختلف التخصصات الهندسية كما تقدم الكلية دورات تدريبية متخصصة واستشارات هندسية‬
.‫وبحوث علمية تساهم فى خدمة المجتمع لتنمية البيئة المحيطة‬

Graduating qualified engineers to meet the needs of the


labour market in the sectors of engineering and technology
through educational programs consistent with the national
academic references standards as well as postgraduate
programs in various engineering disciplines. The faculty
offers specialized training courses and consulting
engineering and scientific research that contribute to
community service and development of the surrounding
environment.

2
Basic Engineering Science. Academic year: 2021-2022
Dept. Academic term: 1st Term
Faculty of Engineering Academic level: 1st power
Menoufiya University

Course Specification

A-Basic Information

Title: Mathematics (2) Code Symbol: BES 112


Element of program: Major Date of specification approval: 2011
Department offering the course: Basic Engineering Science. Dept. By law: 2006

Lecture Tutorial Laboratory Total

4 2 ---- 6

1- Course Subject Area:


Math.
Humanities Applied Computer Projects
and Basic Eng. Discretionary Total
and Social Eng. applicatio and
Basic Science subjects
Science and Design n and ICT practice
Sciences
------ 100% ----- ------ ------- ------ ------- 100%

B-Professional Information
2- Course Aims:

The aims of this course are to provide the Student, To know the basic knowledge and skills for
solving differential equations and To use the multiple integral methods for finding the area, mass,
center of gravity, and the moment of inertia. This course will also provide students, after graduation,
with an understanding of use Laplace transformation and inverse Laplace transformation for solving
differential equations and develop skills of testing series for convergence and how to find Fourier
expansion for functions related to Electrical Engineering applications.

3- Course Objectives:

• Illustrating the philosophy of the differential equations and the importance of solving the differential
equations as an essential part of the mathematical background of engineers.
• Demonstrating the basic definitions and principals of multiple integrals (double and triple integrals).
3
• Demonstrating the concepts of Laplace transformation and inverse Laplace transformation for solving
differential equations in Electrical Engineering applications.
• Demonstrating the basic principles of infinite sequences and series and different tests of convergence
of some important type of series.
• Demonstrating the fundamentals of Fourier series and the difference between Fourier series and
Taylor expansion.
• Realizing the fundamentals of analytical geometry.

4- Relationship between the course and the program

NARS-Engineering Competency Framework


Field Level-A Level-B Level-C
General Specialty Sub-Specialty

Program Academic
Standards that the course A1, A8, A9 ------ -------
contributes in achieving

5- Competencies Based Education (CBEs)


Program CBEs that the course Course CBEs
Field
contributes in achieving
A1) Identify, formulate, and solve a1-1) Discuss the differential equations
complex engineering problems according to order and degree.
by applying engineering
fundamentals, basic science and a1-2) Explain the solvig of different
mathematics. types of ordinary differential
equations.
a1-3) Identify main idea for solving first
order first degree method.
a1-4) Report the basic principles of
multiple integrals.
Level-A
A8) Communicate effectively – a8-1) Explain Methodologies for solving
General
graphically, verbally and in writing differential equation using Laplace
– with a range of audiences using Transformation.
contemporary tools.
a8-2) Discuss infinite series and its
engineering applications.
a8-3) Explain Methodologies of Fourier
series.

A9) Use creative, innovative and a9-1) Organize the solutions obtained
flexible thinking and acquire by different methods of solving
entrepreneurial and leadership skills to differential equations.

4
anticipate and respond to new a9-2) Solve problems serve to illustrate
situations. the studied equations.
a9-3) Analyse the fundamentals of
Laplace Transformation in
engineering applications.

a9-4) Employ a mathematics models


and solve problems in engineering
applications.

6- CBEs AND OBJECTIVES MATRIX

Course Objectives
Field Competencies Based Education (CBEs)
1 2 3 4 5 6

a1-1) Discuss the differential equations


according to order and degree. x x

a1-2) Explain the solvig of different types of


ordinary differential equations.
. x x

a1-3) Identify main idea for solving first order


first degree method.

Level-A x x
General
a1-4) Report the basic principles of multiple
integrals. x

a8-1) Explain Methodologies for solving


differential equation using Laplace
Transformation. x

a8-2) Discuss infinite series and its engineering


applications.
x

a8-3) Explain Methodologies of Fourier series x

5
a9-1) Organize the solutions obtained by
different methods of solving differential
equations.

a9-2) Solve problems serve to illustrate the


x x
studied equations.

a9-3) Analyse the fundamentals of Laplace


Transformation in engineering x
applications.
a9-4) Employ a mathematics models and solve
problems in engineering applications. x

7- Course Topics.
Topic No. General Topics Weeks

1st Introduction 1

2nd Ordinary differential equations. 2-5

3rd Multiple integrals 6-7

4th Laplace transform 9-10

5th Infinite series 11

6th Fourier analysis 12-13

7th Analytical Geometry 14-14

8- CBEs MATRIX TOPICS

Course Topics

Field Competencies Based Education (CBEs) 7t


1st 2nd 3rd 4th 5th 6th h

a1-1) Discuss the differential equations x


according to order and degree. x

Level-A
a1-2) Explain the solving of different types of
General
ordinary differential equations.
x
.

6
a1-3) Identify main idea for solving first order
first degree method.
x

a1-4) Report the basic principles of multiple


integrals. x

a8-1) Explain Methodologies for solving


differential equation using Laplace
Transformation. x

a8-2) Discuss infinite series and its engineering


applications.
x

a8-3) Explain Methodologies of Fourier series X


a9-1) Organize the solutions obtained by
different methods of solving differential X
equations.

a9-2) Solve problems serve to illustrate the


X
studied equations.

a9-3) Analyse the fundamentals of Laplace


Transformation in engineering x X
applications.
a9-4) Employ a mathematics models and solve
problems in engineering applications. X

9- Course Topics/hours/ CBEs

Contact hrs
Course
CBEs
Total
Week No. Sub. Topics Covered
Hours Lec. Tut. Lab.
(By
No.)
Introduction, Classification of differential
equations according to order and degree, a1-1,
Week-1 homogenous 6 4 2 -
and no homogenous a1-2
differential equations.

1st order 1st degree differential equations


with linear coefficient 6 4 2 - a1-3
Week-2
(Part I).

7
Bernoulli's differential equations,
Week-3 integrating factor, second order first degree 6 4 2 - a1-4
differential equations.

1st order higher degree differential


equations, homogenous linear differential
Week-4 equations with constant coefficient, non- 6 4 2 - a8-1
homogenous linear differential equations
with constant coefficient.

Eular differential equations.


Orthogonal trajectory
Week-5 6 4 2 - a9-2
System of simultaneous linear differential
equations

Introduction, double integral application of


Week-6 double integral (area, mass). 6 4 2 - a9-1

double integral application of double


Week-7 integral (moment of inertia), triple integral 6 4 2 - a8-1
volume.

Week-8 Midterm of first Term (written examination)

Week-9 Laplace Transformation 6 4 2 - a1-4

Week-10 Inverse Laplace Transformation 6 4 2 - a8-1

Infinite sequence and series.


Comparison test, ratio test, Cauchy test,
integral test, and raabi's test. 6 4 2 - a9-1
Week-11
Alternating series.
Power series.

Basic idea of function expansion. a9-1


Week-12 6 4 2 -
Fourier expansion.

Change of interval, odd and even expansion a8-3


Week-13 of functions. 6 4 2 -

Normal form of the equation of a plane. a9-2,


a9-4
Transformation of the general equation of a 6 4 2 -
Week-14
plane to the normal form. Direction cosines
of normal to a plane.

8
Determination of a plane under given
conditions.
Equation of a plane in terms of its intercepts
on the axes.

Equation of the plane through three given a8-2,


points. a9-1
Systems of planes.

Week-15 Two sides of a plane length of he 6 4 2 -


perpendicular from a given point to a given
plane.
Bisectors of angles between two planes.

10- Teaching and Learning Method:

Competencies Based Education


Presentations and

Lab Experiments

Problem solving

Simulation and
Grope Working
Brain storming

Research and
(CBEs)

Role playing
Discovering
Discussions

Modelling
Site visits
Tutorials

Roprting
Projects
Lecture

Movies

a1-1 X X X X X X

a1-2 X X X X X X

a1-3 X X X X X X

a1-4 X X X X X X

a8-1 X X X X X
Level-A
a8-2 X X X X X
General
a8-3 X X X X X

a9-1 X X X X X

a9-2 X X X X X

a9-3 X X X X X

a9-4 X X X X

9
11- Teaching and Learning Methods for Low Capacity and Outstanding Students:

Assign a portion of the office hours for those students.

Give them specific tasks.

For low capacity students Repeat the explanation of some of the material and
tutorials.

Assign a teaching assistance to follow up the


performance of this group of students.

Hand out project assignments to those students.

Give them some research topics to be searched using


For outstanding Students the internet and conduct presentation.

Encourage them to take parts in the running research


projects.

12- Assessment
12.1 Assessment Methods:

Assessment Methods
Presentatio
Exam

Quiz

Exam
Take Home
Oral Exam

Tutorial

Discussion

Laboratory test
n
assessment

assessment

assessment
assessment

assessment

assessment
Model
Report
Written

Project

Competencies Based
Education (CBEs)

a1-1 X X X

a1-2 X X X

a1-3 X X X

a1-4 X X X

a8-1 X X X X X X
Level-A
a8-2 X X X X X X
General
a8-3 X X X X X X

a9-1 X X X X

a9-2 X X X X

a9-3 X X X X

a9-4 X X X

10
12.2 Assessment Weight, Schedule and Grades Distribution:

Assessment Method Mark Percentage week

Final-Term Examination 100 66.66% 16th

Mid-Term Examination1 (Written) 40 26.66% 8th

Term work (Tutorial and report


10 6.67% Weekly
assessment)

Total 150 100%

13- Facilities required for teaching and learning:


13-1 1Library Usage:
Students should be encouraged to use library technical resources in the preparation of laboratory
reports and oral presentation. At least one oral presentation should involve a significant component
of library research to encourage this component of study.

14- List of references:


14-1Essential books
1-Matthew Hutton, "Vector Analysis Notes", 2006
2-Hamdy A Taha, "Operation Research an introduction", Eighth, 2003
5-Th.Shifrin, "Multivariable Mathematics", wiley, 2005
6-J.H.Hubbard and B.B.Hubbard, "Vector Calculus, Linear Algebra, and differential Forms", (second
edition), Prentice Hall, 2001.
14.2Periodicals, Web sites---- etc.
1-http://en.wikipedia.org/wiki/cuchy-schawarz_inequality
2-www.lix.polytechnique.fr/~liberti/kissing-ctw.ps.gz
3-http://college.cengage.com/mathematics/larson/calculus_analytic/7e/students/
********************************************************************

Course coordinator Head of the Department

Dr. Muhammad Magdy Prof. Dr. Sayed Mohamed Farag

11
COTENTS

CH. NO. Name Page

Chapter I Differential Equations 13

Chapter II Laplace Transform 100

Chapter III Multiple Integral 143

Chapter IV Infinite Sequences and Series 170

Chapter V Fourier Series 189

References 205

12
Preface

Mathematics is the science that concern all formulations, equations, relations, and
procedures required for all fields and applications. Also, mathematics is used to develop
theorems, construct models, and deduce optimum solutions or convenient manipulations
and to formulate the general laws and models required to solve theoretical and applied
problems. Mathematics is divided into two main parts: pure mathematics which is
dedicated to the studies of theoretical or abstract mathematics which has no physical
interpretation, and applied mathematics which is dedicated to the studies of the applied
sciences concerning the physical and Engineering concepts such as: mechanics, the theory
of fields, automatic control, signals, mathematical physics, Engineering mathematics, …
and many other applications. The third part is computational mathematics which
dedicated to adapt data for the numerical analysis processes and applied numerical
programming.

Otherwise, all branches mathematics are classified as follows: algebra (linear algebra,
Boolean algebra, abstract algebra, complex numbers, operations research, numerical
analysis…). Geometry (analytic geometry, descriptive geometry, differential geometry,
projective geometry…). Calculus (differentiation, partial differentiation, ordinary
differential equations, complex differentiation, computation differentiation, multiple
integrals, complex integral, computational integral…).

Therefore, this course is dedicated to first grade students of electrical Engineering


department to assist them for the understanding of basic concepts and fundamental topics
in Engineering mathematics. First chapter includes ordinary differential equations and its
applications (basic concepts and fundamental definitions, formulation and types,
conditions and methods of solution of some kinds, some applications). Second chapter
shows Laplace transforms (importance, Laplace transforms, inverse Laplace transform,
properties and rules, solution of conditional differential equations). Third chapter
discusses Infinite sequences and series (basic definitions, series and summation notation,
13
convergence and divergence tests of series, power series). Fourth and last chapter
presents Fourier series (background, Fourier's theorem, periodic functions, Fourier series).

The objective of this course

This course is devoted to the first-grade students to assist them for the
understanding of basic concepts and fundamental topics in engineering
mathematics and consequently solving related engineering problems.

14
Chapter One

Ordinary Differential equations

Contents
• Introduction
• First Order First Degree Ordinary Differential Equations
• Second Order First Degree Ordinary Differential Equations
• Differential equations of First Order but not First Degree (Singular Solutions)
• Homogeneous Linear Differential Equations with Constant Coefficients
• Non-Homogeneous Linear Differential Equations with Constant Coefficients
• Euler Cauchy Differential Equations

Applications on Ordinary Differential Equations


• Orthogonal Trajectories
• Systems of Simultaneous Differential Equations

Basic concepts and definitions


Differential equations
The differential equation is an equation relates some functions and its derivatives. If the
real function has only one real dependent variable and its derivatives are ordinary, the
equation is known as ordinary differential equation (ODE). Otherwise, if the real function
has more than one real dependent variable and its derivatives been partial derivatives, the
equation is known as partial differential equation (PDE). Commonly, differential equation
is regarded ordinary differential equation.
A differential equation (D.E.) is one, which connects the independent variable 𝑥,
unknown function 𝑦 = 𝑓(𝑥) and at least one of the derivatives of the unknown function,
may be written in general form as follows:
𝑑𝑦 𝑑 2 𝑦 𝑑𝑛 𝑦
𝑓 (𝑥, 𝑦, , 2 , … … … . , 𝑛 ) = 0
𝑑𝑥 𝑑𝑥 𝑑𝑥

15
For example, the equation 𝑑𝑦/𝑑𝑥 = 𝑥 is a D.E. with the unknown function 𝑦(𝑥), clearly
𝑦 = 𝑥 2 /2 is a solution since satisfies the differential equation sense the left and the right
hands sides of the equation becomes identical. If we add any constants for the above
equation (i.e.) 𝑦 = 𝑥 2 /2 + 𝑐 we get the general solution of the differential equation for all
values of c, so the differential equation may have infinite solutions.
The equation is a differential equation in which the unknown function is a function of two
variable 𝑥 and 𝑦, in this case the equation is called partial differential equation. The
function 𝑢(𝑥, 𝑦) = 𝑠𝑖𝑛(𝑥) + 𝑓(𝑦) is a solution of the above partial differential equation or
all 𝑥 and 𝑦. In this chapter we shall restrict our study to the ordinary differential equations
(O.D.E.)
This chapter is dedicated to assist the students to understand and to learn some
fundamental topics of differential equations and there engineering applications. These
topics include (basic concepts, basic definitions, formulation, types of solution, conditions
and methods of solution, and some engineering applications). The main objectives of this
chapter are listed below to learn the students how to:
i) formulate the differential equation from a phenomenon or from an engineering
system depending on a defined function by deriving a formula which relates this
function with its differentials.
ii) classify, solve, and check the solution of the differential equation.
iii) formulate simpler differential equation and how to select simple and exact
solution.

Order of D.E.
The highest order derivative involved in any D.E. is called the order of the DE.

Degree of D.E.
The exponent of the highest order derivative involved in any D.E. is called the degree of
the DE.

Linear and non-linear O.D.E.


The D.E. 𝑓(𝑥, 𝑦, 𝑦′, 𝑦′′, … … . . ) = 0 is said to be linear if
1- The dependent variable y and its derivatives are of the first degree
2- The coefficients of the dependent variable y and its derivatives are functions of
independent variable x (or constant). Otherwise is said to be non-linear
16
Example
Define the order and the degree of the following D.E.:
𝑑𝑦
1) − 3𝑥𝑦 − 4 = 0
𝑑𝑥
𝑑2𝑦 𝑑𝑦
2) +2 − cos(𝑥) = 0
𝑑𝑥 2 𝑑𝑥
2
𝑑2𝑦 𝑑𝑦
3) ( 2 ) + 3 − 2𝑥𝑦 = 𝑒 𝑥
𝑑𝑥 𝑑𝑥
𝜕2 𝑢 𝜕2 𝑢
4) 2
+ − 2𝑢 = sin(𝑥) sin⁡(𝑦)
𝜕𝑥 𝜕𝑦 2

Equation (1) is of the first order and first degree, equation (2) is of the second order and
first degree, equation (3) is of the second order and second degree, while the last is partial
differential equation of the second order and first degree.

Example
Define linear and non-linear of the following D.E.
𝑦 ′ + 2𝑥𝑦 = 𝒆𝒙 linear
𝑦′′ + 3𝑥𝑦′ = 𝑐𝑜𝑠(𝑥) linear
𝑦′′′′ + 𝑥 3 𝑦′′′ − 2𝑥𝑦′ + 4𝑦⁡𝑐𝑜𝑠(𝑥) = 𝑥⁡𝑠𝑒𝑐(𝑥) linear
𝑦 ′′ + 2𝑥𝑦 ′ + 𝑦 2 = sin(𝑥) non-linear
𝑦 ′′ + (𝑦 ′ )2 + 𝑦 = tan(𝑥) non-linear
𝑦 ′′ + 𝑦𝑦 ′ − 2𝑦 = 𝑥 2 non-linear
𝑦′′ + 2𝑥𝑦′ + 𝑦 = 𝑐𝑜𝑠(𝑦) non-linear
𝑑𝑦 = (𝑥 + 𝑠𝑖𝑛𝑥)𝑑𝑥 (first order first degree L.D.E.)
𝑑𝑦 3 𝑑𝑦 2 𝑑𝑦 4
3
+ + ( ) = sin⁡(𝑥) (3rd order 1st degree N.D.E.)
𝑑𝑥 𝑑𝑥 2 𝑑𝑥
2
𝜕𝑦 𝜕2 𝑦
= 5( ) (2nd order 2nd degree N.L.D.E.)
𝜕𝑥 𝜕𝑥 2

𝜕2 𝑢 𝜕2 𝑢
= 𝑎2 (2nd order 1st degree L.P.D.E.)
𝜕𝑡 2 𝜕𝑥 2

𝜕2 𝑢 𝜕2 𝑢 𝜕2 𝑢
𝑢 = 𝑎2 + (N.L.P.D.E.)
𝜕𝑡 2 𝜕𝑥 2 𝜕𝑦 2

17
𝑦′⁡ − ⁡2𝑦⁡ = ⁡0 (1st order, 1st degree LDE)
𝑦⁡𝑦′⁡ + ⁡𝑥⁡ = 0⁡⁡⁡⁡ (N.L.D.E.)
𝑦′′⁡ + ⁡𝑥⁡𝑦′⁡ = 𝑥 2 (2nd order, 1st degree LDE)
(𝑦′)2⁡ + ⁡𝑥𝑦 2 ⁡ = 𝑒 𝑥 ⁡ (N.L.D.E.)

Homogeneous and non- homogeneous D.E.


The 𝑛𝑡ℎ order linear differential equation;
𝑑𝑛 𝑦 𝑑 𝑛−1 𝑦
𝑎0 𝑛 + 𝑎1 𝑛−1 + ⋯ + 𝑎𝑛 𝑦 = 𝑝(𝑥)
𝑑𝑥 𝑑𝑥
is said to be homogeneous if 𝑝(𝑥) = 0, otherwise is non-homogeneous. The methods of
solution are classified according to the kind of the differential equation (Homog. Or Non-
Homog)

Definition
The solution of a D.E. is any function 𝑦 = 𝑓(𝑥) which, when put into the equation converts
it into an identity.
Example
𝑦′𝑥 − 𝑥 2 − 𝑦 = 0
Its solutions are the functions of the form:
𝑦 = 𝑥 2 + 𝑐𝑥
Where c is a constant. Differentiating these functions with respect to x we find,
𝑦′ = 2𝑥 + 𝑐
Putting 𝑦 and 𝑦′ into the initial equation we get the identity,
𝑥(2𝑥 + 𝑐) − ⁡ 𝑥 2 − 𝑥 2 − 𝑐𝑥 = 0

18
Formulation of D.E. from a known function
The D.E. is formulated from known function by deriving an equation that relates this
function with its derivatives. In all cases, the range of the function that express the solution
of the D.E. must be defined. The followings are some functions and its corresponding D.E.s.
𝑦⁡ = 𝑒 2𝑥 ,⁡⁡⁡⁡⁡⁡⁡⁡⁡⁡⁡⁡⁡⁡⁡⁡⁡⁡⁡⁡⁡⁡⁡⁡⁡⁡⁡⁡⁡⁡⁡𝑦′ − 2𝑦⁡⁡ = ⁡0

𝑦 = √1 − 𝑥 2 ⁡𝑏,⁡⁡⁡⁡⁡⁡⁡⁡⁡⁡⁡⁡⁡⁡⁡⁡⁡⁡⁡𝑦𝑦′ + 𝑥 = 0
𝑦⁡ = 𝑠𝑖𝑛−1 (𝑥),⁡⁡⁡⁡⁡⁡⁡⁡⁡⁡⁡⁡⁡⁡⁡⁡⁡⁡⁡(1 − 𝑥 2 )𝑦′′ − 𝑥𝑦′ = 0
𝑦 = 𝑒 −2𝑥 ⁡𝑠𝑖𝑛⁡(3𝑥),⁡⁡⁡⁡⁡⁡⁡⁡⁡⁡⁡⁡⁡𝑦′′ + 4𝑦′ + 13𝑦 = 0
If the given function includes some arbitrary constants, differentiation can be used to
eliminate these constants or to determine its values, then the D.E. can be derived without
including any constants, the followings are some examples
𝑦 = 𝑐𝑒 𝑥 + 𝑐2 , 𝑦′ = 𝑐𝑒 𝑥 ⁡⁡ → ⁡⁡𝑦 = ⁡𝑦′ + 𝑒 −2𝑥 (𝑦′)2
𝑦⁡ = 𝐴⁡𝑒 𝑥 + 𝐵𝑒 −𝑥 , 𝑦 ′ = 𝐴⁡𝑒 𝑥 − 𝐵𝑒 −𝑥 , 𝑦′′ = 𝐴⁡𝑒 𝑥 + 𝐵𝑒 −𝑥 ⁡ → ⁡𝑦′′ − 𝑦 = 0

Types of constants
The constants are divided into two types: essential or arbitrary constants, and non-
essential constants. Essential or arbitrary constants cannot be merged, while merged
constants are known as non-essential constants, the followings are some examples
𝑓(𝑥) = 𝑎⁡𝑠𝑖𝑛⁡𝑥 + ⁡𝑏⁡𝑐𝑜𝑠⁡𝑥
the constants a and b are both essential arbitrary constants.
𝑔(𝑥) = 𝐴𝑒 𝐵+𝑥 ⁡ = (𝐴𝑒 𝐵 )⁡𝑒 𝑥 = 𝐶⁡𝑒 𝑥
the constants 𝐴 and 𝐵 are both non-essential constants.

Types of solutions of D.E.


Prior to starting the solution procedures of any D.E. one must distinguish all types of the
predicted solutions as well as classification of the given equation. Below all types of the
available solution of D.E.s are defined.

19
i. Unique solution
It is represented by a constant unique function. As example, the D.E. (y/)2 + y2 = 0 has a
unique solution y=0, which is known as zero solution.
ii. General solution
It the solution of the D.E. of order 𝑛 which include 𝑛 essential constants. As an example,
the DE 𝑦′′ − ⁡6𝑥 = 0 of second order, its solution includes two essential constants and is
written 𝑦 = 𝑥 3 + 𝑎𝑥 + 𝑏.
iii. Singular and complete solution
The singular solution is an independent solution of the general solution for the same
D.E. which does not include any constants. Otherwise, the complete solution is considered
a general solution where no any singular solution exists. For examples, the D.E. 𝑦′ = 𝑐𝑜𝑠⁡𝑥
has a complete solution 𝑦 = 𝑠𝑖𝑛⁡𝑥 + 𝑐, where no singular solution exists. While the 1st
order, 2nd degree DE (𝑦′)2 − 𝑥𝑦′ + 𝑦 = 0⁡ℎ𝑎𝑠⁡𝐺𝑆⁡𝑦 = 𝑐𝑥 − 𝑐2 , where 𝑐 is essential
constant, and the solution is not complete because the DE has the singular solution⁡𝑦 =
𝑥 2 /4 which is independent of the GS.
iv. Particular solution
The particular solution PS for any D.E. is the solution that can be derived from the GS by
determining particular values for the essential arbitrary constants by using the available
initial conditions. As example, if the GS of the D.E. 𝑦′′ + 𝑦 = 0 is 𝑦 = 𝑎⁡𝑠𝑖𝑛⁡𝑥 + 𝑏⁡𝑐𝑜𝑠⁡𝑥,
the corresponding PS to the conditions 𝑦(0) = 3 and 𝑦′(0) = 2⁡𝑖𝑠⁡𝑦 = 2𝑠𝑖𝑛⁡𝑥 +
3𝑐𝑜𝑠⁡𝑥⁡where: 𝑦′(0) = 2⁡⁡𝑎 = 2⁡𝑎𝑛𝑑⁡𝑦(0) = 3⁡⁡𝑏 = 3. Obtaining the PS requires
number of associated conditions equals to the number of the arbitrary constants included
in the GS.

Physical meaning of Differential Equations


Differential equations arise in constructing mathematical models for solving problems
in different areas in engineering and physics.
1- Electricity
If 𝐼(𝑡) denotes the current flowing in an electric circuit at a time 𝑡 when an
electromotive force 𝑉(𝑡) = 𝐸𝑠𝑖𝑛(𝜔𝑡) is applied to an 𝐿 − 𝑅 − 𝐶 circuit as shown in the
following figure

20
The sum of drop of the voltage is equal to the external electromotive force in a series
circuit we get:
𝑑𝐼 𝑄
𝐿 + 𝑅⁡𝐼 + = 𝐸𝑠𝑖𝑛(𝜔𝑡)
𝑑𝑡 𝐶
𝑑𝑄
Since, 𝐼 = we get:
𝑑𝑡

𝑑2𝑄 𝑑𝑄 𝑄
𝐿 2 +𝑅 + = 𝐸𝑠𝑖𝑛(𝜔𝑡)
𝑑𝑡 𝑑𝑡 𝐶
The last equation is a differential equation where the charge on the condenser 𝑄(𝑡) is
the unknown variable and 𝑡 is the independent variable.
2- Mechanics
If 𝑥(𝑡) is the vertical distance of a rigid body with mass 𝑚 from the earth surface at a
time t. the body is thrown upwards, if the air resistance per unit mass is 𝒌𝒗 where 𝒗 is
the velocity of the rigid body and 𝑘 is a constant. Applying the third Newton's law we
get:
𝑑𝑣
𝑚 = −𝑚𝑔 − 𝑚𝑘𝑣
𝑑𝑡
𝑑𝑣
𝑚 = −𝑚(𝑔 + 𝑘𝑣)
𝑑𝑡
This equation gives the relation between the velocity and the time. Using the relation
between the velocity and the distance 𝑣 = 𝑑𝑥/𝑑𝑡 the last equation becomes:
𝑑2𝑥 𝑑𝑥
= −𝑔 − 𝑘
𝑑𝑡 2 𝑑𝑡
The last equation is a differential between the distance 𝑥 and the time 𝑡.
3- Geometrical application
To find an equation of curves which satisfy certain restrictions on the slope, curvature
or both at each point. For example, if the slope at each point (𝑥, 𝑦) of the curves equal
to −𝑦/𝑥 then:
𝑑𝑦 𝑦
=−
𝑑𝑥 𝑥
From calculus, one can deduce that such curves are given by:
21
𝑦 = 𝑐/𝑥⁡⁡⁡⁡𝑜𝑟⁡⁡⁡𝑥𝑦 = 𝑐
This is a family of rectangular hyperboles

Exercise 1
1. Show that the following formulations define functions that represent solutions of ach
corresponding equations
𝑖.⁡⁡⁡𝑦⁡ = ⁡𝑠𝑖𝑛𝜃⁡⁡⁡⁡⁡⁡⁡⁡⁡⁡⁡⁡⁡⁡⁡⁡⁡𝑦′′ + ⁡𝑦⁡ = ⁡0
𝑖𝑖.⁡⁡𝐼⁡ = ⁡ 𝑒 3𝑡 ⁡⁡⁡⁡⁡⁡⁡⁡⁡⁡⁡⁡⁡⁡⁡⁡⁡⁡⁡⁡⁡⁡⁡⁡⁡⁡⁡𝐼 ̈ − 9𝐼 = 0
𝑖𝑖𝑖.⁡⁡𝑦 = 0.5𝑥 3 + 5𝑥⁡⁡⁡⁡⁡⁡⁡𝑥𝑦′ − ⁡𝑥3⁡– ⁡𝑦⁡ = ⁡0
𝑖𝑣.⁡⁡𝑟 = ⁡𝑎⁡𝑠𝑖𝑛⁡𝜃⁡ + ⁡𝑏𝑐𝑜𝑠⁡𝜃⁡⁡⁡⁡⁡⁡⁡⁡⁡𝑟′′ + ⁡𝑟⁡ = ⁡0
2
𝑣. 𝑦 = 𝑥 3 + 𝑒 𝑥 ⁡⁡⁡⁡⁡⁡⁡⁡⁡⁡𝑦′⁡ − ⁡2𝑥𝑦⁡ = ⁡3𝑥 2 ⁡ − ⁡2𝑥 4
2. Determine the GS of the following D.E.
𝑖.⁡⁡𝑦′ − 𝑥 2 ⁡ + ⁡2𝑥⁡ = ⁡0
𝑖𝑖. 𝑦̇ − 𝑒 𝑡 = 0⁡⁡
𝑖𝑖𝑖.⁡⁡𝑟′′ − ⁡𝑐𝑜𝑠𝜃⁡ = ⁡0
𝑖𝑣. 𝑦′′′ − ⁡𝑥⁡ + 1 = ⁡0
3. Determine each type and each order of the D.E.:
𝑖.⁡⁡𝑦̈ + 𝑡𝑦 = 0⁡
𝑖𝑖.⁡⁡𝑟′′′ + ⁡𝑐𝑜𝑠⁡𝜃⁡ = ⁡0
𝑖𝑖𝑖. (𝑦′′)2 + 3(𝑦′)3 = ⁡0
𝑖𝑣.⁡⁡𝑦′⁡ + 𝐿𝑛⁡(𝑦) ⁡ − 1⁡ = 0
4. Eliminating the constants derive the D.E. for each function
𝑖.⁡⁡𝑦⁡ = ⁡𝐴𝑒 𝑥 ⁡ + ⁡𝐵𝑒 −𝑥
𝑖𝑖.⁡⁡𝑦⁡ = ⁡𝐴𝑡⁡ + ⁡𝐵⁡𝑐𝑜𝑠⁡𝑡⁡ + ⁡𝐶⁡𝑠𝑖𝑛⁡𝑡
5. Derive second order D.E. from the formula
𝑦 2 + ⁡𝑎𝑥 2 ⁡ + ⁡𝑥⁡ = ⁡0

22
6. Deduce the solutions of the D.E.
i. ⁡(𝑦′′)4 ⁡ + 𝑦 6 ⁡ + ⁡5⁡ = ⁡0
ii. (𝑦′)6 ⁡ + 𝑦 4 = 0
7. Verify that each given function is a solution of the corresponding DE, specify the type of
each solution and determine the real constant c in such a way that each solution satisfies
the corresponding conditions.
𝑖.⁡⁡𝑦 = 1 + 𝑐𝑒 −𝑥 ⁡⁡⁡⁡⁡⁡⁡⁡𝑦′ + 𝑦 = 1⁡⁡⁡⁡⁡⁡⁡⁡⁡⁡⁡⁡⁡⁡⁡⁡𝑦 = 5/2⁡𝑤ℎ𝑒𝑛⁡𝑥 = 0,
2
𝑖𝑖. 𝑦 = 𝑐𝑒 𝑥 ⁡⁡⁡⁡⁡⁡⁡⁡𝑦′⁡ − 2𝑥𝑦 = 0⁡⁡⁡⁡⁡⁡⁡⁡⁡⁡⁡⁡𝑦 = 4⁡𝑤ℎ𝑒𝑛⁡𝑥 = 1,
𝑖𝑖𝑖. 𝑦 = 𝑐𝑥 2 ⁡⁡⁡⁡⁡⁡⁡⁡⁡⁡⁡𝑥𝑦′ − 2𝑦 = 0⁡⁡⁡⁡⁡⁡⁡⁡⁡⁡⁡⁡⁡𝑦 = 12⁡⁡⁡𝑤ℎ𝑒𝑛⁡⁡⁡⁡𝑥 = 2,
𝑖𝑣.⁡⁡𝑦 2 − 𝑥 2 = 𝑐⁡⁡⁡⁡⁡⁡⁡⁡𝑦𝑦′ − 𝑥 = 0⁡⁡⁡⁡⁡⁡⁡⁡⁡⁡⁡⁡⁡⁡⁡𝑦(0) = 1,
𝑣.⁡⁡𝑦 = 𝑐⁡𝑠𝑖𝑛⁡𝑥⁡⁡⁡⁡⁡⁡⁡𝑦′ − 𝑦⁡𝑐𝑜𝑡⁡𝑥 = 0⁡⁡⁡⁡⁡⁡⁡𝑦(−/2) = 2,

𝑣𝑖. 𝑦 2 + 𝑥 2 = 𝑐⁡⁡⁡⁡⁡⁡⁡𝑦𝑦′ + 𝑥 = 0⁡⁡⁡⁡⁡⁡⁡⁡⁡⁡⁡⁡⁡⁡𝑦(2) = ⁡2

23
The differential equations can be classified according to the order and the degree
into the following types:
𝒅𝒚
I- Fisrt Order Fisrt Degree D.Eqs. ( )
𝒅𝒙
1- Separable variables
2- Homogeneous differential equation
3- Exact differential equation
4- Equations with linear coefficients
5- Linear equation
6- Bernoulli's equation

𝒅𝟐 𝒚
II- Second Order First Degree D.Eqs. ( )
𝒅𝒙𝟐
1- Not contain y explicitly
2- Not contain x explicitly
3- Not contain x and y explicitly

𝒅𝒚 𝟐
III- First Order Higher Degree D.Eqs. ( )
𝒅𝒙
1- Solvable for p, where 𝑝 = 𝑑𝑦/𝑑𝑥
2- Solvable for 𝑥
3- Solvable for 𝑦

First Order First Degree Ordinary Differential Equations


The equation may be written in one of the forms:
𝑑𝑦
𝑃(𝑥, 𝑦) + 𝑄(𝑥, 𝑦)⁡ =0
𝑑𝑥
𝑃(𝑥, 𝑦)𝑑𝑥 + 𝑄(𝑥, 𝑦)⁡𝑑𝑦 = 0
𝑑𝑦 𝑃(𝑥, 𝑦)
=
𝑑𝑥 𝑄(𝑥, 𝑦)
Now, we shall discuss the standard forms of this type

1- Separable Differential Equations


i) Direct method of separation of variables method
Letting the first order first degree takes the form
24
𝑃(𝑥)𝑑𝑥 + 𝑄(𝑦)𝑑𝑦 = 0
where, 𝑝(𝑥) is a function of 𝑥 only, and 𝑄(𝑦) is a function of 𝑦 only. Integrating the
first term with respect to 𝑥 and the second term with respect to 𝑦 gets the general
solution.

∫ 𝑃(𝑥) 𝑑𝑥 + ∫ 𝑄(𝑦) 𝑑𝑦 = ∫ 0

∫ 𝑃(𝑥)𝑑𝑥 ⁡ = ∫ 𝑄(𝑦)𝑑𝑦 ⁡ + ⁡𝑐

Example: Solve the D.E.


𝑑𝑦 𝑦
=−
𝑑𝑥 𝑥
Separate variables, we have;
𝑑𝑥 𝑑𝑦
+ =0
𝑥 𝑦
Integrating both sides we get the general solution:
l n(𝑥) + l n(𝑦) = l n(𝑐)
l n(𝑥𝑦) = l n(𝑐)
𝑥𝑦 = 𝐶
is the general solution, where 𝐶 is a constant.

Example: Solve the D.E.


(𝑥 2 − 3𝑥 + 6)𝑑𝑥 − (𝑦 3 − 2𝑦 + 4)𝑑𝑦 = 0
Integrating the first term with respect to 𝑥 and the second term with respect to 𝑦 we have;

∫(𝑥 2 − 3𝑥 + 6) 𝑑𝑥 − ∫(𝑦 3 − 2𝑦 + 4) 𝑑𝑦 = ∫ 0

Then:
𝑥3 3 2 𝑦4
− 𝑥 + 6𝑥 − ( − 𝑦 2 + 4𝑦) = 𝐶
3 2 4

25
Is the general solution and 𝐶 is a constant.

Example: Solve the D.E.


𝑑𝑦
(𝑥 2 + 1) + (𝑦 2 + 1) = 0
𝑑𝑥
Separating the variables, we have:
𝑑𝑦 𝑑𝑥
+ =0
(𝑦 2 + 1) (𝑥 2 + 1)
Integrate both sides we get the general solution in the form
𝑡𝑎𝑛−1 (𝑦) + 𝑡𝑎𝑛−1 (𝑥) = 𝐶
where C is a constant

Example: Classify and solve the equation, then check your result
𝑑𝑦
= 𝑒 𝑥−𝑦 + 𝑥 2 𝑒 −𝑦
𝑑𝑥
The equation is first order first degree D.E.

𝑑𝑦
= 𝑒 𝑥 𝑒 −𝑦 + 𝑥 2 𝑒 −𝑦 = 𝑒 −𝑦 (𝑒 𝑥 + 𝑥 2 )
𝑑𝑥
𝑒 𝑦 ⁡𝑑𝑦 = (𝑒 𝑥 + 𝑥 2 )⁡𝑑𝑥
The equation is first order first degree D.E. Integrating both sides gives the general solution
(complete).

𝑦
𝑥3 𝑥
𝑒 ⁡= 𝑒 + + 𝐶
3
Example: Classify and solve the equation, then check your result
𝑑𝑦
ln ( ) = 𝑎𝑥 + 𝑏𝑦
𝑑𝑥
This equation may take the form

26
𝑑𝑦
= 𝑒 𝑎𝑥+𝑏𝑦 = 𝑒 𝑎𝑥 ⁡𝑒 𝑏𝑦
𝑑𝑥
𝑒 −𝑏𝑦 ⁡𝑑𝑦 = 𝑒 𝑎𝑥 ⁡𝑑𝑥
The equation is first order first degree D.E. Integrating both sides gives the general solution
(complete).
−1 1
𝑒 −𝑏𝑦 = 𝑒 𝑎𝑥 + 𝐶 (try to check)
𝑏 𝑎

Example: Classify and solve the equation, then check your result
𝑑𝑦 𝑥 2 + 2
=
𝑑𝑥 𝑦
This equation may take the form
𝑦⁡𝑑𝑦⁡ = ⁡ (𝑥 2 + 2)⁡𝑑𝑥
The equation is first order first degree DE. Integrating both sides gives the general solution
(complete).
𝑦2 𝑥3
= + 2𝑥 + 𝐶 (try to check)
2 3

ii. Separation of variables by transformation


𝑑𝑦
The substitution 𝑎𝑥 + 𝑏𝑦 + 𝑐 = 𝑢 is used to reduce equations of the form = 𝑓(𝑎𝑥 +
𝑑𝑥
𝑑𝑦
𝑏𝑦 + 𝑐), and the substitution 𝑎𝑥 + 𝑏𝑦 = 𝑣 is used to reduce equations of the forms =
𝑑𝑥
𝑔(𝑎𝑥 + 𝑏𝑦) then, the variables are separated for solution requirements.

Example: Classify and solve the equation, then check your result
𝑑𝑦
= (4𝑥 + 𝑦 + 1)2 ⁡⁡⁡⁡⁡⁡⁡⁡⁡⁡⁡⁡⁡⁡⁡⁡⁡⁡⁡⁡⁡⁡⁡⁡⁡⁡⁡⁡⁡⁡⁡⁡⁡⁡⁡⁡⁡⁡⁡(𝑖)
𝑑𝑥
Using the substitution 4𝑥 + 𝑦 + 1 = 𝑢⁡⁡⁡⁡⁡⁡⁡⁡⁡⁡⁡⁡⁡⁡(𝑖𝑖)
Differentiating (𝑖𝑖) with respect to 𝑥 gives
𝑑𝑦 𝑑𝑢
= − 4⁡⁡⁡⁡⁡⁡⁡⁡⁡⁡⁡⁡⁡⁡⁡⁡⁡⁡⁡⁡⁡⁡⁡⁡⁡⁡(𝑖𝑖𝑖)
𝑑𝑥 𝑑𝑥
Substituting from (𝑖𝑖𝑖)⁡𝑖𝑛⁡(𝑖) gives

27
𝑑𝑢
= 4 + 𝑢2 ⁡⁡⁡⁡⁡⁡⁡⁡⁡⁡⁡⁡⁡⁡⁡⁡⁡⁡⁡⁡⁡⁡⁡⁡⁡⁡(𝑖𝑣)
𝑑𝑥
Now separating variables in (𝑖𝑣) gives
𝑑𝑢
𝑑𝑥 = ⁡⁡⁡⁡⁡⁡⁡⁡⁡⁡⁡⁡⁡⁡⁡⁡⁡⁡⁡⁡⁡⁡⁡⁡⁡⁡⁡⁡⁡⁡⁡⁡(𝑣)
4 + 𝑢2
Integrating both sides of (𝑣) gives the GS
𝑢
𝑥 = 0.5𝑡𝑎𝑛−1 ( ) + 𝐶
2

4𝑥+𝑦+1
𝑥 = 0.5𝑡𝑎𝑛−1 ( )+𝐶 (try to check)
2

Example: Classify and solve the equation, then check your result
𝑑𝑦
= cos(𝑥 + 𝑦)⁡⁡⁡⁡⁡⁡⁡⁡⁡⁡⁡⁡⁡⁡⁡⁡⁡⁡⁡⁡⁡⁡⁡⁡⁡⁡⁡⁡⁡⁡⁡⁡(𝑖)
𝑑𝑥
𝑑𝑦 𝑑𝑣
Letting 𝑥+𝑦 =𝑣  = − 1⁡⁡⁡⁡⁡⁡⁡⁡⁡⁡⁡⁡⁡⁡⁡(𝑖𝑖)
𝑑𝑥 𝑑𝑥

Substituting from (𝑖𝑖)⁡𝑖𝑛⁡(𝑖) gives


𝑑𝑣
= 1 + cos(𝑣)⁡⁡⁡⁡⁡⁡⁡⁡⁡⁡⁡⁡⁡⁡⁡⁡⁡⁡⁡⁡⁡⁡⁡(𝑖𝑖𝑖)
𝑑𝑥

Separating the variables in (𝑖𝑖𝑖) gives


𝑑𝑣 𝑑𝑣
𝑑𝑥 = =
1 + cos(𝑣) 2⁡𝑐𝑜𝑠 2 (𝑣)
2

𝑣
𝑑𝑥 = 0.5𝑠𝑒𝑐 2 ( ) 𝑑𝑣⁡⁡⁡⁡⁡⁡⁡⁡⁡⁡⁡⁡⁡⁡⁡⁡⁡⁡⁡⁡⁡⁡⁡⁡⁡⁡⁡⁡(𝑖𝑣)
2
Integrating both sides of (𝑖𝑣) gives

⁡𝑥 = 𝑡𝑎𝑛(𝑣/2) ⁡ + ⁡𝑐⁡  𝑥 = ⁡𝑡𝑎𝑛[(𝑥 + 𝑦)/2] + 𝑐⁡ (try to check)

Example: Classify and solve the equation, then check your result
𝑑𝑦 4𝑥 + 6𝑦 + 5 2(2𝑥 + 3𝑦) + 5
= = ⁡⁡⁡⁡⁡⁡⁡⁡⁡⁡⁡⁡⁡⁡⁡⁡⁡⁡(𝑖)
𝑑𝑥 2𝑥 + 3𝑦 + 4 (2𝑥 + 3𝑦) + 4
28
𝑑𝑦 𝑑𝑣
Letting 2𝑥 + 3𝑦 = 𝑣⁡  2 + 3 =
𝑑𝑥 𝑑𝑥

𝑑𝑦 1 𝑑𝑣
= ( − 2)⁡⁡⁡⁡⁡⁡⁡⁡⁡⁡⁡⁡⁡⁡⁡⁡⁡⁡⁡⁡⁡⁡⁡⁡(𝑖𝑖)
𝑑𝑥 3 𝑑𝑥
Substituting from (𝑖𝑖)⁡𝑖𝑛⁡(𝑖)⁡gives
1 𝑑𝑣 2𝑣 + 5
( − 2) =
3 𝑑𝑥 𝑣+4
𝑑𝑣 8𝑣 + 23
=
𝑑𝑥 𝑣+4
𝑣+4
𝑑𝑥 = 𝑑𝑣
8𝑣 + 23
1 9
𝑑𝑥 = [ + ] 𝑑𝑣⁡⁡⁡⁡⁡⁡⁡⁡⁡⁡⁡⁡⁡⁡⁡⁡⁡⁡⁡⁡⁡⁡⁡⁡⁡(𝑖𝑖𝑖)
8 8(8𝑣 + 23)
Integrating both sides of (𝑖𝑖𝑖) gives
𝑣 9
𝑥= + ln(8𝑣 + 23) + 𝐶 (try to check)
8 64

2- Homogeneous Differential Equations


The algebraic function 𝑓(𝑥, 𝑦) is called a homogeneous function of degree 𝑛 in two
variables 𝑥⁡𝑎𝑛𝑑⁡𝑦, if for any parameter 𝜆 the following identity is true;
𝑓(𝜆𝑥, 𝜆𝑦) = 𝜆𝑛 𝑓(𝑥, 𝑦)
Example: the function 𝑓(𝑥, 𝑦) = 𝑥 3 − 2𝑥 2 𝑦 + 𝑥𝑦 2 + 3𝑦 3 is homogeneous of degree 3
since;
𝑓(𝜆𝑥, 𝜆𝑦) = 𝜆3 𝑥 3 − 2𝜆3 𝑥 2 𝑦 + 𝜆3 𝑥𝑦 2 + 3𝜆3 𝑦 3
𝑓(𝜆𝑥, 𝜆𝑦) = 𝜆3 (𝑥 3 − 2𝑥 2 𝑦 + 𝑥𝑦 2 + 3𝑦 3 )
𝑑𝑦
A D.E. of the first order = 𝑓(𝑥, 𝑦) is called homogeneous in 𝑥, 𝑦 if the function 𝑓(𝑥, 𝑦)
𝑑𝑥
is a homogeneous function of degree zero.

Solution of a Homogeneous D.E.


We must write the homog. D.E. in the form;

29
𝑑𝑦 𝑀(𝑥, 𝑦)
=
𝑑𝑥 𝑁(𝑥, 𝑦)
Where 𝑀, 𝑁 are algebraic homogeneous functions of the same degree. Using the
substitution:
𝑦 = 𝑣𝑥
where 𝑣 is a function of 𝑥 only, differentiating Eq. (2) 𝑤. 𝑟. 𝑡. 𝑥 we get:
𝑑𝑦 𝑑𝑣
=𝑣+𝑥
𝑑𝑥 𝑑𝑥
Substituting from (2), (3) in (1), gets variable separation

Example: Solve the D.E.


𝑑𝑦 𝑦
𝑥 = 𝑦 − 𝑥⁡𝑐𝑜𝑠 2
𝑑𝑥 𝑥
We must write this equation in the form (1) as follows:
𝑑𝑦 𝑦 𝑦
= − 𝑐𝑜𝑠 2
𝑑𝑥 𝑥 𝑥
This is a homog. Function and we use the substitution (2) and (3), in the given D.E. we get;
𝑑𝑣
𝑣+𝑥 = 𝑣 − 𝑐𝑜𝑠 2 𝑣
𝑑𝑥
Separable the variables,
𝑑𝑥 𝑑𝑣
=− = −𝑠𝑒𝑐 2 𝑣⁡𝑑𝑣
𝑥 𝑐𝑜𝑠 2 𝑣
Integrating both sides we have,
𝑙𝑛(𝑥) = −𝑡𝑎𝑛(𝑣) + 𝐶
𝑦
Where C is a constant, put 𝑣 = then,
𝑥
𝑦
𝑙𝑛(𝑥) = −𝑡𝑎𝑛 ( ) + 𝐶
𝑥
Example: Solve the D.E.
(𝑥 3 + 𝑦 3 )⁡𝑑𝑥 = 2𝑥𝑦 2 ⁡𝑑𝑦

30
The equation is homogeneous D.E.; so, we use the fore mentioned substitution (2), (3) for
the given equation, we get;
𝑦 = 𝑣𝑥
(𝑥 3 + 𝑥 3 𝑣 3 )⁡𝑑𝑥 = 2𝑥 3 𝑣 2 ⁡(𝑥𝑑𝑣 + 𝑣𝑑𝑥)
Rearranging the last equation, and separate the variables, we get;
𝑑𝑥 2𝑣 2
= ⁡𝑑𝑣
𝑥 1 − 𝑣3
Integrating both sides, we have;
2
𝑙𝑛(𝑥) = − ln(1 − 𝑣 3 ) + ln⁡(𝐶)
3
3 ln(𝑥) + 𝑙𝑛⁡(1 − 𝑣 3 )2 = ln(𝐶)
𝑥 3 (1 − 𝑣 3 )2 = 𝐶
𝑦
Put 𝑣 =
𝑥

(𝑥 3 − 𝑦 3 )2 = 𝐶𝑥 3 general solution

3- Exact Differential Equation


The equation
𝑀(𝑥, 𝑦)𝑑𝑥 + 𝑁(𝑥, 𝑦)𝑑𝑦 = 0⁡⁡⁡⁡⁡⁡⁡⁡⁡⁡⁡⁡⁡⁡⁡⁡⁡⁡⁡⁡⁡⁡⁡⁡⁡⁡⁡⁡⁡⁡(1)
Is called an exact if the functions 𝑀(𝑥, 𝑦)⁡𝑎𝑛𝑑⁡𝑁(𝑥, 𝑦) are continuous differentiable
functions for which the following relationship is filled:
𝜕𝑀 𝜕𝑁 𝜕𝑀 𝜕𝑁
= ⁡⁡𝑎𝑛𝑑⁡⁡⁡⁡⁡⁡⁡⁡⁡⁡⁡⁡⁡⁡⁡⁡⁡⁡⁡⁡ , ⁡⁡𝑎𝑟𝑒⁡𝑐𝑜𝑛𝑡𝑖𝑛𝑜𝑢𝑠
𝜕𝑦 𝜕𝑥 𝜕𝑦 𝜕𝑥
If the above condition holds, equation of the form:
𝑑⁡𝑢(𝑥, 𝑦) = 0⁡⁡⁡⁡⁡⁡⁡⁡⁡⁡⁡⁡⁡⁡⁡⁡⁡⁡⁡⁡⁡⁡⁡⁡⁡⁡⁡⁡⁡⁡⁡⁡⁡⁡⁡⁡⁡⁡⁡⁡⁡(2)
and its complete solution is:
𝑢(𝑥, 𝑦) = 𝐶; ⁡⁡⁡⁡⁡⁡⁡⁡⁡⁡⁡⁡⁡⁡⁡⁡⁡⁡⁡⁡⁡⁡⁡⁡⁡⁡⁡𝐶 is constant.
Equation (1), (2) we get,
𝑀(𝑥, 𝑦)𝑑𝑥 + 𝑁(𝑥, 𝑦)𝑑𝑦 = 𝑑𝑢
31
𝜕𝑢 𝜕𝑢
𝑑𝑢 = 𝑑𝑥 + ⁡𝑑𝑦
𝜕𝑥 𝜕𝑦
𝜕𝑢 𝜕𝑢
= 𝑀,⁡⁡⁡⁡⁡⁡⁡ = 𝑁⁡⁡⁡⁡⁡⁡⁡⁡⁡⁡⁡⁡⁡⁡⁡⁡⁡⁡⁡⁡⁡⁡⁡⁡⁡⁡⁡⁡⁡⁡⁡⁡⁡⁡(3)
𝜕𝑥 𝜕𝑦
From (3), we find;

𝑢 = ∫ 𝑀(𝑥, 𝑦)⁡𝑑𝑥 + 𝜙(𝑦)⁡⁡⁡⁡⁡⁡⁡⁡⁡⁡⁡⁡⁡⁡⁡⁡⁡⁡⁡⁡⁡⁡⁡⁡⁡⁡⁡⁡⁡⁡⁡⁡(𝑎)

𝑢 = ∫ 𝑁(𝑥, 𝑦)⁡𝑑𝑥 + 𝜓(𝑦)⁡⁡⁡⁡⁡⁡⁡⁡⁡⁡⁡⁡⁡⁡⁡⁡⁡⁡⁡⁡⁡⁡⁡⁡⁡⁡⁡⁡⁡⁡(𝑏)

The general solution can be obtained from equations (𝑎)⁡𝑎𝑛𝑑⁡(𝑏)


Note:
The sign between the functions of two terms must be positive.

Example: Solve the D.E.


(𝑥 2 + 𝑦)⁡𝑑𝑥 + (𝑥 + 𝑦 2 )⁡𝑑𝑦 = 0
Here,
𝑀(𝑥, 𝑦) = 𝑥 2 + 𝑦 and 𝑁(𝑥, 𝑦) = 𝑥 + 𝑦 2
𝜕𝑀 𝜕𝑁
= 1,⁡⁡⁡⁡⁡⁡⁡ =1
𝜕𝑦 𝜕𝑥
Then, the equation is exact and to get the function 𝑢(𝑥, 𝑦);

𝑢 = ∫ 𝑀(𝑥, 𝑦)⁡𝑑𝑥 + 𝜙(𝑦) = ∫(𝑥 2 + 𝑦)⁡⁡⁡𝑑𝑥 + 𝜙(𝑦)

𝑥3
𝑢= + 𝑥𝑦 + 𝜙(𝑦)⁡⁡⁡⁡⁡⁡⁡⁡⁡⁡⁡⁡⁡⁡⁡⁡⁡⁡⁡⁡⁡⁡⁡⁡⁡⁡⁡⁡(𝑎)
3
Or

𝑢 = ∫ 𝑁(𝑥, 𝑦)⁡𝑑𝑦 + 𝜓(𝑥) = ∫(𝑥 + 𝑦 2 )⁡⁡𝑑𝑦 + 𝜓(𝑥)

32
𝑦3
𝑢= + 𝑥𝑦 + 𝜓(𝑥)⁡⁡⁡⁡⁡⁡⁡⁡⁡⁡⁡⁡⁡⁡⁡⁡⁡⁡⁡⁡⁡⁡⁡⁡⁡⁡⁡⁡⁡⁡⁡⁡(𝑏)
3
From (a) and (b), the general solution is:
𝑥3 𝑦3
𝑢= + + 𝑥𝑦 = 𝐶
3 3
Note:
𝜕𝑢
From (a) we get and equating it with 𝑁 we have,
𝜕𝑦

𝜕𝑢
= 𝑥 + 𝜙 ′ (𝑦) = 𝑁 = 𝑥 + 𝑦 2
𝜕𝑦

′ (𝑦) 2
𝑦3
𝜙 = 𝑦 ,⁡⁡⁡⁡⁡𝜙(𝑦) = ⁡
3
Substitution in (a) we get the general solution,
𝑥3 𝑦3
+ + 𝑥𝑦 = 𝐶; C is a constant
3 3

Example: Solve the D.E.


𝑦
( − 𝑦 sin(𝑥𝑦)) ⁡𝑑𝑥 + (ln(𝑥) − 𝑥 sin(𝑥𝑦))⁡𝑑𝑦 = 0
𝑥

Here,
𝑦
𝑀(𝑥, 𝑦) = − 𝑦 sin(𝑥𝑦) and 𝑁(𝑥, 𝑦) = ln(𝑥) − 𝑥 sin(𝑥𝑦)
𝑥

𝜕𝑀 1
= − sin(𝑥𝑦) − 𝑥𝑦 cos(𝑥𝑦)
𝜕𝑦 𝑥
𝜕𝑁 1
= − sin(𝑥𝑦) − 𝑥𝑦 cos(𝑥𝑦)
𝜕𝑥 𝑥
Then, the equation is exact and to get the function 𝑢(𝑥, 𝑦);
𝑦
𝑢 = ∫ 𝑀(𝑥, 𝑦)⁡𝑑𝑥 + 𝜙(𝑦) = ∫ ( − 𝑦 sin(𝑥𝑦)) 𝑑𝑥 + 𝜙(𝑦)
𝑥

𝑢 = 𝑦 ln(𝑥) + cos(𝑥𝑦) + 𝜙(𝑦)⁡⁡⁡⁡⁡⁡⁡⁡⁡⁡⁡⁡⁡⁡⁡⁡⁡⁡⁡⁡⁡⁡⁡⁡⁡⁡⁡⁡(𝑎)


33
Or

𝑢 = ∫ 𝑁(𝑥, 𝑦)⁡𝑑𝑦 + 𝜓(𝑥) = ∫(ln(𝑥) − 𝑥 sin(𝑥𝑦))⁡𝑑𝑦 + 𝜓(𝑥)

𝑢 = 𝑦 ln(𝑥) + cos(𝑥𝑦) + 𝜓(𝑦)⁡⁡⁡⁡⁡⁡⁡⁡⁡⁡⁡⁡⁡⁡⁡⁡⁡⁡⁡⁡⁡⁡⁡⁡⁡⁡⁡⁡⁡(𝑏)


From (𝑎)⁡𝑎𝑛𝑑⁡(𝑏), the general solution is:
𝑢 = 𝑦 ln(𝑥) + cos(𝑥𝑦) = 𝐶, C is constant

Example: Show that the following equation is exact and find its solution;
(4𝑥 − 3𝑦 − 𝑦 sin(𝑥))𝑑𝑥 + (cos(𝑥) − 3𝑥 − sin(𝑦))𝑑𝑦 = 0
Here,
𝑀(𝑥, 𝑦) = 4𝑥 − 3𝑦 − 𝑦 sin(𝑥) and 𝑁(𝑥, 𝑦) = cos(𝑥) − 3𝑥 − sin(𝑦)
𝜕𝑀
= −3 − sin(𝑥)
𝜕𝑦
𝜕𝑁
= − sin(𝑥) − 3
𝜕𝑥
Then, the equation is exact and to get the function 𝑢(𝑥, 𝑦);

𝑢 = ∫ 𝑀(𝑥, 𝑦)⁡𝑑𝑥 + 𝜙(𝑦) = ∫(4𝑥 − 3𝑦 − 𝑦 sin(𝑥))⁡𝑑𝑥 + 𝜙(𝑦)

𝑢 = 2𝑥 2 − 3𝑥𝑦 + 𝑦 cos(𝑥) + 𝜙(𝑦)⁡⁡⁡⁡⁡⁡⁡⁡⁡⁡⁡⁡⁡⁡⁡⁡⁡⁡⁡⁡⁡⁡⁡⁡⁡⁡⁡(𝑎)


𝜕𝑢
From (𝑎) we get and equating it with 𝑁 we have,
𝜕𝑦

𝜕𝑢
= −3𝑥 + cos(𝑥) + 𝜙 ′ (𝑦) = 𝑁 = cos(𝑥) − 3𝑥 − sin(𝑦)
𝜕𝑦
𝜙 ′ (𝑦) = − sin(𝑦) ,⁡⁡⁡⁡⁡𝜙(𝑦) = 𝑐𝑜𝑠(𝑦) + 𝐶⁡
Substitution in (𝑎) we get the general solution,
𝑢 = 2𝑥 2 − 3𝑥𝑦 + 𝑦cos⁡(𝑥) + cos⁡(𝑦); C is a constant

34
4- Differential Equations with Constant Coefficients
These equations may be written in the form:
(𝑎𝑥 + 𝑏𝑦 + 𝑐)𝑑𝑥 + (𝑎1 𝑥 + 𝑏1 𝑦 + 𝑐1 )𝑑𝑦 = 0
Or
𝑑𝑦 𝑎𝑥 + 𝑏𝑦 + 𝑐
=
𝑑𝑥 𝑎1 𝑥 + 𝑏1 𝑦 + 𝑐1
The solution of these equations' dependent on the relationship between the two lines.
There are two cases:
1) If the lines are intersected, then,
𝑎 𝑏
≠ ⇾ ⁡⁡⁡⁡⁡⁡𝐻𝑜𝑚𝑜𝑔𝑒𝑛𝑒𝑜𝑢𝑠
𝑎1 𝑏1
2) If the lines are parallel, then,
𝑎 𝑏
= ⇾ ⁡⁡⁡⁡⁡⁡𝑆𝑒𝑝𝑎𝑟𝑎𝑏𝑙𝑒
𝑎1 𝑏1
Example: Solve the D.E.
𝑑𝑦 2𝑥 − 3𝑦 + 4
=−
𝑑𝑥 3𝑥 − 2𝑦 + 1
𝑎 𝑏
Here, the two lines are intersected since ≠ and the point of intersection is (1,2). We
𝑎1 𝑏1
must translate the origin to the point of intersection using the substitution:
𝑥 = 𝑋 + 1,⁡⁡⁡⁡𝑦 = 𝑌 + 2
So that, 𝑑𝑥 = 𝑑𝑋,⁡⁡⁡⁡⁡⁡⁡𝑑𝑦 = 𝑑𝑌
𝑑𝑌 2𝑋 − 3𝑌
=− ⁡⁡⁡⁡⁡⁡⁡⁡⁡⁡⁡⁡⁡⁡⁡⁡⁡⁡⁡⁡⁡⁡⁡⁡⁡⁡⁡⁡⁡⁡⁡⁡(𝑖)
𝑑𝑋 3𝑋 − 2𝑌
which is homogeneous
𝑑𝑌 𝑑𝑣
put 𝑌 = 𝑉𝑥,⁡⁡⁡ =𝑣+𝑋
𝑑𝑋 𝑑𝑋

substituting in Eq. (𝑖), we have:


𝑑𝑣 2 − 3𝑣
𝑣+𝑋 =−
𝑑𝑋 3 − 2𝑣

35
𝑑𝑣 2 − 3𝑣 2(𝑣 2 − 1)
𝑋 =− −𝑣 =
𝑑𝑋 3 − 2𝑣 3 − 2𝑣
By separating the variables of the above equation, we get:
𝑑𝑋 3 − 2𝑣
= 𝑑𝑣
𝑋 2(𝑣 2 − 1)
𝑑𝑋 3 − 2𝑣
2 = 𝑑𝑣
𝑋 (𝑣 − 1)(𝑣 + 1)
Using partial fraction tends to:
𝑑𝑋 1 1
2 = 0.5 ( ) − 2.5 ( ) 𝑑𝑣
𝑋 (𝑣 − 1) (𝑣 + 1)
Integrating both sides we get:
4⁡𝑙𝑛(𝑋) = 𝑙𝑛(𝑣 − 1) − 5𝑙𝑛(𝑣 + 1) = 𝑙𝑛⁡(𝐶)
𝑣−1
𝑋4 = 𝐶
(𝑣 + 1)5
𝑌
Put 𝑣 = ⁡⁡⁡⁡𝑎𝑛𝑑⁡𝑋 = 𝑥 + 1,⁡⁡⁡𝑌 = 𝑦 − 2, we have
𝑋

𝑦−2
(𝑥 − 1 − 1)
(𝑥 − 1)4 = 𝐶 5
𝑦−2
(𝑥 − 1 + 1)

Example: Solve the D.E.


𝑑𝑦 3𝑥 − 4𝑦 − 2
=−
𝑑𝑥 6𝑥 − 8𝑦 − 5
𝑎 𝑏
Here, the two lines are parallel since, = , then the equation change to separate of
𝑎1 𝑏1
variables using the substitution:
3𝑥 − 4𝑦 = 𝑧
𝑑𝑦 𝑑𝑧
So that, 3−4 =
𝑑𝑥 𝑑𝑥

substituting in original equation, we have:


𝑑𝑣 2 − 3𝑣
𝑣+𝑋 =−
𝑑𝑋 3 − 2𝑣

36
𝑑𝑧 4𝑧 − 8 2𝑧 − 7
=3− =
𝑑𝑥 2𝑧 − 5 2𝑧 − 5
By separating the variables of the above equation, we get:
2𝑧 − 5 2
𝑑𝑥 = 𝑑𝑧 = (1 + ) 𝑑𝑧
2𝑧 − 7 2𝑧 − 7
Integrating both sides we get:
𝑥 = 𝑧 + 𝑙𝑛(2𝑧 − 7) + 𝐶
Put 𝑧 = 3𝑥 − 4𝑦, we have the general solution;
𝑥 = (3𝑥 − 4𝑦) + ln(6𝑥 − 8𝑦 − 7) + 𝐶

5- Linear differential equation


As mentioned before that, the first order D.E. is an equation that is linear in the
dependent variable y and its derivatives, then this type takes the form:
𝑑𝑦
+ 𝑃(𝑥)𝑦 = 𝑄(𝑥)⁡⁡⁡⁡⁡⁡⁡⁡⁡⁡⁡⁡(1)
𝑑𝑥
Where 𝑃(𝑥) and 𝑄(𝑥) are given functions in⁡𝑥 only (or constants).
𝑑𝑦
Note: the coefficients of must equal positive one
𝑑𝑥

To solve equation (1), we can write it in the following form:


[𝑃(𝑥)𝑦 − 𝑄(𝑥)]𝑑𝑥 + 𝑑𝑦 = 0⁡⁡⁡⁡⁡⁡⁡⁡⁡⁡⁡⁡(2)
Multiplying equation (2) by an integrating factor 𝝁 = 𝝁(𝒙) which convert it to an exact
differential equation as follows:
𝝁[𝑃(𝑥)𝑦 − 𝑄(𝑥)]𝑑𝑥 + 𝝁𝑑𝑦 = 0⁡⁡⁡⁡⁡⁡⁡⁡⁡⁡⁡⁡
where 𝑀 = ⁡𝝁[𝑃(𝑥)𝑦 − 𝑄(𝑥)] and 𝑁 = 𝝁
𝜕𝑀 𝜕𝑁 𝜕𝜇
= 𝝁[𝑃(𝑥)] = = 𝜇′ =
𝜕𝑦 𝜕𝑥 𝜕𝑥
Then, 𝜇 = 𝜇𝑃(𝑥), separate the variables we get:
𝑑𝜇
= 𝑃(𝑥)𝑑𝑥
𝜇
Integrate both sides we get:
37
ln 𝜇 = ∫ 𝑃(𝑥)⁡𝑑𝑥

𝜇 = 𝑒 ∫ 𝑃(𝑥)⁡𝑑𝑥 ⁡⁡⁡⁡⁡⁡⁡⁡⁡(3)
Multiplying Eq.(1)by Eq. (3) to obtain:
𝑑𝑦 ∫ 𝑃(𝑥)⁡𝑑𝑥
𝑒 + 𝑃(𝑥)⁡𝑦⁡𝑒 ∫ 𝑃(𝑥)⁡𝑑𝑥 = 𝑄(𝑥)⁡𝑒 ∫ 𝑃(𝑥)⁡𝑑𝑥 ⁡⁡⁡⁡⁡⁡⁡⁡⁡⁡⁡(4)
𝑑𝑥
Equation (4) can be written in the form:
𝑑
𝑦⁡𝑒 ∫ 𝑃(𝑥)⁡𝑑𝑥 = 𝑄(𝑥)⁡𝑒 ∫ 𝑃(𝑥)⁡𝑑𝑥 ⁡⁡⁡⁡⁡⁡⁡⁡⁡⁡⁡⁡⁡⁡⁡⁡⁡⁡⁡(5)
𝑑𝑥
Integrating Eq.(5) w.r.t. x to get:

𝑦⁡𝑒 ∫ 𝑃(𝑥)⁡𝑑𝑥 = ∫ 𝑄(𝑥)⁡𝑒 ∫ 𝑃(𝑥)⁡𝑑𝑥 + 𝐶

𝜇⁡𝑦 = ∫ 𝜇⁡𝑄(𝑥)⁡⁡𝑑𝑥 + 𝐶⁡⁡⁡⁡⁡⁡⁡⁡⁡⁡⁡⁡⁡⁡⁡⁡⁡⁡⁡⁡⁡(6)

Equation (6) is the general solution where 𝜇 is given by Eq. (3) and C is an arbitrary
constant.
Note: the first order differential equation,
𝑑𝑥
+ 𝑃(𝑦)𝑥 = 𝑄(𝑦)
𝑑𝑦
Is linear in x (dependent variable) and its derivatives, and its solution is:

𝜇 = 𝑒 ∫ 𝑃(𝑦)⁡𝑑𝑦

𝜇⁡𝑥 = ∫ 𝜇⁡𝑄(𝑦)⁡⁡𝑑𝑦 + 𝐶

Example: Solve the D.E.


𝑑𝑦 2
− 𝑦 = (𝑥 + 1)3
𝑑𝑥 𝑥 + 1
The given equation is linear with
2
𝑃(𝑥) = − , ⁡⁡⁡⁡𝑄(𝑥) = (𝑥 + 1)3
𝑥+1
2 1
𝜇 = 𝑒 ∫ −𝑥+1⁡𝑑𝑥 = 𝑒 −2ln⁡(𝑥+1) =
(𝑥 + 1)2
38
𝜇⁡𝑦 = ∫ 𝜇⁡𝑄(𝑥)⁡⁡𝑑𝑥 + 𝐶

𝑦 (𝑥 + 1)3 𝑥2
=∫ ⁡⁡𝑑𝑥 + 𝐶 = +𝑥+𝐶
(𝑥 + 1)2 (𝑥 + 1)2 2
6- Bernoulli's differential equation
The Bernoulli's differential equation takes the form:
𝑑𝑦
+ 𝑃(𝑥)𝑦 = 𝑄(𝑥)𝑦 𝑛 ⁡⁡⁡⁡⁡⁡⁡⁡⁡⁡⁡⁡(1)
𝑑𝑥
where 𝑃(𝑥) and 𝑄(𝑥) are continuous functions in x or constants and (𝑛 ≠ 0, 𝑛 ≠ 1). This
equation reduces to a linear one by using the following steps:
Dividing both sides of equation (1) by 𝑦 𝑛 we get:
𝑑𝑦
𝑦 −𝑛 + 𝑃(𝑥)𝑦 −𝑛+1 = 𝑄(𝑥)⁡⁡⁡⁡⁡⁡⁡⁡⁡⁡(2)
𝑑𝑥
Making the substitution
𝑣 = 𝑦 −𝑛+1
Differentiate it w.r.t. x we get:
𝑑𝑣 𝑑𝑦
= (−𝑛 + 1)𝑦 −𝑛
𝑑𝑥 𝑑𝑥

𝑑𝑦 1 𝑑𝑣
𝑦 −𝑛 =
𝑑𝑥 (−𝑛 + 1) 𝑑𝑥
Substitution into Eq. (2) we have:
1 𝑑𝑣
+ 𝑃(𝑥)𝑣 = 𝑄(𝑥)
(−𝑛 + 1) 𝑑𝑥
𝑑𝑣
+ (−𝑛 + 1)⁡𝑃(𝑥)𝑣 = (−𝑛 + 1)⁡𝑄(𝑥)
𝑑𝑥
Which is a linear differential equation and 𝑣 is the dependent variable.

Example: Solve the D.E.


𝑑𝑦
𝑥 + 𝑦 = 𝑦 2 ln(𝑥)
𝑑𝑥

39
This is Bernoulli's equation due to the existence of 𝑦 2 in the right side of the differential
𝑑𝑦
equation and the coefficient of must be equal one, then dividing both sides by (𝑥𝑦 2 ),
𝑑𝑥
we have:
−2
𝑑𝑦 𝑦 −1 ln(𝑥)
𝑦 + =
𝑑𝑥 𝑥 𝑥
put 𝑦 −1 = 𝑣
𝑑𝑣 𝑣 ln(𝑥)
− + =
𝑑𝑥 𝑥 𝑥

𝑑𝑣 𝑣 ln(𝑥)
− =−
𝑑𝑥 𝑥 𝑥
Which is linear differential equation with:
1 ln(𝑥)
𝑃(𝑥) = ⁡ − ⁡⁡⁡⁡⁡𝑎𝑛𝑑⁡⁡𝑄(𝑥) = −
𝑥 𝑥
1 1
𝜇 = 𝑒 ∫ −𝑥⁡𝑑𝑥 = 𝑒 −ln⁡(𝑥) =
𝑥
The general solution is
1 ln(𝑥) ln(𝑥) 1
𝑣 = ∫ − 2 ⁡𝑑𝑥 = − [− − ]+𝐶
𝑥 𝑥 𝑥 𝑥
Put 𝑣 = 1/𝑦

Thus, the general solution is


1 ln(𝑥) 1
=− + +𝐶
𝑥𝑦 𝑥 𝑥

Inexact equations and its conditions and solutions


The inexact differential equation does not satisfy the necessary and sufficient condition
𝜕𝑀 𝜕𝑁
= . This equation can be converted to exact equation by multiplying by a factor
𝜕𝑦 𝜕𝑥
𝜇(𝑥, 𝑦) which is called an integrating factor. As an example, the equation 𝑦𝑑𝑥– 𝑥𝑑𝑦⁡ = 0
1
is inexact, this equation can convert into exact by multiplying it by the factor or by the
𝑥2
1
factor , this means that is not unique.
𝑦2

40
Methods of determine the integrating factor
Since there is no available general method for determine the integrating factor 𝜇(𝑥, 𝑦)
for the inexact equation
𝑀(𝑥, 𝑦)𝑑𝑥 + 𝑁(𝑥, 𝑦)𝑑𝑦 = 0⁡⁡⁡⁡⁡⁡⁡⁡⁡⁡⁡⁡⁡⁡⁡⁡⁡⁡⁡⁡⁡⁡⁡⁡⁡⁡⁡⁡⁡⁡⁡⁡⁡⁡⁡⁡⁡⁡(1)
there are some helpful particular cases to deduce this factor, considering the equation
𝜇𝑀(𝑥, 𝑦)𝑑𝑥 + ⁡𝜇𝑁(𝑥, 𝑦)𝑑𝑦 = 0 is exact, this yield
𝜕𝜇𝑀 𝜕𝜇𝑁
=
𝜕𝑦 𝜕𝑥
𝜕𝑀 𝜕𝜇 𝜕𝑁 𝜕𝜇
𝜇 +𝑀 =𝜇 +𝑁
𝜕𝑦 𝜕𝑦 𝜕𝑥 𝜕𝑥
𝜕𝑀 𝜕𝑁 1 𝜕𝜇 𝜕𝜇
− = (𝑁 − 𝑀 )⁡⁡⁡⁡⁡⁡⁡⁡⁡⁡(2)
𝜕𝑦 𝜕𝑥 𝜇 𝜕𝑥 𝜕𝑦
The integrating factor 𝜇 can be determined in five cases:

Case 1
Assuming equation (1) has 𝜇 = 𝜇(𝑥), then
𝜕𝜇 𝑑𝜇 𝜕𝜇 𝑑𝜇
= ,⁡⁡⁡⁡⁡⁡⁡⁡⁡⁡⁡⁡⁡⁡⁡⁡⁡⁡⁡⁡⁡ = =0
𝜕𝑥 𝑑𝑥 𝜕𝑦 𝑑𝑦
then equation (2) becomes
𝜕𝑀 𝜕𝑁 1 𝜕𝜇
− = (𝑁 )
𝜕𝑦 𝜕𝑥 𝜇 𝜕𝑥
1 𝜕𝑀 𝜕𝑁 1 𝑑𝜇
( − ) = ( ) = 𝐹(𝑥)
𝑁 𝜕𝑦 𝜕𝑥 𝜇 𝑑𝑥
both sides of this equation depend on 𝑥 only, then
𝑑𝜇
( ) = 𝐹(𝑥)⁡𝑑𝑥
𝜇

ln(𝜇) = ∫ 𝐹(𝑥)⁡𝑑𝑥

𝜇 = 𝑒 ∫ 𝐹(𝑥)⁡𝑑𝑥

41
Example
Prove that the equation (𝑒 𝑥 − 𝑠𝑖𝑛𝑦)𝑑𝑥⁡ + ⁡𝑐𝑜𝑠𝑦⁡𝑑𝑦 = 0 is inexact, then determine its
integrating factor and check your result.
𝜕𝑀
𝑀(𝑥, 𝑦) = 𝑒 𝑥 − 𝑠𝑖𝑛𝑦,⁡⁡⁡ ⇾ ⁡⁡ = −𝑐𝑜𝑠𝑦
𝜕𝑦
𝜕𝑁
𝑁(𝑥, 𝑦) = ⁡𝑐𝑜𝑠𝑦,⁡⁡⁡ =0
𝜕𝑥
𝜕𝑀 𝜕𝑁

𝜕𝑦 𝜕𝑥
i.e., the given equation is not exact. Assuming 𝜇 = 𝜇(𝑥), then
1 𝜕𝑀 𝜕𝑁 −𝑐𝑜𝑠𝑦
𝐹(𝑥) = ( − )= = −1
𝑁 𝜕𝑦 𝜕𝑥 𝑐𝑜𝑠𝑦

𝜇 = 𝑒 ∫ 𝐹(𝑥)⁡𝑑𝑥 = 𝑒 ∫ −1⁡𝑑𝑥 = 𝑒 −𝑥
Verify your result.

Case 2
Assuming equation (1) has 𝜇 = 𝜇(𝑦), then
𝜕𝜇 𝑑𝜇 𝜕𝜇 𝑑𝜇
= ,⁡⁡⁡⁡⁡⁡⁡⁡⁡⁡⁡⁡⁡⁡⁡⁡⁡⁡⁡⁡⁡ = =0
𝜕𝑦 𝑑𝑦 𝜕𝑥 𝑑𝑥
Then equation (2) becomes
𝜕𝑀 𝜕𝑁 1 𝜕𝜇
− = (𝑀 )
𝜕𝑦 𝜕𝑥 𝜇 𝜕𝑦
−1 𝜕𝑀 𝜕𝑁 1 𝑑𝜇
( − ) = ( ) = 𝐺(𝑦)
𝑀 𝜕𝑦 𝜕𝑥 𝜇 𝑑𝑦
both sides of this equation depend on 𝑦 only, then
𝑑𝜇
( ) = 𝐺(𝑦)⁡𝑑𝑦
𝜇

ln(𝜇) = ∫ 𝐺(𝑦)⁡𝑑𝑦

𝜇 = 𝑒 ∫ 𝐺(𝑦)⁡𝑑𝑦
42
Example
Prove that the equation (𝑥𝑦)𝑑𝑥⁡ + ⁡ (1 + 𝑥 2 )𝑑𝑦 = 0 is inexact, then determine its
integrating factor and check your result.
𝜕𝑀
𝑀(𝑥, 𝑦) = ⁡𝑥𝑦, =𝑥
𝜕𝑦
𝜕𝑁
𝑁(𝑥, 𝑦) = ⁡1 + 𝑥 2 , = 2𝑥
𝜕𝑥
𝜕𝑀 𝜕𝑁

𝜕𝑦 𝜕𝑥
i.e., the given equation is inexact. Assuming 𝜇 = 𝜇(𝑥), then
−1 𝜕𝑀 𝜕𝑁 1
𝐺(𝑦) = ( − )=
𝑀 𝜕𝑦 𝜕𝑥 𝑦
1
∫𝑦⁡𝑑𝑥
𝜇= 𝑒 ∫ 𝐺(𝑦)⁡𝑑𝑦 = 𝑒 =𝑦
Using the integrating factor 𝜇 the equation can be converted into exact given by
(𝑥𝑦 2 )𝑑𝑥 + (1 + 𝑥 2 )𝑦𝑑𝑦 = 0 which can be solved as follows
𝑢 = ∫(𝑥𝑦 2 )⁡𝑑𝑥 + 𝜙(𝑦) = 0.5𝑥 2 𝑦 2 + 𝜙(𝑦)⁡⁡⁡⁡⁡⁡⁡⁡⁡⁡⁡⁡⁡⁡⁡⁡(𝑖)

2
𝑦2 2 2
𝑢 = ∫(1 + 𝑥 )𝑦⁡𝑑𝑥 + 𝜓(𝑥) = 0.5𝑥 𝑦 + + 𝜓(𝑥)⁡⁡⁡⁡⁡⁡⁡⁡⁡⁡⁡⁡⁡⁡⁡⁡(𝑖𝑖)
2
From (i) and (ii), the complete solution is written
(1 + 𝑥 2 )𝑦 2 = 𝐶
The deduced solution verifies the exact equation only.

Case 3
Assuming equation (1) has 𝜇 = 𝜇(𝑢),⁡⁡⁡𝑢 = 𝑓(𝑥, 𝑦) then
𝜕𝜇 𝜕𝜇
Letting 𝜇 = 𝑥𝑦, = 𝑦, = 𝑥 , equation (2) becomes
𝜕𝑥 𝜕𝑦

1 𝜕𝑀 𝜕𝑁
(𝑦𝑁 − 𝑥𝑀) = −
𝜇 𝜕𝑦 𝜕𝑥

43
𝜕𝑀 𝜕𝑁
1 𝜕𝑦 − 𝜕𝑥
= = 𝐾(𝑢)
𝜇 𝑦𝑁 − 𝑥𝑀
1
𝑑𝜇 = 𝐾(𝑢)⁡𝑑𝑢
𝜇

ln(𝜇) = ∫ 𝐾(𝑢)⁡𝑑𝑢

𝜇 = 𝑒 ∫ 𝐾(𝑢)⁡𝑑𝑢
Example
Prove that the equation (𝑦 3 + 𝑥𝑦 2 + 𝑦)⁡𝑑𝑥⁡ + ⁡ (𝑥 3 + 𝑥 2 𝑦 + 𝑥)⁡𝑑𝑦 = 0 is inexact, then
find its integrating factor and check your result.
𝜕𝑀
𝑀(𝑥, 𝑦) = 𝑦 3 + 𝑥𝑦 2 + 𝑦, = 3𝑦 2 + 2𝑥𝑦 + 1
𝜕𝑦
𝜕𝑁
𝑁(𝑥, 𝑦) = 𝑥 3 + 𝑥 2 𝑦 + 𝑥, = 3𝑥 2 + 2𝑥𝑦 + 1
𝜕𝑥
𝜕𝑀 𝜕𝑁

𝜕𝑦 𝜕𝑥
i.e., the given equation is inexact. Assuming 𝑢 = 𝑥𝑦, then
3(𝑦 2 − 𝑥 2 ) −3 −3
𝐾(𝑢) = = =
𝑥𝑦(𝑥 2 − 𝑦 2 ) 𝑥𝑦 𝑢
−3 1 1
𝜇 = 𝑒 ∫ 𝐾(𝑢)⁡𝑑𝑢 = 𝑒 ∫ 𝑢 ⁡𝑑𝑢 = =
𝑢3 (𝑥𝑦)3
Verify your result.

Case 4
Assuming equation (1) has 𝜇 = 𝜇(𝑢),⁡⁡⁡𝑢 = 𝑓(𝑥, 𝑦),
Letting 𝜇 = 𝑥/𝑦 then
𝜕𝜇 1 𝜕𝜇 −𝑥
= ,⁡⁡⁡⁡⁡⁡⁡⁡⁡⁡ = 2
𝜕𝑥 𝑦 𝜕𝑦 𝑦
and equation (2) becomes

44
1 𝜕𝜇 𝜕𝜇 𝜕𝑀 𝜕𝑁
(𝑁 −𝑀 )= −
𝜇 𝜕𝑥 𝜕𝑦 𝜕𝑦 𝜕𝑥
𝜕𝑀 𝜕𝑁 𝜕𝑀 𝜕𝑁
1 − 𝑦2 ( − )
𝜕𝑦 𝜕𝑥 𝜕𝑦 𝜕𝑥
= =
𝜇 𝑁 𝜕𝜇 − 𝑀 𝜕𝜇 𝑥𝑀 − 𝑦𝑁
𝜕𝑥 𝜕𝑦
1
𝑑𝜇 = 𝑃(𝑢)⁡𝑑𝑢
𝜇

ln(𝜇) = ∫ 𝑃(𝑢)⁡𝑑𝑢

𝜇 = 𝑒 ∫ 𝑃(𝑢)⁡𝑑𝑢
Example
Prove that the equation 3𝑦𝑑𝑥 − 𝑥𝑑𝑦 = 0 is inexact, then determine its integrating factor
and check your result.
𝜕𝑀 𝜕𝑁
𝑀 = 3𝑦⁡⁡ = 3, 𝑁 = −𝑥⁡⁡ = −1
𝜕𝑦 𝜕𝑥
𝜕𝑀 𝜕𝑁
i.e., ≠ and the equation is inexact.
𝜕𝑦 𝜕𝑥

𝜕𝑀 𝜕𝑁 𝜕𝑀 𝜕𝑁
1 − 𝑦2 ( − ) 𝑦 2 2
𝜕𝑦 𝜕𝑥 𝜕𝑦 𝜕𝑥
𝑃(𝑢) = = = =2 = =
𝜇 𝑁 𝜕𝜇 − 𝑀 𝜕𝜇 𝑥𝑀 − 𝑦𝑁 𝑥 𝑥/𝑦 𝑢
𝜕𝑥 𝜕𝑦
2 𝑥2
𝜇= 𝑒 ∫ 𝑃(𝑢)⁡𝑑𝑢 = 𝑒 ∫𝑢⁡𝑑𝑢 2
=𝑢 = 2
𝑦
Verify your result.

Case 5
𝑦
Assuming equation (1) has 𝜇 = 𝜇(𝑢), 𝜇 = then
𝑥

𝜕𝜇 −𝑦 𝜕𝜇 1
= 2 ,⁡⁡⁡⁡⁡⁡⁡⁡⁡⁡ =
𝜕𝑥 𝑥 𝜕𝑦 𝑥
and equation (2) becomes

45
1 𝜕𝜇 𝜕𝜇 𝜕𝑀 𝜕𝑁
(𝑁 −𝑀 )= −
𝜇 𝜕𝑥 𝜕𝑦 𝜕𝑦 𝜕𝑥

2 𝜕𝑀 𝜕𝑁
1 −𝑥 ( 𝜕𝑦 − 𝜕𝑥 )
= = 𝑄(𝑢)
𝜇 𝑥𝑀 + 𝑦𝑁
1
𝑑𝜇 = 𝑄(𝑢)⁡𝑑𝑢
𝜇

ln(𝜇) = ∫ 𝑄(𝑢)⁡𝑑𝑢

𝜇 = 𝑒 ∫ 𝑄(𝑢)⁡𝑑𝑢
Example
Prove that the equation 𝑦𝑑𝑥 − 3𝑥𝑑𝑦 = 0 is inexact, then determine its integrating factor
and check your result.
Solution
𝜕𝑀 𝜕𝑁
𝑀 = 𝑦⁡⁡ = 1, 𝑁 = −3𝑥⁡⁡ = −3
𝜕𝑦 𝜕𝑥
𝜕𝑀 𝜕𝑁
i.e., ≠ and the equation is inexact.
𝜕𝑦 𝜕𝑥

𝜕𝑀 𝜕𝑁
−𝑥 2 ( − ) −4𝑥 2 2 2
𝜕𝑦 𝜕𝑥
𝑄(𝑢) = = = =
𝑥𝑀 + 𝑦𝑁 −2𝑥𝑦 𝑦/𝑥 𝑢
2
𝑦2
𝜇 = 𝑒 ∫ 𝑄(𝑢)⁡𝑑𝑢 = 𝑒 ∫𝑢⁡𝑑𝑢 = 𝑢2 =
𝑥2

Verify your result.

Second Order First Degree Differential Equations


We shall discuss the cases, which can be convert into the first order, and divide them into
the following types:
1) Equations not contain y explicitly
2) Equations not contain x explicitly
3) Equations not contain x and y explicitly
We shall discuss the above cases in details.
46
1) Equations not contain y explicitly 𝒇(𝒙, 𝒚′, 𝒚′′)
For these equations we use the following substitution to convert them to first order first
degree D.Eqs:
𝑑𝑦 𝑑2𝑦 𝑑𝑧
= 𝑧 differentiate w.r.t. 𝑥 = ⁡⁡⁡⁡⁡⁡⁡⁡⁡⁡⁡⁡⁡⁡⁡⁡⁡⁡⁡(𝑖)
𝑑𝑥 𝑑𝑥 2 𝑑𝑥

Example: solve the following differential equation;


𝑑 2 𝑦 𝑑𝑦
𝑥 2+ = 4𝑥
𝑑𝑥 𝑑𝑥
This equation is not containing 𝑦 explicitly
Using the substitution (𝑖), we get
𝑑𝑧
𝑥 + 𝑧 = 4𝑥
𝑑𝑥
Which is a first order first degree D.E. with z dependent variable and its solution is:
𝑑𝑧 1
+ 𝑧=4
𝑑𝑥 𝑥
1
The last equation is a linear D.E. with 𝑃(𝑥) = ⁡ and 𝑄(𝑥) = 4, then
𝑥
1
𝜇 = 𝑒 ∫ 𝑃(𝑥)⁡𝑑𝑥 = 𝑒 ∫𝑥⁡𝑑𝑥 = 𝑥
And the general solution is:

𝑥𝑧 = ∫ 4𝑥⁡𝑑𝑥 + 𝐶 = 2𝑥 2 + 𝐶

𝑑𝑦
Where C is a constant, put 𝑧 = , then;
𝑑𝑥

𝑑𝑦
𝑥 = 2𝑥 2 + 𝐶
𝑑𝑥
Which is a first order first degree D.E. with y dependent variable, separate variable; we get
the general solution;
𝑑𝑦 𝐶
= 2𝑥 +
𝑑𝑥 𝑥
𝐶
𝑑𝑦 = (2𝑥 + ) ⁡𝑑𝑥
𝑥
Integrating this equation, we get:
𝑦 = 𝑥 2 + 𝐶 ln(𝑥) + 𝐶1
47
Where 𝐶1 is a constant
2) Equations not contain x explicitly 𝒇(𝒚, 𝒚′, 𝒚′′)
For these equations we use the following substitution to convert them to first order first
degree D.Eqs:
𝑑𝑦
=𝑧
𝑑𝑥
differentiate 𝑤. 𝑟. 𝑡. 𝑥
𝑑 2 𝑦 𝑑𝑧 𝑑𝑦 𝑑𝑧 𝑑𝑧
= = = 𝑧 ⁡⁡⁡⁡⁡⁡⁡⁡⁡⁡⁡⁡⁡⁡⁡⁡⁡⁡⁡(𝑖𝑖)
𝑑𝑥 2 𝑑𝑥 𝑑𝑥 𝑑𝑦 𝑑𝑦
Example: Solve the D.E.:
𝑦(𝑦 − 1)𝑦′′ + (𝑦′)2 = 0
Since the equation not contain 𝑥 explicitly, using the substitution (𝑖𝑖), then
𝑑𝑧
𝑦(𝑦 − 1)𝑧 + (𝑧)2 = 0
𝑑𝑦
Which is a first order first degree D.E., separate the variables we have:
𝑑𝑧 𝑑𝑦 1 1
=− = −[ − ] 𝑑𝑦
𝑑𝑧 𝑦(𝑦 − 1) 𝑦−1 𝑦
Integrate both sides we get:
𝑙𝑛(𝑧) = −𝑙𝑛(𝑦 − 1) + 𝑙𝑛(𝑦) + 𝑙𝑛(𝐶)
𝐶𝑦
ln(𝑧) = ln⁡ ( )
𝑦−1
𝐶𝑦
𝑧=
𝑦−1
where C is a constant.
𝑑𝑦
put 𝑧 = , then;
𝑑𝑥

𝑑𝑦 𝐶𝑦
=
𝑑𝑥 𝑦 − 1
Which is a first order first degree D.E. with y dependent variable, separate variable; we get
the general solution;
𝑦−1
𝑑𝑦 = 𝐶⁡𝑑𝑥
𝑦
1
(1 − ) 𝑑𝑦 = 𝐶⁡𝑑𝑥
𝑦
48
Integrating this equation, we get:
𝑦 − ln(𝑦) = 𝐶𝑥 + 𝐶1
Where 𝐶1 is a constant
3) Equations not contain x and y explicitly 𝒇(𝒚′, 𝒚′′)
For these equations we can use one of the forementioned substitution (𝑖), (𝑖𝑖) to
convert them to first order first degree D.Eqs:
𝑑𝑦 𝑑2𝑦 𝑑𝑧
=𝑧 differentiate 𝑤. 𝑟. 𝑡. 𝑥 = ⁡⁡⁡⁡⁡⁡⁡⁡⁡⁡⁡⁡⁡⁡⁡⁡⁡⁡⁡⁡⁡⁡⁡⁡⁡⁡⁡⁡⁡⁡⁡⁡⁡⁡⁡⁡(𝑖)
𝑑𝑥 𝑑𝑥 2 𝑑𝑥

𝑑𝑦 𝑑2𝑦 𝑑𝑧 𝑑𝑦 𝑑𝑧 𝑑𝑧
=𝑧 differentiate 𝑤. 𝑟. 𝑡. 𝑥⁡⁡⁡⁡⁡⁡⁡⁡⁡⁡⁡ = = =𝑧 ⁡⁡⁡⁡⁡⁡⁡⁡⁡⁡⁡⁡⁡⁡⁡⁡⁡⁡⁡(𝑖𝑖)
𝑑𝑥 𝑑𝑥 2 𝑑𝑥 𝑑𝑥 𝑑𝑦 𝑑𝑦

Example: Solve the D.E.:


3𝑦 ′′ = 5𝑦′
Since the equation not contain 𝑥 and y explicitly, using the substitution (𝑖)𝑜𝑟⁡(𝑖𝑖), then
𝑑𝑦 𝑑 2 𝑦 𝑑𝑧
= 𝑧,⁡⁡⁡⁡⁡⁡ 2 =
𝑑𝑥 𝑑𝑥 𝑑𝑥
𝑑𝑧
3 = 5𝑧
𝑑𝑥
𝑑𝑧 5
= ⁡𝑑𝑥
𝑧 3
5
ln⁡(𝑧) = 𝑥 + 𝐶
3
5
𝑧 = 𝑒 3𝑥+𝐶
𝑑𝑦
put 𝑧 = , then;
𝑑𝑥

𝑑𝑦 5
= 𝑒 3𝑥+𝐶
𝑑𝑥
which is a first order first degree D.E. with y dependent variable, separate variable; we get
the general solution;
5
𝑑𝑦 = 𝑒 3𝑥+𝐶 ⁡𝑑𝑥
Integrating this equation, we get:

49
3 5
𝑦 = 𝑒 3𝑥+𝐶 + 𝐶1
5
where 𝐶1 is a constant

Exercise
Solve the following first order first degree differential equations
i) (𝑥 2 + 𝑦 2 )⁡𝑑𝑥 + 𝑥𝑦⁡𝑑𝑦 = 0
ii) 𝑡𝑎𝑛(𝑥)𝑠𝑖𝑛2 (𝑦)𝑑𝑥 + ⁡𝑐𝑜𝑡(𝑦)𝑐𝑜𝑠 2 (𝑥)⁡𝑑𝑦 = 0
iii) (𝑥 2 − 𝑥)⁡𝑑𝑦 = (𝑦 2 + 𝑦)⁡𝑑𝑥
𝑑𝑦
iv) 𝑥 − 𝑦 = √𝑥 2 − 𝑦 2
𝑑𝑥
𝑑𝑦 𝑦 𝑦2
v) 2 = +
𝑑𝑥 𝑥 𝑥2
𝑑𝑦 5𝑥+4𝑦−3
vi) =
𝑑𝑥 10𝑦−𝑥−12
𝑑𝑦 2
vii) + 𝑦 = 𝑥 2 (𝑥 + 1)
𝑑𝑥 𝑥
viii) 𝑥(𝑥 − 2𝑦)𝑑𝑦 = 𝑦(𝑦 − 2𝑥)𝑑𝑥
ix) (𝑥 + 𝑦)𝑑𝑦 = −𝑦⁡𝑑𝑥
x) (1 + 𝑥 2 )⁡𝑑𝑦 − (1 − 𝑦 2 )⁡𝑑𝑥 = 0
xi) (1 − 𝑦)𝑦′ + 𝑦(1 + 𝑥) = 0
xii) 𝑥𝑦 ′ − 𝑦 = 𝑥𝑦
𝑑𝑦 2𝑦−𝑥
xiii) =
𝑑𝑥 2𝑥−𝑦
2 )⁡𝑑𝑦
xiv) 𝑦(1 + 𝑥 + 𝑥 2 (1 − 𝑦 2 )⁡𝑑𝑥 = 0
xv) (𝑥 2 + 𝑦 2 )⁡𝑑𝑥 − 2𝑥𝑦⁡𝑑𝑦 = 0
𝑑𝑦 𝑥+3𝑦−5
xvi) =
𝑑𝑥 2𝑥−𝑦−3
𝑑𝑦 3𝑥−4𝑦−2
xvii) =
𝑑𝑥 6𝑥−8𝑦−5
𝑑𝑦 2𝑥𝑦
xviii) =
𝑑𝑥 𝑥 2 −𝑦 2
xix) 𝑦 + (3𝑥𝑦 − 1)𝑦 ′ = 0
2

𝑑𝑦 𝑦 si n(𝑥)
xx) + =
𝑑𝑥 𝑥 𝑥
𝑑𝑦 𝑦 2
xxi) + =𝑦
𝑑𝑥 𝑥
𝑑𝑦 2𝑥+𝑦−1
xxii) =
𝑑𝑥 4𝑥+2𝑦+5
𝑑𝑦 6𝑥+8𝑦−3
xxiii) =
𝑑𝑥 3𝑥+4𝑦+2
xxiv) (2⁡𝑐𝑜𝑠(2𝑥) + 3𝑦)⁡𝑑𝑥 + (3𝑥 + 𝑠𝑖𝑛(2𝑥) + 3𝑥𝑦)⁡𝑑𝑦 = 0
50
𝑑𝑦 𝑦
xxv) + = 1 + 𝑥2
𝑑𝑥 𝑥
xxvi) 𝑦′𝑐𝑜𝑠(𝑥) + 𝑦𝑠𝑖𝑛(𝑥) = 1
𝑑𝑦
xxvii) + 𝑦se c(𝑥) = 𝑐𝑜𝑠 2 (𝑥)
𝑑𝑥
𝑑𝑦
xxviii) + 𝑦 − 𝑥𝑦 2 = 0
𝑑𝑥
𝑑𝑦
xxix) 𝑦 + 2 = 𝑦 3 (𝑥 − 1)
𝑑𝑥
xxx) (4𝑥𝑦 − 𝑥 + 3𝑦 2 )⁡𝑑𝑥 + 𝑥(2𝑦 + 𝑥)𝑑𝑦 = 0
xxxi) 𝑥 3 𝑦 ′ = 𝑥 2 𝑦 − 𝑦 4 co s(𝑥)
𝑦
xxxii) 𝑥𝑦 ′ = 𝑦 + 𝑥⁡𝑐𝑜𝑠 2 ( )
𝑥
Additional problems
i) 𝑥𝑦 ′ = 𝑦(𝑙𝑛(𝑦) − 𝑙𝑛(𝑥))
𝑦 𝑦
𝑑𝑦 2𝑥⁡𝑠𝑖𝑛ℎ( )+3𝑦⁡𝑐𝑜𝑠ℎ( )
𝑥 𝑥
ii) = 𝑦
𝑑𝑥 3𝑥⁡𝑐𝑜𝑠ℎ(𝑥)
𝑦
iii) 𝑦 ′ =
2𝑦𝑙𝑛(𝑦)+𝑦−𝑥
2
iv) (2𝑥𝑦 + 𝑦)𝑑𝑥 + (𝑥 + 2𝑦𝑥 2 − 𝑥 4 𝑦 3 )𝑑𝑦 = 0
v) 𝑦 2 ⁡𝑑𝑥 + (𝑥 2 − 𝑥𝑦 − 𝑦 2 )⁡𝑑𝑦 = 0
vi) 𝑥𝑦′′ + 𝑦′ + 2𝑥 = 0

Solve the following second order first degree D.E.


i) 𝑥𝑦′′ + 𝑦′ + 2𝑥 = 0
ii) 𝑦𝑦′′ + 1 = (𝑦 ′ )2
iii) ⁡𝑦 2 𝑦 ′′ − 𝑦 ′ = 0
iv) 𝑦 ′′ 𝑐𝑜𝑠 2 𝑥 = 1
v) 𝑦 ′′ = 𝑒 𝑥 + 6𝑥⁡⁡(𝑥 = 0, 𝑦 = 0)
vi) (𝑥 2 − 1)𝑦 ′′ + 𝑥𝑦 ′ = 0
1
vii) 𝑦 ′′ = (𝑥 = 0, 𝑦 = 1, 𝑦 ′ = 0)
𝑦3
viii) 𝑥 ′′ + 𝑥 = 1′ (𝑡 = 0, 𝑥 = 0, 𝑥 ′ = 0)

51
Differential equations of first order but not first degree
The nonlinear first order differential equations may be written in the following form:
𝑓(𝑥, 𝑦, 𝑦 ′ ) = 0⁡⁡⁡⁡⁡⁡⁡⁡⁡⁡⁡⁡⁡⁡⁡⁡⁡⁡⁡⁡⁡⁡⁡⁡⁡⁡⁡⁡⁡⁡⁡⁡⁡(1)
𝑑𝑦
Equation (1) is of degree more than one in 𝑦′. using the notation, 𝑝 = , Equation (1) can
𝑑𝑥
be written as follows:
𝑓(𝑥, 𝑦, 𝑝) = 0⁡⁡⁡⁡⁡⁡⁡⁡⁡⁡⁡⁡⁡⁡⁡⁡⁡⁡⁡⁡⁡⁡⁡⁡⁡⁡⁡⁡⁡⁡⁡⁡⁡⁡⁡(2)
The method of solution equation (2) depends on the possibility of solution with respect to
𝑝, 𝑥, and 𝑦.
1- Equations solvable for 𝒑
Equations (2), called solvable for 𝑝 if it can be solved as algebraic equation with respect
to 𝒑. in this case, we can get the roots of equation (2) in the form:
𝑝𝑛 = 𝑓(𝑥, 𝑦)⁡⁡𝑤𝑖𝑡ℎ⁡𝑛 = 1,2,3, ….
Where 𝑛 is the degree of the given D.E. (2). We obtained set of equations each of them
is of first order first degree differential equation, integrating them we obtain the
general solution.

Example: Solve the D.E.

𝑑𝑦 2 𝑑𝑦
( ) − (𝑥 + 𝑦) + 𝑥𝑦 = 0
𝑑𝑥 𝑑𝑥
𝑑𝑦 2 𝑑𝑦
This equation is nonlinear due to the term ( ) , put 𝑝 =
𝑑𝑥 𝑑𝑥

(𝑝)2 − (𝑥 + 𝑦)𝑝 + 𝑥𝑦 = 0
This equation is solvable for 𝑝 then:
(𝑝 − 𝑥)(𝑝 − 𝑦) = 0
𝑝 = 𝑥⁡⁡⁡⁡⁡⁡⁡⁡⁡⁡⁡⁡⁡⁡⁡⁡⁡⁡⁡⁡⁡⁡⁡⁡⁡⁡⁡⁡⁡⁡⁡⁡⁡⁡⁡⁡⁡⁡⁡⁡⁡⁡⁡⁡⁡⁡⁡⁡⁡⁡⁡𝑝 = 𝑦
𝑑𝑦 𝑑𝑦
= 𝑥⁡⁡⁡⁡⁡⁡⁡⁡⁡⁡⁡⁡⁡⁡⁡⁡⁡⁡⁡⁡⁡⁡⁡⁡⁡⁡⁡⁡⁡⁡⁡⁡⁡⁡⁡⁡⁡⁡⁡⁡⁡⁡⁡⁡⁡⁡⁡⁡⁡⁡⁡⁡ =𝑦
𝑑𝑥 𝑑𝑥
𝑑𝑦
𝑑𝑦 = 𝑥⁡𝑑𝑥⁡⁡⁡⁡⁡⁡⁡⁡⁡⁡⁡⁡⁡⁡⁡⁡⁡⁡⁡⁡⁡⁡⁡⁡⁡⁡⁡⁡⁡⁡⁡⁡⁡⁡⁡⁡⁡⁡⁡⁡⁡⁡⁡⁡⁡⁡ = 𝑑𝑥
𝑦

52
𝑥2
𝑦= + 𝐶⁡⁡⁡⁡⁡⁡⁡⁡⁡⁡⁡⁡⁡⁡⁡⁡⁡⁡⁡⁡⁡⁡⁡⁡⁡⁡⁡⁡⁡⁡⁡⁡⁡⁡⁡⁡⁡⁡⁡⁡⁡ ln(𝑦) = 𝑥 + 𝐶
2
𝑥2
𝑦 − − 𝐶 = 0⁡⁡⁡⁡⁡⁡⁡⁡⁡⁡⁡⁡⁡⁡⁡⁡⁡⁡⁡⁡⁡⁡⁡⁡⁡⁡⁡⁡⁡⁡⁡⁡⁡⁡⁡⁡⁡⁡⁡⁡⁡𝑦 − 𝑒 𝑥+𝐶 = 0
2
Note: We use only one constant 𝐶 in the two solutions because the problem is first order.
Then, the general solution is:
𝑥2
(𝑦 − − 𝐶)(𝑦 − 𝑒 𝑥+𝐶 ) = 0
2
2- Equations solvable for 𝒚
When the equation 𝑓(𝑥, 𝑦, 𝑝) = 0 is solvable for y, it may be written in the form:
𝑦 = 𝑓(𝑥, 𝑝)
𝑑𝑦
Differentiate it 𝑤. 𝑟. 𝑡⁡𝑥 and put 𝑝 = , we will obtain the equation:
𝑑𝑥

𝑑𝑝
𝑝 = 𝑔 (𝑥, 𝑝, )
𝑑𝑥
This is a first order first degree D.E., solve it with respect to 𝑥, we have:
𝑥 = ℎ(𝑝)⁡⁡⁡⁡⁡⁡⁡⁡⁡⁡⁡⁡⁡⁡⁡⁡⁡⁡⁡⁡⁡⁡⁡⁡⁡⁡⁡⁡⁡⁡⁡⁡⁡(1)
𝑦 = 𝑓(ℎ(𝑝), 𝑝)⁡⁡⁡⁡⁡⁡⁡⁡⁡⁡⁡⁡⁡⁡⁡⁡⁡⁡⁡⁡⁡⁡⁡⁡⁡⁡(2)
Equations (1) and (2) is the general solution in parametric form.

Example: Solve the D.E.

𝑑𝑦 2 𝑑𝑦
( ) = 2𝑥 −𝑦
𝑑𝑥 𝑑𝑥
𝑑𝑦
Put 𝑝 = , the D.E. is solvable for 𝑦, then:
𝑑𝑥

(𝑝)2 = 2𝑥𝑝 − 𝑦
𝑦 = 2𝑥𝑝 − 𝑝2 ⁡⁡⁡⁡⁡⁡⁡⁡⁡⁡⁡⁡(𝑖)
𝑑𝑦
Differentiate 𝑤. 𝑟. 𝑡. 𝑥 and put 𝑝 = , we have:
𝑑𝑥

𝑑𝑝 𝑑𝑝
𝑝 = 2𝑥 + 2𝑝 − 2𝑝
𝑑𝑥 𝑑𝑥

53
𝑑𝑝
−𝑝 = (2𝑥 − 2𝑝)
𝑑𝑥
which is first order first degree D.E. in the form:
𝑑𝑥 𝑝−𝑥 𝑥
=2 =2−2
𝑑𝑝 𝑝 𝑝
𝑑𝑥 𝑥
+2 =2
𝑑𝑝 𝑝
The last equation is a linear D.E., and its solution is:
2
∫𝑝𝑑𝑝
𝜇=𝑒 = 𝑒 2l n(𝑝) = 𝑝2
2
𝑝2 𝑥 = ∫ 2𝑝2 ⁡𝑑𝑝 + 𝐶 = 𝑝3 + 𝐶
3
2 𝐶
𝑥= 𝑝+ (1)
3 𝑝2
substitute in equation (𝑖) to get y, then:
2 𝐶 1 𝐶
𝑦 = 2𝑝 ( 𝑝 +
3 𝑝2
) − 𝑝2 = 3 𝑝2 + 2 𝑝 (2)
Equations (1) and (2) is the general solution in parametric form

3- Equations solvable for 𝒙


When the equation 𝑓(𝑥, 𝑦, 𝑝) = 0 is solvable for x, it may be written in the form:
𝑥 = 𝑓(𝑦, 𝑝)
1 𝑑𝑥
Differentiate it 𝑤. 𝑟. 𝑡⁡𝑦 and put = , we will obtain the equation:
𝑝 𝑑𝑦

1 𝑑𝑝
= 𝑔 (𝑦, 𝑝, )
𝑝 𝑑𝑦
This is a first order first degree D.E., solve it with respect to 𝑥, we have:
𝑦 = ℎ(𝑝) (1)
𝑥 = 𝑓(ℎ(𝑝), 𝑝) (2)
Equations (1) and (2) is the general solution in parametric form.

54
Example: Solve the D.E.

𝑑𝑦 2 𝑑𝑦
𝑦 ( ) = 2𝑥 −𝑦
𝑑𝑥 𝑑𝑥
𝑑𝑦
Put 𝑝 = , the D.E. is solvable for 𝑦, then:
𝑑𝑥

𝑦(𝑝)2 = 2𝑥𝑝 − 𝑦
1
𝑥 = 0.5𝑦⁡ (𝑝 + ) (𝑖)
𝑝

1 𝑑𝑥
Differentiate 𝑤. 𝑟. 𝑡. 𝑦 and put = , we have:
𝑝 𝑑𝑦

2 1 𝑑𝑝 1
= 𝑦 (1 − 2 ) + +𝑝
𝑝 𝑝 𝑑𝑦 𝑝
𝑑𝑝
2𝑝 = 𝑦(𝑝2 − 1)
𝑑𝑦
Which is first order first degree D.E., separate the variables, we have:
𝑑𝑦 𝑑𝑝
=−
𝑦 𝑝
Integrate both sides, we get:
𝑙𝑛(𝑦) = −𝑙𝑛(𝑝) + 𝑙𝑛(𝐶)
𝐶
𝑦= (1)
𝑝
substitute in equation (𝑖) to get x, then:
1𝐶 1
𝑥= (𝑝 + 𝑝) (2)
2𝑝
Equations (1) and (2) is the general solution in parametric form

55
Homogeneous Linear Differential Equations with Constant Coefficients

The homogeneous linear 𝑛𝑡ℎ order D.E. with constant coefficients takes the form:
𝑑𝑛 𝑦 𝑑 𝑛−1 𝑦 𝑑 𝑛−2 𝑦
𝑎0 𝑛 + 𝑎1 𝑛−1 + 𝑎2 𝑛−2 + ⋯ + 𝑎𝑛 𝑦 = 0⁡⁡⁡⁡⁡⁡⁡⁡⁡⁡⁡⁡⁡⁡⁡⁡⁡(1)
𝑑𝑥 𝑑𝑥 𝑑𝑥
where 𝑎0 , 𝑎1 , 𝑎2 , … 𝑎𝑛 are constants. Equation (1) is very important in the study of
mechanical and electrical vibrations.
The simplest case of the D.E. is the first order (𝑤ℎ𝑒𝑛⁡𝑛 = 1), which takes the form;
𝑑𝑦
𝑎0 + 𝑎1 𝑦 = 0⁡⁡⁡⁡⁡⁡⁡⁡⁡⁡⁡⁡⁡⁡⁡⁡⁡⁡⁡⁡⁡⁡⁡⁡⁡⁡⁡⁡⁡⁡⁡(2)
𝑑𝑥
By separating the variables, the general solution is;
𝑦 = 𝐶𝑒 𝑚𝑥
𝑎
Where 𝐶 is a constant and 𝑚 = − 1
𝑎0

The above solution of the first order D.E. (2), suggests that 𝑦 = 𝐶𝑒 𝑚𝑥 may be a solution
of the 𝑛𝑡ℎ order D.E. (1), then;
𝑑𝑦 𝑚𝑥
𝑑2𝑦 𝑑𝑛 𝑦
= 𝑚𝑒 , = 𝑚 ⁡𝑒 ,⁡⁡⁡… … . , 𝑛 = 𝑚𝑛 ⁡𝑒 𝑚𝑥 ⁡
2 𝑚𝑥
𝑑𝑥 𝑑𝑥 2 𝑑𝑥
Substitute in equation (1), we have;

𝑎0 ⁡𝑚𝑛 ⁡𝑒 𝑚𝑥 ⁡ + 𝑎1 ⁡𝑚𝑛−1 ⁡𝑒 𝑚𝑥 ⁡ + 𝑎2 ⁡𝑚𝑛−2 ⁡𝑒 𝑚𝑥 ⁡ + ⋯ + 𝑎𝑛 ⁡𝑒 𝑚𝑥 ⁡ = 0⁡⁡⁡⁡⁡⁡⁡⁡⁡⁡⁡⁡⁡⁡⁡⁡⁡(3)

𝑒 𝑚𝑥 (𝑎0 ⁡𝑚𝑛 ⁡ + 𝑎1 ⁡𝑚𝑛−1 + 𝑎2 ⁡𝑚𝑛−2 ⁡ + ⋯ + 𝑎𝑛 ) = 0


Since 𝑒 𝑚𝑥 ≠ 0, then;

𝑎0 ⁡𝑚𝑛 ⁡ + 𝑎1 ⁡𝑚𝑛−1 + 𝑎2 ⁡𝑚𝑛−2 ⁡ + ⋯ + 𝑎𝑛 = 0⁡⁡⁡⁡⁡⁡⁡⁡⁡⁡⁡⁡⁡⁡⁡⁡⁡⁡⁡⁡⁡⁡⁡(4)


Equation (4) is an algebraic equation in 𝑚 and called the auxiliary equation with the given
differential equation (1).

Roots of the auxiliary equation:


The roots of the auxiliary equation take one of the following cases;
1) The roots are real and distinct
2) The roots are real and equal
3) The roots are complex
4) The roots are complex and equal

56
1) The roots are real and distinct
If the roots of the auxiliary equation are 𝑚1 , 𝑚2 , 𝑚3 … … 𝑚𝑛 are real and not equal, then
we have 𝑛 linearly independent solutions are the functions;

𝑒 𝑚1 𝑥 , 𝑒 𝑚2 𝑥 , 𝑒 𝑚3 𝑥 , … . . 𝑒 𝑚𝑛 𝑥
Then, the general solution (complementary function) 𝑦𝑐 has the form;

𝑦𝑐 = 𝐶1 𝑒 𝑚1𝑥 + 𝐶2 𝑒 𝑚2𝑥 + 𝐶3 𝑒 𝑚3𝑥 , … . . 𝐶𝑛 𝑒 𝑚𝑛𝑥

Where 𝐶1 , 𝐶2 , 𝐶3 , , … . . 𝐶𝑛 are arbitrary constants.

Example: Find the general solution of the following D.E.

𝑑2𝑦 𝑑𝑦
− 3 + 2𝑦 = 0
𝑑𝑥 2 𝑑𝑥
The auxiliary equation is;
𝑚2 − 3𝑚 + 2 = 0
(𝑚 − 1)(𝑚 − 2) = 0
Then the roots are 𝑚1 = 1, 𝑚2 = 2 which they are real and distinct. Then, the general
solution is;

𝑦 = 𝑦𝑐 = 𝐶1 𝑒 𝑥 + 𝐶2 𝑒 2𝑥

Example: Find the general solution of the following D.E.

𝑦 ′′′ + 3𝑦 ′′ − 6𝑦 ′ − 8𝑦 = 0
The auxiliary equation is;
𝑚3 + 3𝑚2 − 6𝑚 − 8 = 0
(𝑚 + 1)(𝑚 − 2)(𝑚 + 4) = 0
Then the roots are 𝑚1 = −1, 𝑚2 = 2, 𝑚3 = −4 which they are real and distinct. Then,
the general solution is;

𝑦 = 𝑦𝑐 = 𝐶1 𝑒 −𝑥 + 𝐶2 𝑒 2𝑥 + 𝐶3 𝑒 −4𝑥
2) The roots are real and equal
If the roots of the auxiliary equation are real and equal;
𝑚1 = ⁡ 𝑚2 = 𝑚3 = ⋯ = 𝑚𝑛 = 𝑘
57
Then, the general solution is;
𝑦 = 𝑦𝑐 = 𝑒 𝑘𝑥 (𝑎 + 𝑏𝑥 + 𝑐𝑥 2 + ⋯ )
where 𝑎, 𝑏, 𝑐, … are constants.

Example: Find the general solution of the following D.E.

𝑑2𝑦 𝑑𝑦
− 6 + 9𝑦 = 0
𝑑𝑥 2 𝑑𝑥
The auxiliary equation is;
𝑚2 − 6𝑚 + 9 = 0
(𝑚 − 3)(𝑚 − 3) = 0
Then the roots are 𝑚1 = 3, 𝑚2 = 3 which they are real and distinct. Then, the general
solution is;

𝑦 = 𝑦𝑐 = 𝑒 3𝑥 (𝑎 + 𝑏𝑥)

Example: Find the general solution of the following D.E.

𝑦 ′′′ − 3𝑦 ′′ + 3𝑦 ′ − 𝑦 = 0
The auxiliary equation is;
𝑚3 − 3𝑚2 + 3𝑚 − 1 = 0
(𝑚 − 1)(𝑚 − 1)(𝑚 − 1) = 0
Then the roots are 𝑚1 = 𝑚2 = 𝑚3 = 1 which they are real and distinct. Then, the
general solution is;
𝑦 = 𝑦𝑐 = 𝑒 𝑥 (𝑎 + 𝑏𝑥 + 𝑐𝑥 2 )

3) The roots are complex


The complex roots of the auxiliary equation are conjugate in pairs and may be written as;
𝑚1 = 𝛼 + 𝛽𝑖,⁡⁡⁡⁡⁡⁡⁡⁡⁡⁡⁡⁡⁡𝑚2 = 𝛼 − 𝛽𝑖
In this case, the general solution (complementary function) has the form;
𝑦 = 𝑦𝑐 = 𝑒 𝛼𝑥 (𝐶1 cos(𝛽𝑥) + 𝐶2 sin(𝛽𝑥))
Example: Find the general solution of the following D.E.

𝑑2𝑦 𝑑𝑦
+ 4 + 5𝑦 = 0
𝑑𝑥 2 𝑑𝑥
58
The auxiliary equation is;
𝑚2 + 4𝑚 + 5 = 0
−4 ± √16 − 20
𝑚1,2 = = −2 ± 𝑖
2
Then, the general solution is;
𝑦 = 𝑦𝑐 = 𝑒 −2𝑥 (𝐶1 cos(𝑥) + 𝐶2 sin(𝑥))

4) The roots are complex and equal


When the roots are complex and equal, 𝑚1,2 = 𝑚3,4 = 𝛼 ± 𝛽𝑖, then the general solution
takes the form;
𝑦 = 𝑦𝑐 = 𝑒 𝛼𝑥 (𝐶1 cos(𝛽𝑥) + 𝐶2 sin(𝛽𝑥)) + 𝑒 𝛼𝑥 (𝐶3 𝑥 cos(𝛽𝑥) + 𝐶4 𝑥 sin(𝛽𝑥))

𝑦 = 𝑦𝑐 = 𝑒 𝛼𝑥 [(𝐶1 + 𝐶3 𝑥) cos(𝛽𝑥) + (𝐶2 + 𝐶4 𝑥) sin(𝛽𝑥)]


Where 𝐶1 , 𝐶2 , 𝐶3 , 𝐶4 are arbitrary constants.

Example: Find the general solution of the following D.E.

𝑑4𝑦 𝑑2𝑦
+2 2+𝑦 =0
𝑑𝑥 4 𝑑𝑥
The auxiliary equation is;
𝑚4 + 2𝑚2 + 1 = 0
Let 𝑚2 = 𝑑
𝑑 2 + 2𝑑 + 1 = 0
−2 ± √4 − 4
𝑑= = −1
2
𝑚1,2 = 𝑚3,4 = ±𝑖

Then, the general solution is;


𝑦 = 𝑦𝑐 = (𝐶1 + 𝐶3 𝑥) cos(𝑥) + (𝐶2 + 𝐶4 𝑥) sin(𝑥)

Example: Find the general solution of the following D.E.

𝑦 ′′′′ − 2𝑦 ′′ − 3𝑦 = 0
59
The auxiliary equation is;
𝑚4 − 2𝑚2 − 3 = 0
2 ± √4 + 12
𝑚2 = = 3, −1
2
𝑚1,2 = ±𝑖,⁡⁡⁡𝑚3,4 = ±√3

Then, the general solution is;


𝑦 = 𝑦𝑐 = 𝐶1 cos(𝑥) + 𝐶2 sin(𝑥) + 𝐶3 𝑒 √3𝑥 + 𝐶4 𝑒 −√3𝑥

Exercise
1) Solve the following equations;
i) 𝑥 − 2𝑝 = 𝑙𝑛(𝑝)
ii) 𝑦 − 𝑝𝑥 = cos(𝑥)
iii) 𝑝2 − 2𝑝𝑥 − 3𝑥 2 = 0
iv) 𝑥 2 𝑝2 + 𝑥𝑝 − 𝑦 = 0
v) 𝑦 − 𝑥𝑝 = 𝑝2 − 1
vi) 𝑝2 − 2𝑝⁡𝑐𝑜𝑠ℎ(𝑥) + 1 = 0
vii) 𝑝2 − 4𝑝𝑥 − 8𝑥 2 = 0
viii) 𝑝2 + 𝑝 − 𝑒 𝑥 = 0
2
ix) 𝑦 = 2𝑝3 +
𝑝
2) Solve the following equations;
i) 𝑦′′ + 25𝑦 = 0
ii) 𝑦′′ + 𝑦′ − 6𝑦 = 0
iii) 𝑦′′′ − 𝑦′′ + 9𝑦′ − 9𝑦 = 0
iv) 𝑦′′ + 6𝑦′ + 9𝑦 = 0
v) 𝑦′′′′ + 3𝑦′′ = 0
vi) 𝑦′′ − 6𝑦′ − 16𝑦 = 0

60
The differential Operator
The differential operator is denoted by D and defined as follows;
𝑑 𝑑𝑦
𝐷= ,⁡⁡⁡𝑡ℎ𝑒𝑛⁡⁡⁡⁡⁡𝐷𝑦 = ⁡
𝑑𝑥 𝑑𝑥
Similarly,

2
𝑑2𝑦 𝑛
𝑑𝑛 𝑦
𝐷 𝑦 = 2,………….𝐷 𝑦 = 𝑛
𝑑𝑥 𝑑𝑥
Now, the higher order non- homogeneous linear differential equation:
𝑑𝑛 𝑦 𝑑 𝑛−1 𝑦 𝑑 𝑛−2 𝑦
𝑎0 𝑛 + 𝑎1 𝑛−1 + 𝑎2 𝑛−2 + ⋯ + 𝑎𝑛 𝑦 = 𝑃(𝑥)
𝑑𝑥 𝑑𝑥 𝑑𝑥
Where 𝑎0 , 𝑎1 , 𝑎2 , … 𝑎𝑛 and 𝑃 are functions of 𝑥 (or constants) can be rewritten in the
differential operator form as follows:

(𝑎0 𝐷𝑛 + 𝑎1 𝐷𝑛−1 + 𝑎2 𝐷𝑛−2 + ⋯ + 𝑎𝑛 )𝑦 = 𝑃(𝑥)


Or
𝐹(𝐷)𝑦 = 𝑃(𝑥)
Where 𝐹(𝐷) is an operator polynomial is given by:
𝐹(𝐷) = 𝑎0 𝐷𝑛 + 𝑎1 𝐷𝑛−1 + 𝑎2 𝐷𝑛−2 + ⋯ + 𝑎𝑛
The differential operator satisfies the algebraic laws;
Distributive law
If 𝑢 and 𝑣 are functions in 𝑥, then;
𝐹(𝐷)(𝑢 + 𝑣) = 𝐹(𝐷)⁡𝑢 + 𝐹(𝐷)⁡𝑣
Commutative law
If 𝑢 is function in 𝑥 and 𝐶 is a constant, then;
𝐹(𝐷)(𝐶⁡𝑢) = 𝐶⁡𝐹(𝐷)⁡𝑢
Note: if 𝐷 denoted the differential operator, then 𝐷−1 is the inverse differential
operator (integral).

The properties of operator polynomial


1- 𝐹(𝐷)𝑒 𝛼𝑥 = 𝑒 𝛼𝑥 ⁡𝐹(𝛼)
61
Examples:
❖ (𝐷3 + 2𝐷2 − 3𝐷)𝑒 2𝑥 = 𝑒 2𝑥 (8 + 8 − 6) = 10⁡𝑒 2𝑥
❖ (𝐷2 − 12𝐷 + 7)𝑒 −𝑥 = 𝑒 −𝑥 (1 + 12 + 7) = 20⁡𝑒 −𝑥

2- 𝐹(𝐷2 ) sin 𝛽𝑥 = sin 𝛽𝑥 ⁡𝐹(−𝛽2 )


Examples:
❖ (𝐷2 + 9) sin 2𝑥 = sin 2𝑥 (−4 + 9) = 5⁡ sin 2𝑥
2 2
❖ (𝐷3 − 3𝐷2 + 4𝐷 − 15) sin 3𝑥 = sin 3𝑥 (𝐷𝐷 − 3𝐷 ) = (12 − 5𝐷)⁡sin 3𝑥 =
+4𝐷 − 15
12 sin(3𝑥) − 15 cos(3𝑥)
Note:
𝐹(𝐷2 ) sin 𝛽𝑥 = sin 𝛽𝑥 ⁡𝐹(−𝛽 2 )
3- 𝐹(𝐷)𝑒 𝛼𝑥 𝑓(𝑥) = 𝑒 𝛼𝑥 𝐹(𝐷 + 𝛼)𝑓(𝑥)
Example:
❖ (𝐷2 + 3𝐷 − 6)𝑒 2𝑥 cos 2𝑥 = 𝑒 2𝑥 ((𝐷 + 2)2 + 3(𝐷 + 2) − 6)⁡cos 2𝑥 =
𝑒 2𝑥 (𝐷2 + 7𝐷 + 4)⁡cos 2𝑥 = 𝑒 2𝑥 (−4 + 7𝐷 + 4)⁡cos 2𝑥 = 14⁡𝑒 2𝑥 cos 2𝑥

4- 𝐹1 (𝐷)𝐹2 (𝐷)𝐹3 (𝐷)𝑓(𝑥) = 𝐹1 (𝐷)𝐹2 (𝐷)(𝐹3 (𝐷)𝑓(𝑥)) = 𝐹1 (𝐷)𝐹2 (𝐷)(𝑔(𝑥) =


𝐹1 (𝐷)(𝐹2 (𝐷)𝑔(𝑥)) = 𝐹1 (𝐷)(ℎ(𝑥)) = 𝑞(𝑥)

Example:
❖ 𝐷(𝐷 + 2)(𝐷2 − 5)𝑒 3𝑥 = 4𝐷(𝐷 + 2)𝑒 3𝑥 = 20𝐷𝑒 3𝑥 = 60𝑒 3𝑥

Particular Solution of Non-Homogeneous Linear D.E.


As mentioned before that, the higher order non-homogeneous linear D.E.
(𝑎0 𝐷𝑛 + 𝑎1 𝐷𝑛−1 + 𝑎2 𝐷𝑛−2 + ⋯ + 𝑎𝑛 )𝑦 = 𝑃(𝑥)
has a general solution (complementary function) 𝑦𝑐 depend on the roots of the auxiliary
equation. The particular solution 𝑦𝑝 of non-homogeneous D.E. depend on the function
𝑃(𝑥). The total solution is the sum of the general solution and the particular solution 𝑦𝑡 =
𝑦𝑐 + 𝑦𝑝 . Now, we shall discuss the different cases of the function 𝑃(𝑥) in this part.
62
Case 1- 𝑷(𝒙) = 𝒄𝒐𝒏𝒔𝒕𝒂𝒏𝒕
Put 𝐷 = 0, if the denominator equal zero using the integration
Example: Find the total solution of the differential equation
𝑦 ′′ − 𝑦 ′ − 6𝑦 = 9
The given equation in differential operator form;
(𝐷2 − 𝐷 − 6)𝑦 = 9
The auxiliary equation is
𝑚2 − 𝑚 − 6 = 0
(𝑚 − 3)(𝑚 + 2) = 0
𝑚 = −2, 3
The roots are real and distinct; thus, the general solution has a form;
𝑦𝑐 = 𝐶1 𝑒 −2𝑥 + 𝐶2 𝑒 3𝑥 ⁡⁡⁡⁡⁡⁡⁡⁡⁡⁡⁡⁡⁡⁡⁡⁡⁡⁡⁡⁡⁡⁡⁡⁡⁡⁡⁡⁡⁡⁡⁡⁡⁡⁡⁡⁡⁡⁡⁡⁡⁡(1)
To get the particular solution 𝑦𝑝 ;
9 −9 −3
𝑦𝑝 = = = ⁡⁡⁡⁡⁡⁡⁡⁡⁡⁡⁡⁡⁡⁡⁡⁡⁡⁡⁡⁡⁡⁡⁡⁡⁡⁡⁡(2)
𝐷2 − 𝐷 − 6 6 2
The total solution from (1), (2) is;
3
𝑦𝑡 = 𝑦𝑐 + 𝑦𝑝 = 𝐶1 𝑒 −2𝑥 + 𝐶2 𝑒 3𝑥 −
2
Example: Find the total solution of the differential equation
𝐷(𝐷 − 2)(𝐷 + 3)𝑦 = 1
The auxiliary equation is
𝑚(𝑚 − 2)(𝑚 + 3) = 0
𝑚 = 0, 2, −3
The roots are real and distinct; thus, the general solution has a form;
𝑦𝑐 = 𝐶1 𝑒 2𝑥 + 𝐶2 𝑒 −3𝑥 + 𝐶3 ⁡⁡⁡⁡⁡⁡⁡⁡⁡⁡⁡⁡⁡⁡⁡⁡⁡⁡⁡⁡⁡⁡⁡⁡⁡⁡⁡⁡⁡⁡⁡⁡⁡⁡⁡⁡⁡⁡⁡(1)
To get the particular solution 𝑦𝑝 ;

63
1
𝑦𝑝 = ⁡
𝐷(𝐷 − 2)(𝐷 + 3)
When we put 𝐷 = 0, then the denominator is zero, thus; the integration method can be
used
1 1 −𝑥
𝑦𝑝 = 𝐷−1 = 𝐷−1 = ⁡⁡⁡⁡⁡⁡⁡⁡⁡⁡⁡⁡⁡⁡⁡⁡⁡⁡⁡(2)
(𝐷 − 2)(𝐷 + 3) (−2)(3) 6
The total solution from (1), (2) is;
𝑥
𝑦𝑡 = 𝑦𝑐 + 𝑦𝑝 = 𝐶1 𝑒 2𝑥 + 𝐶2 𝑒 −3𝑥 + 𝐶3 −
6
Case 2- 𝑷(𝒙) = 𝒂𝒍𝒈𝒆𝒃𝒓𝒊𝒄⁡𝒇𝒖𝒏𝒄𝒕𝒊𝒐𝒏⁡𝒊𝒏⁡𝒙
In this case the series of the binomial theory should be used

Example: Find the total solution of the differential equation


(𝐷2 + 2𝐷 − 8)𝑦 = 𝑥 2 + 1
The auxiliary equation is
𝑚2 + 2𝑚 − 8 = 0
𝑚 = ⁡2, −4
The roots are real and distinct; thus, the general solution has a form;
𝑦𝑐 = 𝐶1 𝑒 2𝑥 + 𝐶2 𝑒 −4𝑥 ⁡⁡⁡⁡⁡⁡⁡⁡⁡⁡⁡⁡⁡⁡⁡⁡⁡⁡⁡⁡⁡⁡⁡⁡⁡⁡⁡⁡⁡⁡⁡⁡⁡⁡⁡⁡⁡⁡⁡(1)
To get the particular solution 𝑦𝑝 ;

𝑥2 + 1 𝑥2 + 1
𝑦𝑝 = = ⁡
𝐷2 + 2𝐷 − 8 𝐷2 + 2𝐷
−8 (1 +
−8 )
−1
1 𝐷2 + 2𝐷
𝑦𝑝 = (1 + ) (𝑥 2 + 1)
−8 −8
2
1 𝐷2 + 2𝐷 −1 ∗ −2 𝐷2 + 2𝐷
𝑦𝑝 = (1 + + ( ) + ⋯ ) (𝑥 2 + 1)
−8 8 2! −8

1 𝐷2 + 2𝐷 𝐷4 + 4𝐷2 + 4𝐷3
𝑦𝑝 = (1 + + ) (𝑥 2 + 1)
−8 8 64
64
1 𝐷2 + 2𝐷 4𝐷2
𝑦𝑝 = (1 + + ) (𝑥 2 + 1)
−8 8 64
1 2 + 4𝑥 8
𝑦𝑝 = (𝑥 2 + 1 + + )
−8 8 64
1 4𝑥 3
𝑦𝑝 = (𝑥 2 + 1 + + )⁡⁡⁡⁡⁡⁡⁡⁡⁡⁡⁡⁡⁡⁡⁡⁡⁡⁡⁡⁡⁡⁡⁡⁡⁡⁡⁡⁡⁡⁡⁡⁡⁡⁡⁡⁡(2)
−8 8 8
The total solution from (1), (2) is;
1 4𝑥 3
𝑦𝑡 = 𝑦𝑐 + 𝑦𝑝 = 𝐶1 𝑒 2𝑥 + 𝐶2 𝑒 −4𝑥 − (𝑥 2 + 1 + + )
8 8 8

Case 3- 𝑷(𝒙) = 𝒆𝒙𝒑𝒐𝒏𝒆𝒏𝒕𝒊𝒂𝒍⁡𝒇𝒖𝒏𝒄𝒕𝒊𝒐𝒏 = 𝑒 𝛼𝑥


We put 𝐷 = 𝛼, if the denominator is zero, we use 𝐷 = 𝐷 + 𝛼

Example: Find the total solution of the differential equation


(𝐷2 + 4)𝑦 = 𝑒 −𝑥
The auxiliary equation is
𝑚2 + 4 = 0
𝑚 = ⁡ ±2𝑖
The roots are complex; thus, the general solution has a form;
𝑦𝑐 = 𝐶1 cos⁡(2𝑥) + 𝐶2 sin⁡(2𝑥)⁡⁡⁡⁡⁡⁡⁡⁡⁡⁡⁡⁡⁡⁡⁡⁡⁡⁡⁡⁡⁡⁡⁡⁡⁡⁡⁡⁡⁡⁡⁡⁡⁡⁡⁡⁡⁡⁡⁡(1)
To get the particular solution 𝑦𝑝 ;
𝑒 −𝑥
𝑦𝑝 = 2
𝐷 +4
We put 𝐷 = −1
1 1
𝑦𝑝 = 𝑒 −𝑥 = 𝑒 −𝑥
⁡⁡⁡⁡⁡⁡⁡⁡⁡⁡⁡⁡⁡⁡⁡⁡⁡⁡⁡⁡⁡⁡⁡⁡⁡⁡⁡⁡⁡⁡⁡(2)
(−1)2 + 4 5
From (1) and (2) the total solution of the differential equation is;
𝑒 −𝑥
𝑦𝑡 = 𝑦𝑐 + 𝑦𝑝 = 𝐶1 cos(2𝑥) + 𝐶2 sin(2𝑥) +
5
65
Example: Find the total solution of the differential equation
(𝐷3 − 3𝐷2 + 3𝐷 − 1)𝑦 = 𝑒 𝑥
The auxiliary equation is
𝑚3 − 3𝑚2 + 3𝑚 − 1 = 0
𝑚 = 1,1,1
The roots are real and equal; thus, the general solution has a form;
𝑦𝑐 = 𝑒 𝑥 (𝐶1 + 𝐶2 𝑥 + 𝐶3 𝑥 2 )⁡⁡⁡⁡⁡⁡⁡⁡⁡⁡⁡⁡⁡⁡⁡⁡⁡⁡⁡⁡⁡⁡⁡⁡⁡⁡⁡⁡⁡⁡⁡⁡⁡⁡⁡⁡⁡⁡⁡(1)
To get the particular solution 𝑦𝑝 ;
𝑒𝑥 𝑒𝑥
𝑦𝑝 = 3 =
𝐷 − 3𝐷2 + 3𝐷 − 1 (𝐷 − 1)3
When we put 𝐷 = 1, the denominator is zero, thus we use 𝐷 = 𝐷 + 1

𝑥
1 𝑥
1 𝑥 (𝐷)−3 (1)
𝑥3 𝑥
𝑦𝑝 = 𝑒 =𝑒 =𝑒 = 𝑒 ⁡⁡⁡⁡⁡⁡⁡⁡⁡⁡⁡⁡⁡⁡⁡⁡⁡⁡⁡⁡(2)
(𝐷 + 1 − 1)3 (𝐷)3 6
From (1) and (2) the total solution of the differential equation is;
𝑥3 𝑥
𝑦𝑡 = 𝑦𝑐 + 𝑦𝑝 = 𝑒 𝑥 (𝐶1 + 𝐶2 𝑥 + 𝐶3 𝑥 2)
+ 𝑒
6

Case 4- 𝑷(𝒙) = 𝒕𝒓𝒊𝒈𝒐𝒏𝒐𝒎𝒆𝒕𝒓𝒊𝒄⁡𝒇𝒖𝒏𝒄𝒕𝒊𝒐𝒏⁡(𝒔𝒊𝒏⁡(𝜷𝒙)⁡𝒐𝒓⁡𝒄𝒐𝒔(𝜷𝒙))


We put 𝐷2 = −𝛽 2 , if the denominator is zero, we replace the trigonometric function
into exponential function as Euler rule for complex number, then we decide the
particular solution according to the trigonometric function

𝑒 𝑖𝛼𝑥 = cos(𝛼𝑥) + 𝑖⁡𝑠𝑖𝑛(𝛼𝑥)

Example: Find the total solution of the differential equation


(𝐷2 + 5)𝑦 = sin⁡(2𝑥)
The auxiliary equation is
𝑚2 + 5 = 0

𝑚 = ⁡ ±√5𝑖
66
The roots are complex; thus, the general solution has a form;

𝑦𝑐 = 𝐶1 cos⁡(√5𝑥) + 𝐶2 sin⁡(√5𝑥)⁡⁡⁡⁡⁡⁡⁡⁡⁡⁡⁡⁡⁡⁡⁡⁡⁡⁡⁡⁡⁡⁡⁡⁡⁡⁡⁡⁡⁡⁡⁡⁡⁡⁡⁡⁡⁡⁡⁡(1)
To find the particular solution 𝑦𝑝 ;
sin⁡(2𝑥)
𝑦𝑝 =
𝐷2 + 5
We put 𝐷2 = −𝛽 2 = −4
𝑦𝑝 = sin⁡(2𝑥)⁡⁡⁡⁡⁡⁡⁡⁡⁡⁡⁡⁡⁡⁡⁡⁡⁡⁡⁡⁡⁡⁡⁡⁡⁡⁡⁡⁡⁡⁡⁡(2)

From (1) and (2) the total solution of the differential equation is;

𝑦𝑡 = 𝑦𝑐 + 𝑦𝑝 = 𝐶1 cos⁡(√5𝑥) + 𝐶2 sin⁡(√5𝑥) + sin⁡(2𝑥)

Example: Find the total solution of the differential equation


(𝐷3 − 1)𝑦 = cos⁡(𝑥)
The auxiliary equation is
𝑚3 − 1 = 0

1 √3
𝑚 = 1, − ± 𝑖
2 2
The roots are complex; thus, the general solution has a form;

√3 √3
𝑦𝑐 = 𝐶1 𝑒 𝑥 + 𝑒 −0.5𝑥 (𝐶2 cos⁡( 𝑥) + 𝐶3 sin⁡( 𝑥))⁡⁡⁡⁡⁡⁡⁡⁡⁡⁡⁡⁡⁡⁡⁡⁡⁡⁡⁡⁡⁡⁡⁡⁡⁡⁡⁡⁡⁡⁡⁡⁡⁡⁡⁡⁡⁡⁡⁡(1)
2 2

To find the particular solution 𝑦𝑝 ;


cos⁡(𝑥)
𝑦𝑝 =
𝐷3 − 1
We put 𝐷2 = −𝛽 2 = −1
cos(𝑥) cos(𝑥)
𝑦𝑝 = =−
−𝐷 − 1 𝐷+1
Multiply by conjugate (𝐷 − 1), we have;
cos(𝑥) 𝐷 − 1 𝐷−1
𝑦𝑝 = − × =− 2 cos(𝑥)
𝐷+1 𝐷−1 𝐷 −1
67
𝐷−1 1 1
𝑦𝑝 = − cos(𝑥) = (𝐷 − 1) cos(𝑥) = (− sin(𝑥) − cos(𝑥))⁡⁡⁡⁡⁡⁡⁡⁡⁡⁡⁡⁡(2)
−1 − 1 2 2
From (1) and (2) the total solution of the differential equation is;

√3 √3 1
𝑦𝑡 = 𝑦𝑐 + 𝑦𝑝 = 𝐶1 𝑒 𝑥 + 𝑒 −0.5𝑥 (𝐶2 cos⁡( 𝑥) + 𝐶3 sin⁡( 𝑥)) − (sin(𝑥) + cos⁡(𝑥))
2 2 2

Example: Find the total solution of the differential equation


(𝐷2 + 4)𝑦 = sin⁡(2𝑥)
The auxiliary equation is
𝑚2 + 4 = 0
𝑚 = ±2𝑖
The roots are complex; thus, the general solution has a form;
𝑦𝑐 = 𝐶1 cos⁡(2𝑥) + 𝐶2 sin⁡(2𝑥)⁡⁡⁡⁡⁡⁡⁡⁡⁡⁡⁡⁡⁡⁡⁡⁡⁡⁡⁡⁡⁡⁡⁡⁡⁡⁡⁡⁡⁡⁡⁡⁡⁡⁡⁡⁡⁡⁡⁡(1)
To find the particular solution 𝑦𝑝 ;
sin⁡(2𝑥)
𝑦𝑝 =
𝐷2 + 4
We put 𝐷2 = −𝛽 2 = −4, the denominator is zero, we replace the trigonometric function
sin⁡(2𝑥) into exponential function 𝑒 2𝑖𝑥 , then

𝑒 2𝑖𝑥 2𝑖𝑥
1
= 𝑒
𝐷2 + 4 (2𝑖)2 + 4
When we put the effect of the exponential function on the differential operator 𝐷 = 2𝑖,
the denominator is zero, thus; we can use 𝐷 = 𝐷 + 2𝑖

𝑒 2𝑖𝑥 2𝑖𝑥
1 2𝑖𝑥
1 2𝑖𝑥
1
= 𝑒 = 𝑒 = 𝑒
𝐷2 + 4 (𝐷 + 2𝑖)2 + 4 𝐷2 + 4𝑖𝐷 − 4 + 4 𝐷2 + 4𝑖𝐷
When we put the effect of the constant on the differential operator 𝐷 = 0, the
denominator is zero, thus; we can use the integration method;
1 1 1 𝑥 𝑥𝑖 2𝑖𝑥
𝑒 2𝑖𝑥 = 𝑒 2𝑖𝑥 𝐷−1 = 𝑒 2𝑖𝑥 𝐷−1 = 𝑒 2𝑖𝑥 = 𝑒
𝐷2 + 4𝑖𝐷 𝐷 + 4𝑖 4𝑖 4𝑖 −4
We replace the function by using the Euler formula for complex number;

68
𝑥𝑖 2𝑖𝑥 𝑥𝑖 𝑥⁡𝑠𝑖𝑛(2𝑥) 𝑥⁡𝑐𝑜𝑠(2𝑥)
𝑒 = (cos(2𝑥) + 𝑖⁡𝑠𝑛(2𝑥)) = − 𝑖
−4 −4 4 4
Because of the 𝑃(𝑥) = sin⁡(2𝑥) in the original differential equation, we choose the
imaginary part of the solution
𝑥 cos(2𝑥)
𝑦𝑝 = − ⁡⁡⁡⁡⁡⁡⁡⁡⁡⁡⁡⁡⁡⁡⁡⁡⁡⁡⁡⁡⁡⁡⁡(2)
4
From (1) and (2) the total solution of the differential equation is;
𝑥 cos(2𝑥)
𝑦𝑡 = 𝑦𝑐 + 𝑦𝑝 = 𝐶1 cos⁡(2𝑥) + 𝐶2 sin⁡(2𝑥) −
4
Note: if the original differential equation has 𝑃(𝑥) = cos⁡(2𝑥), we choose the real part of
the solution.
Case 5- 𝑷(𝒙) = 𝒆𝜶𝒙 𝒇(𝒙)
in this case we first analyse the effect of the exponential function 𝑒 𝛼𝑥 on the differential
operator by 𝐷 = 𝐷 + 𝛼, then study the effect of 𝑓(𝑥) on the differential operator
according its classification.

Example: Find the total solution of the differential equation


𝑦 ′′ + 2𝑦 ′ + 2𝑦 = 𝑒 −𝑥 𝑥 2
The differential equation can be written as;
(𝐷2 + 2𝐷 + 2)𝑦 = 𝑒 −𝑥 𝑥 2
The auxiliary equation is
𝑚2 + 2𝑚 + 2 = 0
𝑚 = −1 ± 𝑖
The roots are complex; thus, the general solution has a form;
𝑦𝑐 = 𝑒 −𝑥 (𝐶1 cos⁡(𝑥) + 𝐶2 sin⁡(𝑥))⁡⁡⁡⁡⁡⁡⁡⁡⁡⁡⁡⁡⁡⁡⁡⁡⁡⁡⁡⁡⁡⁡⁡⁡⁡⁡⁡⁡⁡⁡⁡⁡⁡⁡⁡⁡⁡⁡⁡(1)
To find the particular solution 𝑦𝑝 ;

𝑒 −𝑥 𝑥 2
𝑦𝑝 = 2
𝐷 + 2𝐷 + 2
First, we put 𝐷 = 𝐷 − 1

69
𝑥2 −𝑥
𝑥2 −𝑥
𝑥2
𝑦𝑝 = 2 =𝑒 =𝑒
𝐷 + 2𝐷 + 2 (𝐷 − 1)2 + 2(𝐷 − 1) + 2 𝐷2 + 1
Second, we use the binomial theory as;

−𝑥
𝑥2
𝑦𝑝 = 𝑒 2
= 𝑒 −𝑥 (1 + 𝐷2 )−1 𝑥 2 = 𝑒 −𝑥 (1 − 𝐷2 )𝑥 2 = 𝑒 −𝑥 (𝑥 2 − 2)⁡⁡⁡⁡⁡⁡⁡⁡(2)
𝐷 +1

From (1) and (2) the total solution of the differential equation is;
𝑦𝑡 = 𝑦𝑐 + 𝑦𝑝 = 𝑒 −𝑥 (𝐶1 cos⁡(𝑥) + 𝐶2 sin⁡(𝑥)) + 𝑒 −𝑥 (𝑥 2 − 2)

Example: Find the total solution of the differential equation


(𝐷2 + 3𝐷 + 2)𝑦 = 𝑒 2𝑥 cos⁡(𝑥)
The auxiliary equation is
𝑚2 + 3𝑚 + 2 = 0
𝑚 = −1, −2
The roots are real and distinct; thus, the general solution has a form;
𝑦𝑐 = 𝐶1 𝑒 −𝑥 + 𝐶2 𝑒 −2𝑥 ⁡⁡⁡⁡⁡⁡⁡⁡⁡⁡⁡⁡⁡⁡⁡⁡⁡⁡⁡⁡⁡⁡⁡⁡⁡⁡⁡⁡⁡⁡⁡⁡⁡⁡⁡⁡⁡⁡⁡(1)
To find the particular solution 𝑦𝑝 ;

𝑒 2𝑥 cos⁡(𝑥)
𝑦𝑝 = 2
𝐷 + 3𝐷 + 2
First, we put 𝐷 = 𝐷 + 2
𝑒 2𝑥 cos⁡(𝑥) cos⁡(𝑥) cos⁡(𝑥)
𝑦𝑝 = 2 = 𝑒 2𝑥 = 𝑒 2𝑥
𝐷 + 3𝐷 + 2 (𝐷 + 2)2 + 3(𝐷 + 2) + 2 𝐷2 + 7𝐷 + 12
Second, we use the 𝐷2 = −𝛽 2 = −1;
cos⁡(𝑥) cos⁡(𝑥)
𝑦𝑝 = 𝑒 2𝑥 = 𝑒 2𝑥
𝐷2 + 7𝐷 + 12 7𝐷 + 11
Multiply by conjugate (7𝐷 − 11)

2𝑥
cos(𝑥) 2𝑥
(7𝐷 − 11) cos(𝑥) 𝑒 2𝑥
𝑦𝑝 = 𝑒 =𝑒 = (7𝐷 − 11) cos(𝑥)
7𝐷 + 11 49𝐷2 − 121 −170

70
𝑒 2𝑥
𝑦𝑝 = (−7 sin(𝑥) − 11cos⁡(𝑥))⁡⁡⁡⁡⁡⁡⁡⁡⁡⁡⁡⁡⁡(2)
−170
From (1) and (2) the total solution of the differential equation is;

−𝑥 −2𝑥
𝑒 2𝑥
𝑦𝑡 = 𝑦𝑐 + 𝑦𝑝 = 𝐶1 𝑒 + 𝐶2 𝑒 + (7 sin(𝑥) + 11cos⁡(𝑥))
170

Example: Find the total solution of the differential equation


(𝐷2 + 1)𝑦 = 𝑥⁡cos⁡(𝑥)
The auxiliary equation is
𝑚2 + 1 = 0
𝑚 = ±𝑖
The roots are complex; thus, the general solution has a form;
𝑦𝑐 = 𝐶1 cos⁡(𝑥) + 𝐶2 sin⁡(𝑥)⁡⁡⁡⁡⁡⁡⁡⁡⁡⁡⁡⁡⁡⁡⁡⁡⁡⁡⁡⁡⁡⁡⁡⁡⁡⁡⁡⁡⁡⁡⁡⁡⁡⁡⁡⁡⁡⁡⁡(1)
To find the particular solution 𝑦𝑝 ;
𝑥⁡cos⁡(𝑥)
𝑦𝑝 =
𝐷2 + 1
First, by using the Euler formula; we replace cos(𝑥) = 𝑒 𝑖𝑥

𝑥⁡𝑒 𝑖𝑥 𝑒 𝑖𝑥 ⁡𝑥⁡ 𝑖𝑥
𝑥 𝑖𝑥
𝑥
= = 𝑒 = 𝑒
𝐷2 + 1 (𝐷 + 𝑖)2 + 1 𝐷2 + 2𝑖𝐷 𝐷
2𝑖𝐷 (1 + )
2𝑖

𝑖𝑥
𝑥 𝑒 𝑖𝑥 −1 𝐷 −1 𝑒 𝑖𝑥 −1 1
𝑒 = 𝐷 (1 + ) 𝑥 = 𝐷 (𝑥 − )
𝐷
2𝑖𝐷 (1 + ) 2𝑖 2𝑖 2𝑖 2𝑖
2𝑖
𝑒 𝑖𝑥 −1 1 𝑒 𝑖𝑥 𝑥 2 𝑥
𝐷 (𝑥 − ) = ( − )
2𝑖 2𝑖 2𝑖 2 2𝑖

Return to the Euler formula;

71
𝑒 𝑖𝑥 𝑥 2 𝑥 cos(𝑥) + 𝑖 sin(𝑥) 𝑥 2 𝑥 𝑥2 𝑥
( − )= ( − ) = (cos(𝑥) + 𝑖 sin(𝑥)) ( + )
2𝑖 2 2𝑖 2𝑖 2 2𝑖 4𝑖 4
1
= (𝑥 − 𝑖𝑥 2 )(cos(𝑥) + 𝑖 sin(𝑥))
4
1
= (𝑥𝑐𝑜𝑠(𝑥) + 𝑥 2 ⁡sin⁡(𝑥) − 𝑖𝑥 2 cos(𝑥) + 𝑥𝑖 sin(𝑥))
4
Because of the original differential equation has 𝑐𝑜𝑠(𝑥) we choose the real part of the
solution.
1
𝑦𝑝 = (𝑥𝑐𝑜𝑠(𝑥) + 𝑥 2 ⁡sin⁡(𝑥))⁡⁡⁡⁡⁡⁡⁡⁡⁡⁡⁡⁡⁡(2)
4
From (1) and (2) the total solution of the differential equation is;
1
𝑦𝑡 = 𝑦𝑐 + 𝑦𝑝 = 𝐶1 cos⁡(𝑥) + 𝐶2 sin⁡(𝑥) + (𝑥𝑐𝑜𝑠(𝑥) + 𝑥 2 ⁡sin⁡(𝑥))
4

Exercise
Find the complete solution of the following differential equations;
1) (𝐷2 − 16)𝑦 = 32
2) (𝐷3 − 𝐷2 − 2𝐷)𝑦 = 2
3) (𝐷2 + 3𝐷 − 18)𝑦 = 15
4) 𝐷2 (𝐷 + 1)(𝐷 − 2)𝑦 = 𝑥 2 + 1
5) (𝐷2 + 2𝐷 + 1)𝑦 = 2𝑥 3 + 1
6) (𝐷4 − 2𝐷2 + 1)𝑦 = 𝑥 2 − 3𝑥 + 2
7) (𝐷2 − 1)𝑦 = 𝑥 2 − 2𝑥 + 1
8) (𝐷2 + 𝐷 − 6)𝑦 = 𝑒 𝑥
9) (𝐷2 − 4𝐷 − 5)𝑦 = sinh(3𝑥)
10) (𝐷2 − 3𝐷 − 4)𝑦 = 𝑒 −𝑥
11) (𝐷3 − 𝐷2 − 2𝐷)𝑦 = 2𝑒 𝑥
12) (𝐷2 + 𝐷 + 3)𝑦 = cos(2𝑥) , (𝐷2 + 1)𝑦 = sin(3𝑥)
13) (𝐷2 + 2𝐷 − 3)𝑦 = sin(𝑥) + cos(2𝑥)
14) (𝐷4 − 2𝐷2 − 3)𝑦 = cos⁡(2𝑥)
15) (𝐷3 − 3𝐷)𝑦 = sin(𝑥)
16) 𝐷2 (𝐷 − 1)𝑦 = 𝑒 −𝑥 cos(𝑥)
17) (𝐷2 − 2𝐷 − 3)𝑦 = 𝑒 𝑥 ⁡𝑥 2
18) (𝐷3 − 𝐷2 − 2𝐷)𝑦 = 𝑐𝑜𝑠 2 (𝑥)
19) (𝐷2 − 2𝐷 − 8)𝑦 = 2𝑥 − 𝑒 2𝑥
72
20) (𝐷2 − 3𝐷 + 2)𝑦 = 𝑥⁡sin⁡(𝑥)
21) (𝐷3 − 𝐷2 − 2𝐷)𝑦 = 2 − 𝑥 + sin(𝑥)
22) (𝐷2 + 2𝐷 + 1)𝑦 = 𝑒 −𝑥 + 𝑥 + ⁡sin⁡(2𝑥)

Euler-Cauchy Differential Equations


These equations take the form;
𝑑𝑛 𝑦
𝑛 𝑛−1
𝑑 𝑛−1 𝑦 𝑛−2
𝑑 𝑛−2 𝑦
𝑎0 𝑥 + 𝑎1 𝑥 + 𝑎2 𝑥 + ⋯ + 𝑎𝑛 𝑦 = 𝑓(𝑥)⁡⁡⁡⁡⁡⁡⁡⁡⁡⁡⁡⁡⁡(1)
𝑑𝑥 𝑛 𝑑𝑥 𝑛−1 𝑑𝑥 𝑛−2

Bu using the differential operator, this equation can be written as;

(𝑎0 𝑥 𝑛 𝐷𝑛 + 𝑎1 𝑥 𝑛−1 𝐷𝑛−1 + 𝑎2 𝑥 𝑛−2 𝐷𝑛−2 + ⋯ + 𝑎𝑛 )𝑦 = 𝑓(𝑥)

𝐹(𝑥𝐷)𝑦 = 𝑓(𝑥)

where 𝑎0 , 𝑎1 , 𝑎2 , … 𝑎𝑛 are constants. Equation (1) is a linear differential equation with


coefficients are functions in 𝑥, thus we cannot solve it by using previous methods for
differential equations. Equation (1) can be transform into a linear differential equation
with constant coefficients, using a new independent variable 𝑡, and the substitution;

𝑥 = 𝑒 𝑡 ,⁡⁡⁡⁡⁡⁡⁡⁡⁡⁡𝑡 = ln(𝑥)⁡⁡⁡⁡⁡⁡⁡⁡⁡⁡⁡⁡⁡⁡⁡⁡⁡⁡⁡⁡⁡⁡⁡⁡⁡⁡⁡⁡⁡⁡⁡⁡⁡⁡⁡⁡⁡(2)

Differentiate equation (2) w.r.t 𝑡, we have;


𝑑𝑥 𝑑𝑡 1
= 𝑒 𝑡 ,⁡⁡⁡⁡⁡⁡⁡⁡⁡⁡⁡⁡⁡⁡ = 𝑒 −𝑡 =
𝑑𝑡 𝑑𝑥 𝑥
𝑑
If we define a new differential operator 𝜃 to be 𝜃 = , then we have;
𝑑𝑡

𝑑𝑦 𝑑𝑦 𝑑𝑡 1 𝑑𝑦
= = ⁡⁡⁡⁡⁡⁡⁡⁡⁡⁡⁡⁡⁡⁡⁡⁡⁡⁡⁡⁡⁡⁡⁡⁡⁡⁡⁡⁡⁡⁡⁡⁡(3)
𝑑𝑥 𝑑𝑡 𝑑𝑥 𝑥 𝑑𝑡
𝑑 𝑑
Using the definitions of 𝐷 = , 𝜃= in equation (3), then;
𝑑𝑥 𝑑𝑡

1
𝐷𝑦 = 𝜃𝑦⁡⁡⁡⁡⁡⁡⁡⁡⁡⁡⁡⁡ ⇾⇾⇾ ⁡⁡⁡⁡⁡⁡𝑥𝐷𝑦 = 𝜃𝑦⁡⁡⁡⁡⁡ ⇾⇾⇾ ⁡⁡⁡⁡⁡𝑥𝐷 = 𝜃
𝑥
Similarly, we can analyze the higher differentiation order as;

73
𝑑2𝑦 𝑑 𝑑𝑦 𝑑 𝑑𝑦 𝑑𝑡 𝑑 1 𝑑𝑦 1 𝑑 𝑑𝑦 𝑑𝑦 𝑑 1
= ( ) = ( ) = ( ) = ( ) + ( )
𝑑𝑥 2 𝑑𝑥 𝑑𝑥 𝑑𝑥 𝑑𝑡 𝑑𝑥 𝑑𝑥 𝑥 𝑑𝑡 𝑥 𝑑𝑥 𝑑𝑡 𝑑𝑡 𝑑𝑥 𝑥
1 𝑑2𝑦 𝑑𝑦 𝑑 1 1 𝑑 2 𝑦 𝑑𝑡 1 𝑑𝑦 1 𝑑 2 𝑦 1 1 𝑑𝑦
= + ( )= ( ) − = ( ) −
𝑥 𝑑𝑥⁡𝑑𝑡 𝑑𝑡 𝑑𝑥 𝑥 𝑥 𝑑𝑡 2 𝑑𝑥 𝑥 2 𝑑𝑡 𝑥 𝑑𝑡 2 𝑥 𝑥 2 𝑑𝑡
1 2 1 1 2
𝐷2 𝑦 = 𝜃 𝑦 − ⁡ 𝜃𝑦 = (𝜃 − ⁡𝜃)𝑦
𝑥2 𝑥2 𝑥2
𝑥 2 𝐷2 = 𝜃 2 − ⁡𝜃 = 𝜃(𝜃 − 1)
𝑥 3 𝐷3 = 𝜃(𝜃 − 1)(𝜃 − 2)
𝑥 𝑛 𝐷𝑛 = 𝜃(𝜃 − 1)(𝜃 − 2) … … . (𝜃 − 𝑛 + 1)
By using this transformation, equation (1) convert to a linear differential equation with
constant coefficients, which can be solve using forementioned methods.

Example: Find the complete solution of the differential equation;


(𝑥 2 𝐷2 + 3𝑥𝐷 − 3)𝑦 = 𝑥
Using the previous substitution in the differential equation
𝑥 = 𝑒 𝑡 ,⁡⁡⁡⁡⁡⁡⁡⁡⁡⁡𝑡 = ln(𝑥) ,⁡⁡⁡⁡⁡⁡⁡⁡𝑥𝐷 = 𝜃, 𝑥 2 𝐷2 = 𝜃(𝜃 − 1)
[𝜃(𝜃 − 1) + 3𝜃 − 3]𝑦 = 𝑒 𝑡
(𝜃 2 + 2𝜃 − 3)𝑦 = 𝑒 𝑡
The auxiliary equation is
𝑚2 + 2𝑚 − 3 = 0
𝑚 = 1, −3
The roots are real and distinct; thus, the general solution has a form;
𝑦𝑐 = 𝐶1 𝑒 𝑡 + 𝐶2 𝑒 −3𝑡 ⁡⁡⁡⁡⁡⁡⁡⁡⁡⁡⁡⁡⁡⁡⁡⁡⁡⁡⁡⁡⁡⁡⁡⁡⁡⁡⁡⁡⁡⁡⁡⁡⁡⁡⁡⁡⁡⁡⁡(1)
To get the particular solution;
𝑒𝑡
𝑦𝑝 =
(𝜃 − 1)(𝜃 + 3)
In the first bracket put 𝜃 = 𝜃 + 1, and in the second bracket put 𝜃 = 1, then;

74
𝑡
1 𝑡𝑒 𝑡
𝑦𝑝 = 𝑒 = ⁡⁡⁡⁡⁡⁡⁡⁡⁡⁡⁡⁡⁡⁡⁡⁡⁡⁡⁡⁡⁡⁡⁡⁡⁡⁡⁡⁡⁡⁡(2)
4(𝜃 + 1 − 1) 4
The total solution from (1) and (2)

𝑡 −3𝑡
𝑡𝑒 𝑡
𝑦𝑡 = 𝑦𝑐 + 𝑦𝑝 = 𝐶1 𝑒 + 𝐶2 𝑒 +
4
Substitute for 𝑥, we have;
1 𝑥 ln(𝑥)
𝑦 = 𝐶1 𝑥 + 𝐶2 +
𝑥3 4

Orthogonal Trajectories
Suppose we have a one parameter family of curves;
𝜙(𝑥, 𝑦, 𝐶) = 0,⁡⁡⁡⁡⁡⁡⁡⁡𝑤ℎ𝑒𝑟𝑒⁡𝐶 = 𝑐𝑜𝑛𝑠𝑡𝑎𝑛𝑡⁡⁡⁡⁡⁡⁡⁡⁡⁡⁡⁡⁡⁡⁡⁡⁡⁡⁡⁡⁡(1)
To get the equation of orthogonal the trajectories, and eliminate C from the equation (1)
follow the steps;
1) Write the differential equation of the given family of curves and eliminating the
parameter C from the equations, we get the differential equation of the family (1) in
the form;
𝑑𝑦
𝐹 (𝑥, 𝑦, ) = 0⁡⁡⁡⁡⁡⁡⁡⁡⁡⁡⁡⁡⁡⁡⁡⁡⁡⁡⁡⁡⁡⁡⁡⁡⁡⁡⁡(2)
𝑑𝑥
𝑑𝑦
2) Here, is the slope of the tangent to family (1). The orthogonal trajectories are
𝑑𝑥
𝑑𝑦 𝑡
perpendicular to the curves (1), the slope of tangent to them is ( ) ; which
𝑑𝑥
𝑑𝑦
connected with by the relation:
𝑑𝑥
𝑚1 × 𝑚2 = −1
𝑑𝑦 𝑑𝑦 𝑡
( ) × ( ) = −1
𝑑𝑥 𝑑𝑥
𝑑𝑦 𝑡 𝑑𝑥
( ) = − ⁡⁡⁡⁡⁡⁡⁡⁡⁡⁡⁡⁡⁡⁡⁡⁡⁡⁡⁡⁡⁡⁡⁡⁡⁡⁡⁡⁡⁡⁡⁡⁡⁡(3)
𝑑𝑥 𝑑𝑦
3) Now dropping the superscript t from expression (3) and substitute into equation (2),
we get the differential equation of the orthogonal trajectories;

75
𝑑𝑥
𝐹 (𝑥, 𝑦, − ) = 0⁡⁡⁡⁡⁡⁡⁡⁡⁡⁡⁡⁡⁡⁡⁡⁡⁡⁡⁡⁡⁡⁡⁡⁡⁡⁡⁡(4)
𝑑𝑦
4) Integrating equation (4) yields the family of the orthogonal trajectories.
Example: Find the orthogonal trajectories of the curve;
𝑦 = 𝐶𝑥 2 ⁡⁡⁡⁡⁡⁡⁡⁡⁡⁡⁡⁡⁡⁡⁡⁡⁡⁡⁡⁡⁡⁡⁡(𝐼)
The given family of curves is parabolas with 𝑦 axis is the axis of them. Differentiate
equation (𝐼) w.r.t 𝑥 to get the differential equation of them;
𝑑𝑦
= 2𝐶𝑥⁡⁡⁡⁡⁡⁡⁡⁡⁡⁡(𝐼𝐼)
𝑑𝑥
Eliminate the parameter C between equations (𝐼), (𝐼𝐼), we have;
𝑑𝑦 𝑦
= 2 ⁡⁡⁡⁡⁡⁡⁡⁡⁡⁡⁡⁡⁡⁡⁡⁡⁡⁡⁡⁡⁡⁡⁡⁡⁡⁡⁡(𝐼𝐼𝐼)
𝑑𝑥 𝑥
𝑑𝑦 𝑑𝑥
To get the differential equation of the orthogonal family, put =− into equation
𝑑𝑥 𝑑𝑦
(𝐼𝐼𝐼), then;
𝑑𝑥 𝑦
− =2
𝑑𝑦 𝑥
−𝑥⁡𝑑𝑥 = 2𝑦⁡𝑑𝑦
The last equation is a differential equation of the orthogonal trajectories, which is a first
order first degree differential equation, integrate it to get the equation of the orthogonal
curves;
𝑥2
− + 𝑘 = 𝑦2
2

2
𝑥2
𝑦 + =𝑘
2
Hence, the orthogonal trajectories of the given parabolas are a family of ellipses.

76
System of simultaneous Differential Equations
A system of differential equations is called linear if the dependent variables and there
derivatives are linear. Consider the following system of first order first degree differential
equations;
𝑑𝑦1
= 𝑓1 (𝑥, 𝑦1 , 𝑦2 , … … . . , 𝑦𝑛 )
𝑑𝑥
𝑑𝑦2
= 𝑓2 (𝑥, 𝑦1 , 𝑦2 , … … . . , 𝑦𝑛 )
𝑑𝑥
𝑑𝑦𝑛
= 𝑓𝑛 (𝑥, 𝑦1 , 𝑦2 , … … . . , 𝑦𝑛 )
𝑑𝑥
where 𝑦1 , 𝑦2 , … … . . , 𝑦𝑛 are unknown functions and x is the argument. This system is called
normal where the left side contain first order derivatives, while the right side not contain
derivatives. To determine the unknown functions 𝑦1 , 𝑦2 , … … . . , 𝑦𝑛 in the previous system,
eliminate all unknown functions except one of the systems, then we get only one
differential equations of higher order to evaluate this function. Obtain the other functions
from the given system without integration (if possible).

Example: Solve the following system of simultaneous linear differential equation;


𝑑𝑦
− 𝑧 = 0⁡⁡⁡⁡⁡⁡⁡⁡⁡⁡⁡⁡⁡⁡⁡⁡⁡(1)
𝑑𝑥
𝑑𝑧
− 𝑦 = 0⁡⁡⁡⁡⁡⁡⁡⁡⁡⁡⁡⁡⁡⁡⁡(2)
𝑑𝑥
𝑑
where 𝐷 = , the argument is 𝑥 and the unknown functions are 𝑦, 𝑧. Multiply the first
𝑑𝑥
equation by 𝐷 and adding to the second equation, we get;
𝐷2 𝑦 − 𝐷𝑧 = 0
𝐷𝑧 − 𝑦 = 0
Which is a higher order, homogeneous linear differential equation in one unknown
function;
𝐷2 𝑦 − 𝑦 = 0
(𝐷2 − 1)𝑦 = 0
The auxiliary equation;
77
𝑚2 − 1 = 0
𝑚 = −1,1
The general solution is;
𝑦 = 𝐶1 𝑒 𝑥 + 𝐶1 𝑒 −𝑥
To get 𝑧, subistitute for 𝑦 in first equation
𝑑𝑦
𝑧= = 𝐶1 𝑒 𝑥 − 𝐶1 𝑒 −𝑥
𝑑𝑥

Exercise
1) Solve the following differential equation
𝟏
i) (𝑥 4 𝐷4 + 6𝑥 3 𝐷3 + 15𝑥 2 𝐷2 + 9𝑥𝐷)𝑦 =
𝒙𝟐
ii) (𝑥 3 𝐷3 + 4𝑥 2 𝐷2 − 5𝑥𝐷 − 15)𝑦 = 𝑥 4
iii) (𝑥 3 𝐷3 + 6𝑥 2 𝐷2 + 6𝑥𝐷)𝑦 = 𝑠𝑖𝑛⁡(𝑙𝑛(𝑥))

2) Find the orthogonal trajectories of the curves;


i) 𝑦 2 = 𝐶𝑥 3
ii) 𝑥 2 + 𝑦 2 = 𝑎𝑥

3) Solve the following systems;


𝑑
i) 𝐷𝑦 = 𝑦 + 𝑧 + 𝑥, 𝐷𝑧 = −4𝑦 − 3𝑧 + 2𝑥; ⁡⁡𝐷 =
𝑑𝑥
𝑑𝑥 𝑑𝑦
ii) + 5𝑥 + 𝑦 = 𝑒 𝑡 , − 𝑥 − 3𝑦 = 𝑒 2𝑡 ; 𝑎𝑟𝑔𝑢𝑚𝑒𝑛𝑡⁡𝑡
𝑑𝑡 𝑑𝑡
iii) (𝐷2 − 2)𝑥 − 3𝑦 = 𝑒 ,⁡⁡⁡⁡⁡⁡⁡(𝐷2 + 2)𝑦 + 𝑥 = 0
2𝑡

78
Find the total solution of the following non-homogenous differential equation by the
linear differential operator method
Example 1 𝑦 ′′ + 2𝑦 ′ + 5𝑦 = 𝑒 −𝑥 sin(𝑥)
(𝐷 2 + 2𝐷 + 5)𝑦 = 𝑒 −𝑥 sin(𝑥)
To find the complimentary function (𝑦𝑐 )

𝑚2 + 2𝑚 + 5 = 0

−2 ± √4 − 20
𝑚= = −1 ± 2𝑖
2
𝑦𝑐 = 𝑒−𝑥 (𝐶1 cos(2𝑥) + 𝐶2 sin(2𝑥))

To find the particular solution (𝑦𝑝 )


𝑒−𝑥 sin(𝑥) −𝑥 sin(𝑥) −𝑥 sin 𝑥
( )
𝑦𝑝 = = 𝑒 = 𝑒
𝐷2 + 2𝐷 + 5 (𝐷 − 1)2 + 2(𝐷 − 1) + 5 𝐷2 + 4
sin(𝑥) 𝑒−𝑥 sin(𝑥)
𝑦𝑝 = 𝑒−𝑥 =
−1 + 4 3
𝑒−𝑥 sin(𝑥)
𝑦𝑡 = 𝑦𝑐 + 𝑦𝑝 = 𝑒−𝑥 (𝐶1 cos(2𝑥) + 𝐶2 sin(2𝑥)) +
3
sin(𝑥)
𝑦𝑡 = 𝑒−𝑥 (𝐶1 cos(2𝑥) + 𝐶2 sin(2𝑥) + )
3
Example 2
𝑦 ′′′ − 16𝑦 ′ = 𝑥 2 + 5⁡cos⁡(2𝑥)
(𝐷 3 − 16𝐷)𝑦 = 𝑥 2 + 5⁡cos⁡(2𝑥)
To find the complimentary function (𝑦𝑐 )

𝑚3 − 16𝑚 = 0
𝑚(𝑚2 − 16) = 0
𝑚 = 0, 4, −4

𝑦𝑐 = 𝐶1 + 𝐶2 𝑒4𝑥 + 𝐶3 𝑒−4𝑥

To find the particular solution (𝑦𝑝 )

𝑥2 + 5⁡cos⁡(2𝑥) 𝑥2 5⁡cos⁡(2𝑥)
𝑦𝑝 = 3
= 3
+ 3
𝐷 − 16𝐷 𝐷 − 16𝐷 𝐷 − 16𝐷
𝑥2 5 cos(2𝑥)
𝑦𝑝 = +
2 −4𝐷 − 16𝐷
−16𝐷(1 − 𝐷 )
16

79
2 −1
1 𝐷 1
𝑦𝑝 = 𝐷 −1 (1 − ) 𝑥2 − 𝐷 −1 cos(2𝑥)
−16 16 4
2
1 𝐷 1 sin⁡(2𝑥)
𝑦𝑝 = 𝐷 −1 (1 + ) ⁡𝑥2 −
−16 16 4 2
1 1 1
𝑦𝑝 = 𝐷 −1 (𝑥2 + ) ⁡− sin(2𝑥)
−16 8 8
1 𝑥3 𝑥 1
𝑦𝑝 = ( + ) ⁡− sin(2𝑥)
−16 3 8 8

1 𝑥3 𝑥 1
𝑦𝑡 = 𝑦𝑐 + 𝑦𝑝 = 𝐶1 + 𝐶2 𝑒4𝑥 + 𝐶3 𝑒−4𝑥 − ( + ) ⁡− sin(2𝑥)
16 3 8 8

Find the solution of the following non-homogenous differential equation system by the
linear differential operator method
𝑑𝑥 𝑑𝑦
⁡ ⁡ − 7𝑥⁡ + ⁡𝑦⁡ = 𝑒 𝑡 ,⁡⁡⁡⁡⁡⁡⁡⁡⁡ ⁡ − ⁡2𝑥 − ⁡5𝑦⁡ = ⁡𝑡
𝑑𝑡 𝑑𝑡
The system can be rewritten as
(𝐷 − 7)𝑥 + 𝑦 = 𝑒 𝑡
−2𝑥⁡ + ⁡ (𝐷 − 5)𝑦⁡ = ⁡𝑡
we now eliminate 𝑥 (say)as follows: multiplying (1) by 2 and operating (2) by (𝐷 − 7), we
get

2(𝐷⁡ − ⁡7)𝑥⁡ + ⁡2𝑦⁡ = ⁡2𝑒 𝑡


−2(𝐷⁡ − ⁡7)𝑥⁡ + ⁡ (𝐷 − ⁡7)⁡(𝐷⁡ − 5)𝑦⁡ = ⁡1 − 7𝑡
adding, we get
[(𝐷⁡ − ⁡7)(𝐷⁡ − ⁡5) + 2]𝑦⁡ = ⁡1 − 7𝑡 + 2 𝑒 𝑡

[⁡𝐷2 − ⁡12𝐷⁡ + 37]𝑦⁡ = ⁡1 − 7𝑡 + 2 𝑒 𝑡


Auxiliary equation is

[𝑚2 ⁡– ⁡12𝑚⁡ + 37] ⁡ = ⁡⁡0⁡⁡⁡⁡⁡ => ⁡⁡⁡⁡⁡𝑚⁡ = ⁡6⁡ ± 𝑖


80
𝑦𝑐 = 𝑒 6𝑡 (𝑐1 𝑐𝑜𝑠𝑡 + 𝑐2 𝑠𝑖𝑛𝑡)
1 7𝑡 2𝑒 𝑡
𝑦𝑝 = − +
D2 - 12D +37 D2 - 12D +37 D2 - 12D +37

1 7 D2 - 12D −1 2𝑒 𝑡
𝑦𝑝 = − (1 + ) 𝑡 +
37 37 37 26

1 7 D2 - 12D −1 2𝑒 𝑡
𝑦𝑝 = − (1 + ) 𝑡 +
37 37 37 26

6𝑡 (
1 7 12 2𝑒 𝑡
y = 𝑦𝑐 + 𝑦𝑝 = 𝑒 𝑐1 𝑐𝑜𝑠𝑡 + 𝑐2 𝑠𝑖𝑛𝑡) + − (𝑡 + ) +
37 37 37 26

6𝑡 (
1 7 12 𝑒𝑡
y=𝑒 𝑐1 𝑐𝑜𝑠𝑡 + 𝑐2 𝑠𝑖𝑛𝑡) + − (𝑡 + ) +
37 37 37 13

81
Chapter Two
Laplace Transform

Objectives
This chapter is dedicated to assist the students to understand and to learn the
fundamental topics of Laplace transforms and its applications. These topics include
(definition, L.T. of basic functions, fundamental properties, inverse L.T., solution of linear
DEs. of constant coefficients with boundary or initial conditions). The main objectives of
this chapter are listed below to learn the students how to:
i) use and apply L.T. in many different engineering fields and applied sciences such as:
automatic control, electromagnetic fields, system analysis, and signal, structure
analysis, fluid mechanics, solid mechanics, mechanics of materials, and many other
fields and miscellaneous applications.
ii) solve linear differential equations with its associated conditions.
iii) relate L.T. with Fourier transform.
iv) use L.T. in many other engineering applications such as: automatic control,
electromagnetic fields, systems analysis, signals analysis, structure analysis, fluid
mechanics, solid mechanics, mechanics of materials, and many other applications.

Introduction
Laplace transform is dedicated to the continuous time function (signal) defined for
specific time interval 𝑡 ≥ 0. It is named in honor of the great French mathematician
«Pierre Simon De Laplace» (1749-1827). L.T. is represented by an infinite integral which
transforms the time domain t0 into the complex frequency domain 𝑠 > 0. Some basic
time functions are transferred form time domain into frequency domain, also the inverse
transformations of these functions are included. Some properties, rules, theorems, and
applications based on L.T. are discussed and reported.

Laplace transforms are widely used in many Engineering and physical applications. The
relations of these applications are expressed in the form of linear differential equations
associated with its initial conditions. The differential equations can be solved by using L.T.
and that by converting it form differential equations into algebraic equations. Laplace
transforms are related with Fourier transforms which has great importance in numerous
applications in Engineering and scientific fields. These fields include; automatic control,

82
theory of electromagnetic fields, systems analysis, signals analysis, structure analysis, fluid
mechanics, mechanics of materials, and many other fields and miscellaneous applications.

1. Laplace Transforms
Considering the signal (function) 𝑓 defined for 𝑡⁡ ≥ ⁡0. Laplace transform of this signal is
function 𝐹 = 𝐿{𝑓} is denoted by 𝐹(𝑠) = 𝐿{(𝑓)}

𝐹(𝑠) = ∫ 𝑓(𝑡)𝑒 −𝑠𝑡 𝑑𝑡


0

𝑭 is a complex-valued function of complex variables, 𝒔 is called the (complex) frequency


variable with units 𝑠𝑒𝑐 −1 ; 𝑡 is called the time variable (𝑖𝑛⁡𝑠𝑒𝑐); (𝒔⁡𝒕) is unitless.
Note that this definition involves integration of a product so it will involve frequent use of
integration by parts for a reminder of the formula and of the definition of an infinite
integral. This immediately raises the question of why to use such a procedure. In fact, the
reason is strongly motivated by real engineering problems. There, typically we encounter
models for the dynamics of phenomena which depend on rates of change of functions, e.g.
velocities and accelerations of particles or points on rigid bodies, which prompts the use
of ordinary differential equations (𝑂𝐷𝐸𝑠). We can use ordinary calculus to solve 𝑂𝐷𝐸𝑠,
provided that they are nicely behaved-which functions means continuous and with
continuous derivatives. Unfortunately, there is much interest in engineering dynamical
problems involving functions that input step change or spike impulses to systems-playing
pool is one example. Now, there is an easy way to smooth out discontinuities in functions
of time: simply take an average value over all time. But an ordinary average will replace
the function by a constant, so use a kind of moving average which takes continuo averages
over all possible intervals of 𝒕. This very neatly deals with the discontinuities by encoding
them as a smooth function of interval length 𝒔.
The amazing thing about using Laplace Transforms is that we can convert a whole 𝑂𝐷𝐸
initial value problem into a Laplace transformed version as functions of s, simplify the
algebra, find the transformed solution 𝑭(𝒔), then undo the transform to get back to the
required solution f as a function of 𝒕.
Interestingly, it turns out that the transform of a derivative of a function is a simple
combination of the transform of the function and its initial value. So, a calculus problem is
converted into an algebraic
83
problem involving polynomial functions, which is easier. There is one further point of great
importance: calculus operations of differentiation and integration are linear. So, the
Laplace Transform of a sum of functions is the sum of their Laplace Transforms and
multiplication of a function by a constant can be done before or after taking its transform.

2. Sufficient conditions for existence of Laplace Transforms


In order to be able to state sufficient condition on 𝑓(𝑡) under which we can guarantee
the existence of 𝐿(𝑓(𝑡)), we introduce the concepts of piecewise continuity and
exponential order as follows.

1. Piecewise continuity
A function 𝑓(𝑡) is said to be piecewise continuous in an interval if and only if:
i. The interval of the function can be divided into a finite number of subintervals
in each of which 𝑓(𝑡) is continuous.
ii. The limits of 𝑓(𝑡) as 𝒕⁡approaches the endpoints of each subinterval are finite.
Another way of stating this is to say that a piecewise continuous function is one that has
only a finite number of finite discontinuities. An example of a piecewise continuous
function is shown in the figure.

2. Exponential order
A function 𝑓(𝑡) is said to be of exponential order for 𝑡 > 𝑇 if we can find constants 𝑀 and
𝛼 such that |𝑓(𝑡)| ⁡ ≤ 𝑀𝑒 𝑎𝑡 for 𝑡⁡ > ⁡𝑇.

84
Theorem 1
If 𝑓(𝑡) is piecewise continuous in every finite interval 0 ≤ 𝑡⁡ ≤ 𝑇 and is of exponential
order for 𝑡⁡ > 𝑇, then 𝐿{𝑓(𝑡)}; exists for 𝑠⁡ > ⁡𝑎 .

Proof
It should be emphasized that these conditions are only sufficient (and not necessary), i.e.
if the conditions are not satisfied 𝐿(𝑓(𝑡)) may be exist. For example, 𝐿{𝑡 −1/2 } exists even
though 𝑡 −1/2 is not piecewise continuous in
0⁡ ≤ 𝑡⁡ ≤ 𝑇
∞ 𝑇 ∞

𝐹⁡(𝑠) = ⁡𝐿(𝑓(𝑡)) = ∫ 𝑓(𝑡)𝑒 −𝑠𝑡 = ∫ 𝑓(𝑡)𝑒 −𝑠𝑡 𝑑𝑡 + ⁡ ∫ 𝑓(𝑡) 𝑒 −𝑠𝑡 𝑑𝑡


0 0 𝑇

Now since 𝑓(𝑡) is a piecewise continuous for 0⁡ ≤ 𝑡⁡ ≤ 𝑇 so also is 𝑓(𝑡)𝑒 −𝑠𝑡 , and thus the
first integral on the right side exists. To show that the second integral on the right also
exists, one can use the fact that |𝑓(𝑡)| ⁡ ≤ 𝑀𝑒 𝑎𝑡 so that
∞ ∞

|∫ 𝑓(𝑡)𝑒 −𝑠𝑡 𝑑𝑡| ⁡ ≤ ⁡ ∫ ⌈𝑓(𝑡)⌉𝑒 −𝑠𝑡 𝑑𝑡


𝑇 𝑇

≤ ⁡ ∫ 𝑀⁡𝑒 𝑎𝑡 𝑒 −𝑠𝑡 𝑑𝑡
𝑇

𝑀
≤ 𝑀 ∫ 𝑒 −(𝑠−𝑎)𝑡 𝑒 −𝑠𝑡 𝑑𝑡 =
𝑠−𝑎
0

For 𝑠⁡ > ⁡𝛼 and the required result is proved

Theorem 2
If 𝑓(𝑡) satisfies the conditions of the theorem 1, then
lim 𝐿{𝑓(𝑡)} =⁡ lim 𝐿{𝑓(𝑠)} = 0⁡
𝑠→∞ 𝑠→∞

It follows that if⁡ lim 𝐹(𝑠)⁡ ≠ 0, then 𝑓(𝑡) cannot satisfy the conditions of the theorem 1.
𝑠→∞

Proof
We have as in the theorem 1,

85
𝑇 ∞

|𝐹(𝑠)| = ⁡ |𝐿{𝑓(𝑡)}| ≤ ⁡ ∫|𝑓(𝑡)|𝑒 −𝑠𝑡 𝑑𝑡 + ⁡ ∫ |𝑓(𝑡)|𝑒 −𝑠𝑡 𝑑𝑡


0 𝑇

Now since 𝑓(𝑡) is piecewise continuous for 0⁡ ≤ 𝑡⁡ ≤ 𝑇, it is bounded, i.e. |𝑓(𝑡)| ≤ 𝐾 for
some constant 𝐾. Using this and the result in the theorem 1, we have
𝑇 ∞
𝑀 𝑀 𝐾+𝑀
|𝐹(𝑠)| ≤ ⁡ ∫ 𝐾⁡𝑒 −𝑠𝑡 𝑑𝑡 + ⁡ ≤ ⁡⁡ ∫ 𝐾⁡𝑒 −𝑠𝑡 ⁡𝑑𝑡 + ⁡ =
𝑠−𝑎 𝑠 − 𝑎 𝑠 − 𝑎⁡
0 0

Taking the limit as 𝑠⁡ > ⁡∞, it follows that lim 𝐹(𝑠) = 0⁡as required.
𝑠→∞

3. Laplace Transform of Some Functions


i. The Unit Step Function 𝒖(𝒕 − 𝒂)
The unit step function, also called Heaviside's
unit step function, is defined as
0⁡⁡⁡⁡⁡⁡⁡⁡𝑡 < 𝑎
𝑢(𝑡 − 𝑎) = {
1⁡⁡⁡⁡⁡⁡⁡⁡𝑡 > 𝑎
and is shown graphically in the figure

We can show that the Laplace transform of the unit step function is calculated using the
definition as follows:
𝑎 ∞

𝐿{𝑢(𝑡 − 𝑎)} = ⁡ ∫(0)𝑒 −𝑠𝑡 𝑑𝑡 + ⁡ ∫ (1)𝑒 −𝑠𝑡 𝑑𝑡


0 𝑎

𝑒 −𝑠𝑡 𝑒 −𝑎𝑠
=0+ | =⁡ ⁡⁡⁡⁡⁡𝑖𝑓⁡𝑠 > 0
−𝑠 𝑠

86
Special Case of Unit Step Function
If 𝑎⁡ = 0 then the unit step function becomes as
follows
0⁡⁡⁡⁡⁡⁡⁡⁡𝑡 < 0
𝑢(𝑡) = {
1⁡⁡⁡⁡⁡⁡⁡⁡𝑡 > 0
and laplace transform of it becomes

𝐿{𝑢(𝑡)} = ⁡ ∫ (1)𝑒 −𝑠𝑡 𝑑𝑡


0

𝑒 −𝑠𝑡 1
=⁡ = ⁡𝑖𝑓⁡𝑠 > 0
−𝑠 𝑠

ii. The function 𝒇(𝒕) ⁡ = ⁡𝒔𝒊𝒏⁡𝒕


Using the defintion
∞ ∞

𝐿{𝑠𝑖𝑛𝑡} = 𝐹(𝑠) = ⁡ ∫ (𝑠𝑖𝑛𝑡)𝑒 −𝑠𝑡⁡ 𝑑𝑡 = ⁡ |𝑒 −𝑠𝑡 (−𝑐𝑜𝑠𝑡)| − s ∫ (𝑐𝑜𝑠𝑡)𝑒 −𝑠𝑡 𝑑𝑡


0 0


= 1 − |(𝑠𝑖𝑛𝑡)𝑒 −𝑠𝑡 | − 𝑠 2 ∫ (𝑠𝑖𝑛𝑡)𝑒 −𝑠𝑡 𝑑𝑡
0
0

∴ 𝐹(𝑠) = ⁡ ∫ (𝑠𝑖𝑛𝑡)𝑒 −𝑠𝑡 𝑑𝑡


0

∴ 𝐹(𝑠) = 1 − ⁡ 𝑠 2 𝐹(𝑠)
1
∴ 𝐹(𝑠) = ⁡
𝑠2 + 1
iii. The function 𝒇(𝒕) ⁡ = ⁡𝒄𝒐𝒔𝒕
Using the definition by a similar way can show that

𝐿{𝑐𝑜𝑠𝑡⁡} = 𝐹(𝑠) = ⁡ ∫ (cos 𝑡)⁡𝑒 −𝑠𝑡⁡ 𝑑𝑡⁡


0
𝑠
𝐹(𝑠) =
𝑠 2 + 1⁡

87
iv. The fuction 𝒇(𝒕) ⁡ = ⁡ 𝒆𝒂𝒕


𝑎𝑡 } 𝑎𝑡 −𝑠𝑡
𝐿{𝑒 = 𝐹(𝑠) = ⁡ ∫ 𝑒 ⁡𝑒 𝑑𝑡 = ⁡ ∫ 𝑒 −(𝑠−𝑎)𝑡⁡ 𝑑𝑡
0
0

𝑒 −(𝑠−𝑎)𝑡 ∞ 1
=⁡ | =⁡
−(𝑠 − 𝑎) 0 𝑠−𝑎
𝑨𝒍𝒔𝒐⁡, 𝒇(𝒕) = ⁡ 𝒆−𝒂𝒕
∞ ∞

𝐿{𝑒 −𝑎𝑡 } = 𝐹(𝑠) = ⁡ ∫ (𝑒 −𝑎𝑡 )𝑒 −𝑠𝑡 𝑑𝑡 = ⁡ ∫ 𝑒 −(𝑠+𝑎)𝑡 𝑑𝑡


0 0

𝑒 −(𝑠−𝑎)𝑡 ∞ 1
= ⁡⁡ | =⁡
−(𝑠 − 𝑎) 0 𝑠−𝑎

Using the Laplace of these functions , we can evaluate the Laplace Transform for the
hyperbolic functions sinht and cosht as follows:
𝑒 𝑡 − ⁡ 𝑒 −𝑡 1 1 1 1
𝐿{𝑠𝑖𝑛ℎ𝑡} = 𝐿⁡ { }= ( −⁡ ) =⁡ 2
2 2 𝑠−1 𝑠+1 𝑠 −1
𝑒 𝑡 + ⁡ 𝑒 −𝑡 1 1 1 𝑠
𝐿{𝑐𝑜𝑠ℎ𝑡} = 𝐿⁡ { }= ( +⁡ ) =⁡ 2
2 2 𝑠−1 𝑠+1 𝑠 −1

v. The function 𝒇(𝒕) ⁡ = ⁡𝒕


𝐿{𝑡} = 𝐹(𝑠)⁡∫ 𝑒 −𝑠𝑡 ⁡𝑑𝑡


0

𝑡 −𝑠𝑡 1 1
=⁡ ⁡𝑒 + ⁡∫ 𝑒 −𝑠𝑡 ⁡𝑑𝑡 = 2⁡
−𝑠 𝑠 𝑠
0
𝑖𝑛⁡𝑡ℎ𝑒⁡𝑠𝑎𝑚𝑒⁡𝑤𝑎𝑦⁡, 𝑤𝑒⁡𝑐𝑎𝑛⁡𝑝𝑟𝑜𝑣𝑒⁡𝑡ℎ𝑎𝑡 ∶
𝑛!
𝐿⁡{𝑒 𝑡 } = ⁡
𝑠 𝑛+1
𝑖𝑛𝑡𝑒𝑔𝑟𝑎𝑡𝑖𝑛𝑔⁡𝑏𝑦⁡𝑝𝑎𝑟𝑡𝑠⁡𝑔𝑖𝑣𝑒𝑠
𝑏 𝑏

∫ 𝑢(𝑡) 𝑣 ′ (𝑡)𝑑𝑡 = 𝑢(𝑡)𝑣(𝑡) − ⁡⁡ ∫ 𝑣(𝑡)𝑢′ (𝑡)𝑑𝑡⁡


𝑎 𝑎
𝑤𝑖𝑡ℎ⁡𝑢(𝑡) = ⁡ 𝑡 𝑛 ⁡, 𝑣 ′ (𝑡) = ⁡ 𝑒 −𝑠𝑡⁡ , 𝑎 = 0⁡, 𝑏 = ∞
∞ ∞
𝑛 −𝑠𝑡⁡ 𝑛
−𝑒 −𝑠𝑡 ∞ 𝑛 𝑛
𝐹(𝑠) = ∫ 𝑡 𝑒 𝑑𝑡 = 𝑡 ( )| + ∫ 𝑡 𝑛−1 𝑒 −𝑠𝑡 = 𝑙(𝑡 𝑛−1 )
𝑠 0 𝑠 𝑠
0 0
88
Provided 𝑡 𝑛 𝑒 −𝑠𝑡⁡ > ⁡0 if 𝑡⁡→⁡∞, which is true for 𝑅𝑠⁡ > ⁡0 applying the formula recusively
one obtain
𝑛!
𝐹(𝑠) = ⁡
𝑠 𝑛+1
vi. The function 𝒇(𝒂𝒕)

𝐿{𝑓(𝑎𝑡)} = ⁡ ∫ 𝑓(𝑎𝑡)𝑒 −𝑠𝑡 𝑑𝑡


0

Let 𝑎𝑡⁡ = ⁡𝑥 then , 𝑑𝑡⁡ = 1/𝑎⁡⁡𝑑𝑥



1 −
𝑠𝑥 1 𝑠
𝐿{𝑓(𝑥)} = ∫ 𝑓(𝑥)𝑒 𝑎 𝑑𝑥 = 𝐹 ( )
𝑎 𝑎 𝑎
0
𝑓𝑜𝑟⁡𝑒𝑥𝑎𝑚𝑝𝑙𝑒⁡;
1 1 𝑠
𝐿{sin 𝑎𝑡} = ⁡⁡ = ⁡
𝑎 𝑠 2 𝑠 2 + ⁡ 𝑎2
(𝑎) + ⁡⁡1

L.T. of various elementary functions:

Time Domain f(𝒕) Laplace Transform 𝑭(𝒔)

0⁡⁡𝑡 < 𝑎 𝑒 −𝑎𝑠


1 𝑢(𝑡 − 𝑎) = {
1⁡⁡𝑡 > 𝑎 𝑠

2 𝑢(𝑡) 1/𝑠

1
3 sin⁡(𝑡)
𝑠2 + 1
𝑎
4 𝑠𝑖𝑛⁡(𝑎𝑡)
𝑠 2 ⁡ + ⁡ 𝑎2
𝑠
5 cos⁡(𝑡)
𝑠2 + 1
𝑠
6 𝑐𝑜𝑠⁡(𝑎𝑡)
𝑠 2 ⁡ + ⁡ 𝑎2
1
7 𝑒 ±𝑎𝑡⁡
𝑠⁡ ∓ 𝑎

89
𝑎
8 𝑠𝑖𝑛ℎ(𝑎𝑡)
𝑠 2 − ⁡ 𝑎2
𝑠
9 𝑐𝑜𝑠ℎ(𝑎𝑡)
𝑠 2 − ⁡ 𝑎2
𝑛!
10 𝑡𝑛
𝑠 𝑛+1
1 𝑠
11 𝐹(𝑎⁡𝑡) ⁡𝐹 ( )
𝑎 𝑎

4. Some important properties of Laplace Transforms


i. Linearity Property
If a and b are any constants while 𝑓1 (𝑡)⁡𝑎𝑛𝑑⁡𝑓2 (𝑡) are functions with Laplace transforms
𝐹2 (𝑠)⁡𝑎𝑛𝑑⁡𝐹1 (𝑠) respectively, then
𝐿(𝑎𝑓1 (𝑡) ⁡ + ⁡𝑏𝑓2 (𝑡)) ⁡ = ⁡𝑎⁡𝐿(𝑓1 (𝑡) ⁡ + 𝑏𝐿(𝑓2 (𝑡)) ⁡ + ⁡ …
= ⁡𝑎𝐹1 (𝑠) + 𝑏𝐹2 (𝑠) + ⁡ …
The result is easily extended to more than two functions
Example 1
Find Laplace transform of the function
𝒇(𝒕) ⁡ = ⁡⁡𝟒𝒕𝟐 ⁡ − ⁡𝟑𝒄𝒐𝒔⁡𝟐𝒕⁡ + ⁡𝟓𝒆−𝒕
𝐹(𝑠) = ⁡𝐿(𝑓(𝑡)) ⁡ = ⁡𝐿⁡(⁡4𝑡 2 ⁡ − ⁡3𝑐𝑜𝑠⁡2𝑡⁡ + ⁡5𝑒 −𝑡 ⁡)
𝑈𝑠𝑖𝑛𝑔⁡𝑙𝑖𝑛𝑒𝑎𝑟𝑖𝑡𝑦⁡𝑝𝑟𝑜𝑝𝑒𝑟𝑡𝑦⁡𝑡ℎ𝑒𝑛,
(𝑠) ⁡ = ⁡4𝐿(𝑡 2 )⁡– ⁡3𝐿(𝑐𝑜𝑠⁡2𝑡) ⁡ + ⁡5𝐿(𝑒 −𝑡 )
2! 𝑠 1
𝐹(𝑠) = 4( 3 ) − 3 ( 2 )+ 5( )
𝑠 𝑠 +4 𝑠+1

ii. First Shift Property (First Translation)


If 𝐿(𝑓(𝑡)) ⁡ = ⁡𝐹(𝑠) then
𝐿{𝑒 𝑎𝑡 ⁡𝑓(𝑡)} = 𝐹(𝑠 − 𝑎)
To prove that , we have

𝐿{𝑓(𝑡)} = ⁡ ∫ 𝑓(𝑡)𝑒 −𝑠𝑡⁡ 𝑑𝑡 = 𝐹(𝑠)


0
90
∞ ∞
Then 𝐿{𝑒 𝑎𝑡 ⁡𝑓(𝑡)} = ∫0 ( 𝑒 𝑎𝑡 𝑓(𝑡))𝑒 −𝑠𝑡 𝑑𝑡 = ∫0 𝑓(𝑡)𝑒 −(𝑠−𝑎)𝑡 𝑑𝑡 = 𝐹(𝑠 − 𝑎)

Example 2
Find laplace transform of the function 𝒆−𝒕 ⁡𝒄𝒐𝒔⁡𝟐𝒕

𝑓(𝑡) ⁡ = ⁡𝑐𝑜𝑠(2𝑡) and multiplying by 𝑒 −𝑡 ⁡𝑡ℎ𝑒𝑛,


𝑠
𝐹(𝑠) = 2 ⁡⁡⁡⁡⁡⁡⁡⁡⁡⁡⁡𝑎𝑛𝑑⁡⁡⁡⁡⁡⁡⁡⁡𝑎 = ⁡ −1
𝑠 +4
Put 𝑠⁡ = ⁡𝑠⁡– ⁡𝑎⁡⁡⁡⁡ => ⁡⁡𝑠⁡ = ⁡𝑠⁡ + ⁡1 then
𝑠⁡ + 1
𝐿{𝑒 −𝑡 𝑐𝑜𝑠2𝑡} = ⁡
(𝑠 + 1⁡)2 + 4

Example 3
Find laplace transform of the function 𝒆−𝟐𝒕 ⁡𝒕𝟐

𝑓(𝑡) ⁡ = ⁡ 𝑡 2 and multiplying by 𝑒 −2𝑡 ⁡𝑡ℎ𝑒𝑛,


2
𝐹(𝑠) = ⁡⁡⁡⁡⁡⁡⁡⁡⁡⁡⁡𝑎𝑛𝑑⁡⁡⁡⁡⁡⁡⁡⁡𝑎 = ⁡ −2
𝑠3
Put 𝑠⁡ = ⁡𝑠⁡– ⁡𝑎⁡⁡⁡⁡ => ⁡⁡𝑠⁡ = ⁡𝑠⁡ + ⁡2 then
𝑠⁡
𝐿{𝑒 −2𝑡 𝑡 2 } = ⁡
(𝑠 + 1⁡)3
iii. Second Shift Property (Second Translation)
0⁡⁡⁡⁡⁡⁡⁡⁡⁡⁡⁡⁡⁡𝑡 < 𝑎
If 𝐿(𝑓(𝑡)) ⁡ = ⁡𝐹(𝑠) and 𝐺(𝑡) ⁡ = ⁡ { ⁡then
𝑓(𝑡 − 𝑎)⁡⁡⁡𝑡 > 𝑎
𝐿(𝐺(𝑡)) ⁡ = 𝑒 −𝑎𝑠⁡ 𝐹(𝑠)
Note that
0⁡⁡⁡⁡⁡⁡⁡⁡⁡⁡⁡⁡⁡𝑡 < 𝑎 0⁡⁡⁡⁡⁡⁡⁡⁡⁡⁡⁡⁡⁡𝑡 < 𝑎
𝐺(𝑡) ⁡ = { => ⁡⁡𝐺(𝑡)𝑓(𝑡 − 𝑎) {
𝑓(𝑡 − 𝑎)⁡⁡⁡𝑡 > 𝑎 1⁡⁡⁡⁡⁡⁡⁡⁡⁡⁡⁡⁡𝑡 > 𝑎
Then
𝐺(𝑡) ⁡ = ⁡𝑓(𝑡⁡– ⁡𝑎⁡)⁡𝑢(𝑡⁡ − 𝑎⁡)⁡𝑜𝑟⁡𝐺(𝑡) ⁡ = ⁡⁡𝑢(𝑡 − 𝑎)⁡𝑓(𝑡 − 𝑎)

91
Example 4
Find laplace transform of the function
𝟎⁡⁡⁡⁡⁡⁡⁡⁡⁡⁡⁡⁡⁡⁡⁡⁡𝒕 < 𝟐
𝑮(𝒕) ⁡ = {
(𝒕 − 𝟐)𝟑 ⁡⁡⁡𝒕 > 𝟐
𝐺(𝑡) ⁡ = ⁡⁡𝑢(𝑡 − 𝑎)⁡𝑓(𝑡 − 𝑎) ⁡ = ⁡𝑢(⁡𝑡⁡– ⁡2)⁡𝑓(⁡𝑡⁡– ⁡2)
= ⁡𝑢(𝑡⁡– ⁡2⁡)⁡(𝑡⁡– ⁡2⁡)⁡3
3! 6
𝐹(𝑡) ⁡ = 𝑡 3 𝐹(𝑠) ⁡ = =
𝑠 3+1 𝑠4

and 𝑎⁡ = 2
0⁡⁡⁡⁡⁡⁡⁡⁡⁡⁡⁡⁡⁡⁡⁡⁡𝑡 < 2
The laplace transform of the function 𝐺(𝑡) ⁡ = {
(𝑡 − 2)3 ⁡⁡⁡𝑡 > 2

−𝑎𝑠
6⁡𝑒 −2𝑠
𝐿⁡(𝐺(𝑡)) ⁡ = 𝑒 𝐹(𝑠) ⁡ =
𝑠4

Example 5
Find laplace transform of the function
𝟐𝝅
𝟎⁡⁡⁡⁡⁡⁡⁡⁡⁡⁡⁡⁡⁡⁡⁡⁡⁡⁡⁡⁡⁡⁡⁡⁡𝒕 <
𝑮(𝒕) ⁡ = { 𝟑
𝟐𝝅 𝟐𝝅
𝒄𝒐𝒔⁡(𝒕 − )⁡⁡⁡⁡𝒕 >
𝟑 𝟑

𝟐𝝅 𝟐𝝅
𝐺(𝑡) ⁡ = ⁡⁡𝑢(𝑡 − 𝑎)⁡𝑓(𝑡 − 𝑎) ⁡ = ⁡𝑢(⁡𝑡⁡– ⁡⁡)⁡𝑓(⁡𝑡⁡–⁡ )
𝟑 𝟑
𝟐𝝅
= ⁡𝑢(𝑡⁡– ⁡2⁡)⁡𝑐𝑜𝑠(𝑡⁡–⁡ ⁡)
𝟑
𝑠
𝐹(𝑡) ⁡ = ⁡𝑐𝑜𝑠𝑡 𝐹(𝑠) =
𝑠 2 +1
𝟐𝝅
and 𝑎⁡ = ⁡
𝟑
2𝜋
0⁡⁡⁡⁡⁡⁡⁡⁡⁡⁡⁡⁡⁡⁡⁡⁡⁡⁡⁡⁡⁡⁡⁡⁡𝑡 <
3
The laplace transform of the function 𝐺(𝑡) ⁡ = { 2𝜋 2𝜋
𝑐𝑜𝑠⁡(𝑡 − )⁡⁡⁡⁡𝑡 >
3 3
−2𝜋
− 𝑠
𝑠⁡𝑒 3
𝐿⁡(𝐺(𝑡)) ⁡ = 𝑒 −𝑎𝑠 𝐹(𝑠) ⁡ =
𝑠 2 +1

iv. Multiplication by power of 𝒕⁡(⁡𝒕𝒏 𝒇(𝒕))


92
If 𝐿⁡(𝑓(𝑡)) ⁡ = ⁡𝐹(𝑠) then

𝒏
𝑑𝑛 𝑛
𝐿(𝒕 𝒇(𝒕)) ⁡ = (−1) ⁡𝐹(𝑠)⁡𝑤ℎ𝑒𝑟𝑒⁡𝑛 = 1, 2⁡, 3, … …
𝑑𝑠 𝑛 ⁡

Example 6
Find the laplace transform of the function 𝒕𝒔𝒊𝒏(𝒂𝒕)
𝑎
𝐹(𝑡) ⁡ = ⁡𝑠𝑖𝑛(𝑎𝑡) ⁡⁡⁡ => ⁡⁡⁡𝐹(𝑠) ⁡ =
𝑠 2 +⁡𝑎2
𝑑 2𝑎𝑠⁡ 2𝑎𝑠⁡
∴ 𝐿⁡{𝑡 sin(𝑎𝑡)} = (−1) ( 2 ) =
𝑑𝑠 (𝑠 + ⁡ 𝑎2 )2 (𝑠 2 + ⁡ 𝑎2 )2

Eample 7
Find the laplace transform of the function 𝒕𝟐 ⁡𝒄𝒐𝒔(𝒂𝒕)
𝑠
𝐹(𝑡) ⁡ = ⁡𝑐𝑜𝑠(𝑎𝑡) ⁡⁡⁡ => ⁡⁡⁡𝐹(𝑠) ⁡ =
𝑠 2 +⁡𝑎2

2
𝑑2 𝑠 2
2𝑠 3 − 6⁡𝑎2 𝑠⁡
∴ 𝐿⁡{𝑡 cos(𝑎𝑡)} = (−1) ( )=( 2 )
𝑑𝑠 2 𝑠 2 + ⁡ 𝑎2 (𝑠 + ⁡ 𝑎2 )3

Example 8
Find the laplace transform of the function 𝒕𝟐 𝒆𝟐𝒕
1
𝐹(𝑡) ⁡ = ⁡ 𝑒 2𝑡 => ⁡⁡⁡𝐹(𝑠) ⁡ =
𝑠−2
𝟐 𝟐𝒕 }
𝑑2 12
2
∴ 𝐿⁡{𝒕 𝒆 = (−1) ( ) = ⁡
𝑑𝑠 2 𝑠 − 2 (𝑠 − 2⁡)3
Another method
This example can be solved first shift property since
2
𝐹(𝑡) ⁡ = 𝑡 2 => ⁡⁡⁡⁡⁡⁡⁡⁡⁡⁡⁡⁡𝐹(𝑠) ⁡ =
(𝑠 − 2)3
𝒇(𝒕)
v. Division by 𝒕⁡⁡⁡ ( )
𝒕
𝑓(𝑡)
Assume that 𝐿(𝑓(𝑡)) exists and lim exists , then
𝑡−→0 𝑡

93

𝑓(𝑡)
𝐿{ } = ⁡ ∫ 𝐹(𝑠)𝑑𝑠⁡⁡⁡⁡⁡𝑎𝑛𝑑⁡⁡⁡⁡𝐹(𝑠) = 𝐿{𝑓(𝑡)}
𝑡
𝑠

Example 9
𝟏−⁡𝒆−𝒕
Find the laplace transform of the function 𝒇(𝒕) =
𝒕
1 1
𝑓(𝑡) ⁡ = 1 − 𝑒 −𝑡 then 𝐹(𝑠) = 𝐿{𝑓(𝑡)} = −
𝑠 𝑠+1
∞ ∞
𝑓(𝑡) 1 − ⁡ 𝑒 −𝑡 1 1
𝐿{ } = ⁡ ∫ 𝐹(𝑠)𝑑𝑠⁡⁡ => ⁡⁡𝐿 { } =⁡∫ ( −⁡ ) 𝑑𝑠⁡
𝑡 𝑡 𝑠 𝑠+1
𝑠 𝑠

1 − ⁡ 𝑒 −𝑡 ∞
𝐿{ } ⁡ = ⁡ |𝑙𝑛𝑠 − ln⁡(𝑠 + 1)|
𝑡 𝑠
𝑠 ∞ 𝑠
= |𝑙𝑛 | = ⁡0 − 𝑙𝑛
𝑠+1 𝑠 𝑠+1
𝑠+1
= ⁡𝑙𝑛 | |
𝑠
Example 10
𝟏−⁡𝒄𝒐𝒔𝒕
Find the laplace transform of the function 𝒇(𝒕) ⁡ =
𝒕
1 1
𝑓(𝑡) ⁡ = ⁡1⁡ − ⁡𝑐𝑜𝑠𝑡⁡⁡⁡𝑡ℎ𝑒𝑛 𝐹(𝑠) = 𝐿{𝑓(𝑡)} = −
𝑠 𝑠 2 +1
∞ ∞
𝑓(𝑡) 1 − ⁡𝑐𝑜𝑠𝑡 1 1
𝐿{ } = ⁡ ∫ 𝐹(𝑠)𝑑𝑠⁡⁡ => ⁡⁡𝐿 { } =⁡∫ ( −⁡ 2 ) 𝑑𝑠⁡
𝑡 𝑡 𝑠 𝑠 +1
𝑠 𝑠
1 − ⁡𝑐𝑜𝑠𝑡 1 ∞
𝐿{ } = ⁡ |𝑙𝑛𝑠 − ln⁡(𝑠 2 + 1)|
𝑡 2 𝑠
𝑠 ∞ 𝑠
= |𝑙𝑛 | = ⁡0 − 𝑙𝑛
√𝑠 2 + 1 𝑠 √𝑠 2 + 1
√𝑠 2 + 1
= ⁡𝑙𝑛 | |
𝑠

vi. Laplace transform of derivatives


If 𝐿(𝑓(𝑡)) = 𝐹(𝑠)⁡𝑎𝑛𝑑⁡𝑓(𝑡) is continuous function for all 𝑡⁡ ≥ ⁡0, and has an
exponential order 𝛼 and has derivative 𝑓́ (𝑡) which is piecewise continuous on every
finite interval in the range 𝑡⁡ ≥ ⁡0 then in general:

𝐿{𝑓 (𝑛) (𝑡)} = ⁡ 𝑠 𝑛 𝐹(𝑠) − ⁡ 𝑠 𝑛−1 𝑓(0) − ⁡ 𝑠 𝑛−2 𝑓 ′ (0) − ⋯ − 𝑓 (𝑛−1) (0)
94
Special cases :
𝑳{𝒇′ (𝒕)} = 𝒔𝑭(𝒔) − 𝒇(𝟎)
𝑳{𝒇′′ (𝒕)} = ⁡ 𝒔𝟐 𝑭(𝒔) − 𝒔𝒇(𝟎) − 𝒇′ (𝟎)
Example 11
Find laplace transform 𝑳{𝒇′ (𝒕)}⁡(𝒖𝒔𝒊𝒏𝒈⁡𝒕𝒉𝒊𝒔⁡𝒑𝒓𝒐𝒑𝒆𝒓𝒕𝒚) the function
𝒇(𝒕) = ⁡ 𝒕𝟐 + 𝟐𝒕 − 𝟒
𝐿{𝑓 ′ (𝑡)} = 𝑠𝐹(𝑠) − 𝑓(0)
2 2 4
𝐹(𝑠) = + ⁡ −
𝑠3 𝑠2 𝑠
𝐹(0) ⁡ = ⁡ (0)2 ⁡ + ⁡2(0)⁡– ⁡4⁡ = ⁡ −4
2 2 2 2
𝐿{𝑓′(𝑡)} = + − 4 + 4 = +
𝑠2 𝑠 𝑠2 𝑠
another solution without this property 𝑓 ′ (𝑡) = 2𝑡 + 2
2 2
𝐿{𝑓′(𝑡)} = + ⁡𝐴𝑛𝑠⁡⁡⁡𝑡ℎ𝑒⁡𝑠𝑎𝑚𝑒⁡𝑟𝑒𝑠𝑢𝑙𝑡
𝑠2 𝑠
Example 12
Find laplace transform 𝑳{𝒇′ (𝒕)}⁡𝒖𝒔𝒊𝒏𝒈⁡𝒕𝒉𝒊𝒔⁡𝒑𝒓𝒐𝒑𝒆𝒓𝒕𝒚⁡𝒐𝒇 the function
𝒇(𝒕) = ⁡𝒔𝒊𝒏⁡𝒕
𝐿{𝑓 ′ (𝑡)} = 𝑠𝐹(𝑠) − 𝑓(0)
1
𝐹(𝑠) ⁡ = ⁡ 𝑓(𝑡)
𝑠2 + 1
𝐹(0) ⁡ = ⁡𝑠𝑖𝑛⁡0⁡ = ⁡0
1 𝑠
𝐿{𝑓′(𝑡)} = 𝑠 ( ) =
𝑠2 + 1 𝑠2 + 1
another solution without this property 𝑓 ′ (𝑡) = cos 𝑡
𝑠
𝐿{𝑓′(𝑡)} = 2
𝑠 +1

vii. Laplace transform of integrals


𝑓(𝑡) satisfies the sufficient conditions (piecewise continuous and has an exponential
order) for the existence of Laplace transform, then

95
𝑡
𝐹(𝑠)
𝐿 {∫ 𝑓(𝑡)𝑑𝑡} = ⁡⁡⁡⁡⁡𝑤ℎ𝑒𝑟𝑒⁡⁡⁡⁡𝐹(𝑠) = 𝐿{𝑓(𝑡)}
𝑠
0

Example 13
𝒕 𝒔𝒊𝒏𝒕
Find Laplace transform of the function ∫𝟎 𝒅𝒕
𝒕

First we must calculate 𝐹(𝑠) = ⁡⁡𝐿(𝑓(𝑡))⁡𝑤ℎ𝑒𝑟𝑒⁡𝑓(𝑡) ⁡ = ⁡𝑠𝑖𝑛𝑡/𝑡


To calculate 𝐹(𝑠) ⁡ = ⁡𝐿(𝑓(𝑡)) we must apply the property of dividing by t as the following
1
𝑔(𝑡) = ⁡𝑠𝑖𝑛𝑡 then 𝐺(𝑠) =
𝑠 2 ⁡+⁡⁡⁡1
∞ ∞
𝑔(𝑡) 𝑠𝑖𝑛𝑡 1
𝐿{ ⁡} = ⁡ ∫ 𝐺(𝑠)𝑑𝑠⁡⁡ => ⁡⁡𝐿 { }=∫( 2 ) 𝑑𝑠
𝑡 𝑡 𝑠 +1
𝑠 𝑠

𝑠𝑖𝑛𝑡 ∞
𝐿{ } = ⁡ |tan−1 𝑠|
𝑡 𝑠
𝜋
𝐹(𝑠) = − ⁡ tan−1 𝑠
2

𝐹(𝑠) = cot −1 𝑠

𝑡
𝑠𝑖𝑛𝑡 𝐹(𝑠) cot −1 𝑠
𝐿 {∫ ⁡𝑑𝑡} = =⁡
𝑡 𝑠 𝑠
0

viii. Laplace transform of periodic functions


Consider a function 𝑓(𝑡) which is defined for all value, of t > 0 and has period 𝑝⁡ > ⁡0
i.e. 𝑓(𝑡) = ⁡𝑓⁡(𝑡⁡ + ⁡𝑝) = ⁡𝑓⁡(𝑡⁡ + ⁡2𝑝)...
then its laplace transform exists and we can write it as
𝑝
∫0 𝑓(𝑡). 𝑒 −𝑠𝑡 ⁡𝑑𝑡
𝐹(𝑠) = 𝑙{𝑓(𝑡)} = ⁡
1 − ⁡⁡ 𝑒 −𝑠𝑝
Proof
∞ 𝒑 𝟐𝒑

𝑳{𝒇(𝒕)}⁡∫ 𝒇(𝒕). 𝒆−𝒔𝒕 𝒅𝒕 = ⁡ ∫ 𝒇(𝒕)⁡. 𝒆−𝒔𝒕 ⁡𝒅𝒕 + ⁡ ∫ 𝒇(𝒕)⁡. 𝒆−𝒔𝒕 ⁡𝒅𝒕 + ⋯ .


𝟎 𝟎 𝒑
96
∴ 𝒇(𝒕) = 𝒇⁡(𝒕 + 𝒑) = 𝒇⁡(𝒕 + 𝟐𝒑) = ⋯ … …

Put 𝑡⁡ = ⁡𝑢⁡ + ⁡𝑝 in the second integral and


Put 𝑡⁡ = ⁡𝑢⁡ + ⁡2𝑝 in the third integral and so on , we get
𝒑 𝒑 𝟐𝒑

𝑳{𝒇(𝒕)} = ∫ 𝒇(𝒕). 𝒆−𝒔𝒕 𝒅𝒕 = ⁡ ∫ 𝒇(𝒖 + 𝒑)⁡. 𝒆−𝒔(𝒖+𝒑) ⁡𝒅𝒖 + ⁡ ∫ 𝒇(𝒕)⁡. 𝒆−𝒔𝒕 ⁡𝒅𝒕 + ⋯ .
𝟎 𝟎 𝒑
𝒑 𝒑

= ∫ 𝒇(𝒕). 𝒆−𝒔𝒕 𝒅𝒕 + ⁡ 𝒆−𝒔𝒑 ⁡∫ 𝒇(𝒕)⁡. 𝒆−𝒔(𝒕) ⁡𝒅𝒕 + ⋯ .


𝟎 𝟎

where 𝒇(𝒕) ⁡ = ⁡𝒇(𝒕⁡ + ⁡𝒑) ⁡ = ⁡𝒇(𝒕⁡ + ⁡𝟐𝒑) = ⁡ … ..


𝒑 𝒑
−𝒔𝒑⁡ −𝒔𝒑⁡
∫𝟎 𝒇(𝒕). 𝒆−𝒔𝒑⁡ 𝒅𝒕⁡
𝑳{𝒇(𝒕)} = (⁡𝟏 + ⁡ 𝒆 + ⋯ ) ∫ 𝒇(𝒕). 𝒆 𝒅𝒕 = ⁡
𝟏 − ⁡ 𝒆−𝒔𝒑
𝟎

Example 14
Find Laplace transform of periodic function 𝒇(𝒕) ⁡ =
𝒕, 𝟎⁡ < ⁡𝒕 < ⁡𝟏

The function is periodic with period 𝑝⁡ = 1


𝒑 1
∫𝟎 𝒇(𝒕). 𝒆−𝒔𝒑⁡ 𝒅𝒕⁡ ∫0 𝑡⁡𝑒 −𝑠𝑡 𝑑𝑡
𝐿{𝑓(𝑡)} = =
𝟏 − ⁡ 𝒆−𝒔𝒑 1 − ⁡ 𝑒 −𝑠𝑝
1 𝑡⁡𝑒 −𝑠𝑡 1 𝑒 −𝑠𝑡 1
= ⁡(| | −⁡| 2 | )
1 − ⁡ 𝑒 −𝑠𝑝 −𝑠 0 𝑠 0
1 𝑒 −𝑠 𝑒 −𝑠 1 1 − ⁡ 𝑒 −𝑠 (1 + 𝑠)
= ⁡(− − ⁡ 2 + ⁡ 2) = ⁡ 2
1 − ⁡ 𝑒 −𝑠𝑝 𝑠 𝑠 𝑠 𝑠 (1 − ⁡ 𝑒 −𝑠𝑝 )
Example 15
Find Laplace transform of the periodic function 𝒇(𝒕) = 𝒆𝒕 ,𝟎⁡ < ⁡𝒕 < ⁡𝟐𝝅
The function is periodic with period 𝑝⁡ = ⁡2𝜋

97
𝒑 2𝜋
∫𝟎 𝒇(𝒕). 𝒆−𝒔𝒑⁡ 𝒅𝒕⁡ ∫0 ⁡𝑒 𝑡 𝑒 −𝑠𝑡 𝑑𝑡
𝐿{𝑓(𝑡)} = =
𝟏 − ⁡ 𝒆−𝟐𝝅𝒔 1 − ⁡ 𝑒 −2𝜋𝑠
2𝜋
1 𝑡(1−𝑠)
1 𝑒 𝑡(1−𝑠) 2𝜋 1 𝑒 2𝜋(1−𝑠) − 1
= ⁡(∫ ⁡𝑒 𝑑𝑡 = ⁡| | = ( )
1 − ⁡ 𝑒 −2𝜋𝑠 1 − ⁡ 𝑒 −2𝜋𝑠 1 − 𝑠 0 1 − ⁡ 𝑒 −2𝜋𝑠 1−𝑠
0

The Inverse of Laplace Transform

If the Laplace transform of a function 𝑓(𝑡)⁡𝑖𝑠⁡𝐹(𝑠)⁡𝑖. 𝑒. , 𝑖𝑡. 𝐿(𝑓(𝑡)) − 𝐹(𝑠),⁡then 𝑓(𝑡) is


called an inverse Laplace Transform of 𝐹(𝑠) and we write symbolically 𝑓(𝑡) =
⁡𝐿−1 (𝐹(𝑠))𝑤ℎ𝑒𝑟𝑒⁡𝐿−1 is called the inverse Laplace transformation operator.
For existence of the inverse Laplace transformation we must obtain lim 𝐹(𝑠) = 0, this
𝑠−→⁡∞
means that all the following theorems can be used to obtain the function 𝑓(𝑡) and this will
be clear from the following cases:
Example
𝟏
Since 𝑳{𝒆−𝟓𝒕 } = ⁡
𝒔+𝟓

we can write
𝟏
𝒇(𝒕) ⁡ = 𝑳−𝟏 { } = ⁡ 𝒆−𝟓𝒕
𝒔+𝟓

98
i. Functions which takes the standard form
Example 16
𝟓
Find the inverse Laplace transform of the function 𝑭(𝒔) = ⁡
𝟑𝒔−𝟔

First we must test the limit of 𝐹(𝑠) as the following


5
lim 𝐹(𝑠) = ⁡ lim = 0 then the inverse L.T. exists.
𝑠→∞ 𝑠→⁡∞ 3𝑠−6

To compute the inverse Laplace transform the coefficient of ′𝒔′ must be equal one (𝑠⁡ =
⁡1), then
𝟓 𝟓 𝟏
𝑭(𝒔) = = ( )
𝟑𝒔 − 𝟔 𝟑 𝒔 − 𝟐
5 1 5
𝑓(𝑡) = ⁡ 𝐿−1 {𝐹(𝑠)} = ⁡ 𝐿−1 { ⁡( )} = ⁡𝑒 2𝑡
3 𝑠−2 3
Example 17
Find the inverse Laplace transform of the function
𝟐𝒔 − 𝟔
𝑭(𝒔) = ⁡
𝒔𝟐 − 𝟗
First we must test the limit of 𝐹(𝑠) as the following
2𝑠−6
lim 𝐹(𝑠) = ⁡ lim = 0 then the inverse Laplace transform exists
𝑠→∞ 𝑠→⁡∞ 𝑠 2 −9

2𝑠 − 6 𝒔 𝟑
𝐹(𝑠) ⁡ = = 𝟐 ( ) − 𝟐
𝑠2 − 9 𝒔𝟐 − 𝟗 𝒔𝟐 − 𝟗
𝑠 3
𝑓(𝑡) = ⁡ 𝐿−1 {𝐹(𝑠)} = ⁡ 𝐿−1 {2 2 −2 2 } = 2𝑐𝑜𝑠ℎ3𝑡 − 2𝑠𝑖𝑛ℎ3𝑡
𝑠 −9 𝑠 −9
Example 18
Find the inverse Laplace transform of the function
𝟏 𝟑
𝑭(𝒔) = − ⁡
𝒔𝟑 𝒔𝟐 + 𝟓
First we must test the limit of 𝐹(𝑠) as follows
𝟏 𝟑
lim 𝐹(𝑠) = ⁡ lim −⁡ ⁡ = 0 then the inverse Laplace transform exists
𝑠→∞ 𝑠→⁡∞ 𝒔𝟑 𝒔𝟐 +𝟓

𝟏 𝟑 𝟏 𝟐! 𝟑 √𝟓
𝐹(𝑠) ⁡ = − ⁡ = −
𝒔𝟑 𝒔𝟐 + 𝟓 𝟐 𝒔𝟑 √𝟓 𝒔𝟐 + 𝟓
99
1 2! 3 √5 1 2 3
𝑓(𝑡) = ⁡ 𝐿−1 {𝐹(𝑠)} = ⁡ 𝐿−1 { 3 − } = 𝑡 + 𝑠𝑖𝑛√5𝑡
2𝑠 √5 𝑠 2 + 5 2 √5

ii. Linearity Property


if 𝐶1 ⁡⁡𝑎𝑛𝑑⁡𝐶2 are any constants while 𝐹1 (𝑠)⁡𝑎𝑛𝑑⁡𝐹2 (𝑠) are the laplace transform of
𝑓1 (𝑡)⁡𝑎𝑛𝑑⁡𝑓2 (𝑠) respectively , then ;
𝐿−1⁡ {𝐶1 𝐹1 (𝑠) + ⁡ 𝐶2 𝐹2 (𝑠)} = ⁡ 𝐶1 𝐿−1 {𝐹1 (𝑠)} + ⁡ 𝐶2 𝐿−1 {𝐹2 (𝑠)}
= ⁡ 𝐶1 𝐹1 (𝑡) + ⁡ 𝐶2 𝐹2 (𝑡)
The result is easily extended to more than two functions.
Example 19
Find the inverse laplace of the function
𝟒 𝟑𝒔 𝟓
𝑭(𝒔) ⁡ = ⁡ −⁡ 𝟐 + 𝟐
𝒔 − 𝟐 𝒔 + 𝟏𝟔 𝒔 + 𝟒
First we must test the limit of 𝐹(𝑠) as the following
𝟒 𝟑𝒔 𝟓
lim 𝐹(𝑠) = lim −⁡ + ⁡⁡then the inverse Laplace transform exists.
𝑠→∞ 𝑠→∞ 𝒔−𝟐 𝒔𝟐 +𝟏𝟔 𝒔𝟐 +𝟒
Using the previous case to calculate the inverse Laplace transform as the following
4 3𝑠 5
𝑓(𝑡) = ⁡ 𝐿−1 {𝐹(𝑠)} = ⁡ 𝐿−1 { −⁡ 2 + 2 }
𝑠 − 2 𝑠 + 16 𝑠 + 4
4 𝑠 1
= 4𝐿−1 { } − 3𝐿−1 { 2 } + 5𝐿−1 { 2 }
𝑠−2 𝑠 +4 𝑠 +4
5
= 4𝑒 2𝑡 − 3𝑐𝑜𝑠4𝑡 + 𝑠𝑖𝑛2𝑡
2
Because of this property we can say that 𝐿−1 is a linear operator or that it has the linearity
property.

iii. Functions which take the form of the First Shift Property
If 𝐿−1 {𝐹(𝑠)} = 𝑓(𝑡), then 𝐿−1 {𝐹(𝑠 − 𝑎)} = 𝑒 𝑎𝑡 𝑓(𝑡)
In this case, the denominator of 𝐹(𝑠) is of 2nd degree but can not be decomposed.

100
Example 20
Find inverse Laplace of the function
𝟏
𝑭(𝒔) ⁡ = ⁡
𝒔𝟐 − 𝟐𝒔 + 𝟓
First we must test the limit of 𝐹(𝑠) as the following
𝟏
lim 𝐹(𝑠) = lim = 0, then the inverse Laplace transform exists.
𝑠→∞ 𝑠→∞ 𝒔𝟐 −𝟐𝒔+𝟓

1
𝑓(𝑡) = ⁡ 𝐿−1 {𝐹(𝑠)} = ⁡ 𝐿−1 { 2 }
𝑠 − 2𝑠 + 5
1 1 𝑡 −1 2 1 𝑡
= 𝐿−1 { } = ⁡𝑒 𝐿 { } = 𝑒 𝑠𝑖𝑛2𝑡
(𝑠 − 1)2 + 4 2 𝑠2 + 4 2
Example 21
Find inverse Laplace of the function
𝟏
𝑭(𝒔) ⁡ =
𝒔𝟐 + 𝟐𝒔 + 𝟓
First we must test the limit of 𝐹(𝑠) as the following
𝟏
lim 𝐹(𝑠) = lim = 𝟎⁡ then the inverse Laplace transform exists.
𝑠→∞ 𝑠→∞ 𝒔𝟐 +𝟐𝒔+𝟓

1
𝑓(𝑡) = ⁡ 𝐿−1 {𝐹(𝑠)} = ⁡ 𝐿−1 { 2 }
𝑠 + 2𝑠 + 5
1 1 −𝑡 −1 2 1 −𝑡
= 𝐿−1 { } = ⁡𝑒 𝐿 { } = 𝑒 𝑠𝑖𝑛2𝑡
(𝑠 + 1)2 + 4 2 𝑠2 + 4 2

Example 22
Find inverse Laplace of the function
𝒔−𝟑
𝑭(𝒔) ⁡ =
𝒔𝟐 − 𝟒𝒔 − 𝟒
First we must test the limit of 𝐹(𝑠) as the following
𝑠−3
lim 𝐹(𝑠) = lim = 0⁡, then the inverse Laplace transform exists.
𝑠→∞ 𝑠→∞ 𝑠 2 −4𝑠−4

By using partial fractions:

101
√8 − 1 1 + ⁡ √8
𝑎 = 2 + √8⁡⁡,⁡⁡⁡⁡⁡𝑏 = 2 − ⁡ √8, 𝐴 = ⁡⁡, 𝐵 =⁡
4√2⁡ 4√2
𝑠−3
𝑓(𝑡) = ⁡ 𝐿−1 {𝐹(𝑠)} = ⁡ 𝐿−1 { 2 }
𝑠 − 4𝑠 − 4
(𝑠 − 2) − 1 (𝑠 − 2) 1
= 𝐿−1 { } = 𝐿−1
{ − ⁡ }
(𝑠 − 1)2 − 8 (𝑠 − 2)2 − 8 (𝑠 − 2)2 − 8

𝑠 1 √8
= 𝑒 2𝑡 { 2 − }
𝑠 − 8⁡ √8 𝑠 2 − 8
1
𝑓(𝑡) = ⁡ 𝑒 2𝑡 (𝑐𝑜𝑠ℎ√8⁡𝑡 − 𝑠𝑖𝑛ℎ√8⁡𝑡)
√8

iv. Functions which take the form of the Second Shift Property
If 𝐿−1 {𝐹(𝑠)} = 𝑓(𝑡), 𝑡ℎ𝑒𝑛
0⁡⁡⁡⁡⁡⁡⁡⁡⁡⁡⁡⁡⁡⁡𝑡 < 𝑎
𝐿−1 {𝑒 −𝑎𝑠 𝐹(𝑠)} = ⁡ {
𝑓(𝑡 − 𝑎)𝑡 > 𝑎
In the case the function 𝐹(𝑠) is multiplied by 𝑒 −𝑎𝑠

Example 23
Find inverse L.T. of the function
𝝅𝒔
𝒆− 𝟑
𝐹(𝑠) = 𝟐
𝒔 +𝟏
First we must test the limit of 𝐹(𝑠) as the following
𝜋𝑠

𝑒 3
lim 𝐹(𝑠) = lim = 0⁡ then the inverse Laplace transform exists.
𝑠→∞ 𝑠→∞ 𝑠 2 +1
𝜋𝑠

𝑒 3 𝜋𝑠 1
𝐿−1 {𝑒 −𝑎𝑠 𝐹(𝑠)} = ⁡ 𝐿−1 { } = ⁡ 𝐿−1 {𝑒 − 3 ⁡⁡ }
𝑠2 + 1 𝑠2 + 1

1
𝐿−1 { 2 } = sin 𝑡
𝑠 +1

102
𝜋𝑠 𝜋
𝑒− 3 0⁡⁡⁡⁡⁡⁡⁡⁡⁡⁡⁡⁡⁡⁡𝑡 <
= 𝐿−1 {𝑒 −𝑎𝑠 𝐹(𝑠)} = ⁡ 𝐿−1 { 2 }={ 3 = 𝐺(𝑡)
𝑠 +1 𝜋 𝜋
𝑓 (𝑡 − ) 𝑡 >
3 3

v. Functions which can be decomposed using Partial Fractions Method


Any rational function 𝑃(𝑠)/𝑄(𝑠)⁡𝑤ℎ𝑒𝑟𝑒⁡𝑃(𝑠)⁡𝑎𝑛𝑑⁡𝑄(𝑠) are polynomials, with the
degree of 𝑃(𝑠)⁡𝑙𝑒𝑠𝑠⁡𝑡ℎ𝑎𝑛⁡𝑡ℎ𝑎𝑡⁡𝑜𝑓⁡𝑄(𝑠) can be written as the sum of rational function
(called partial fractions) having the form
𝐴 𝐴𝑠 + 𝐵
⁡,
(𝑎𝑠 + 𝑏)𝑟 (𝑎𝑠 2 + 𝑏𝑠 + 𝑐)𝑟
where 𝑟 = 1,2,3, . ..

By finding the inverse Laplace transform of each of the partial fractions , we can find

𝑃(𝑠)
𝐿−1 { }
𝑄(𝑠)
Example 24
Find inverse Laplace of the function
𝟏
𝒔(𝒔 − 𝟏)(𝒔 − 𝟐)(𝒔 − 𝟑)
First we must test the limit of 𝐹(𝑠) as the following
𝟏
lim 𝐹(𝑠) = lim = 0,⁡then the inverse Laplace transform exists.
𝑠→∞ 𝑠→∞ 𝒔(𝒔−𝟏)(𝒔−𝟐)(𝒔−𝟑)

Using the previous case to calculate the inverse of Laplace transform as the following:
𝟏 𝑨 𝑩 𝒄 𝑫
𝐹(𝑠) = = +⁡ +⁡ +
𝒔(𝒔 − 𝟏)(𝒔 − 𝟐)(𝒔 − 𝟑) 𝒔 (𝒔 − 𝟏) (𝒔 − 𝟐) (𝒔 − 𝟑)
By solving this equation, it is obtained.
1 1 1 1
𝐴 = − ⁡, 𝐵 = ⁡, 𝐶 = ⁡ − ⁡, 𝐷 =
6 2 2 6
1 −1 1 1 1 −1 1 1 1
𝐹(𝑠) = ⁡ = +⁡ +⁡ +
𝑠(𝑠 − 1)(𝑠 − 2)(𝑠 − 3) 6 𝑠 2 (𝑠 − 1) 2 (𝑠 − 2) 6 (𝑠 − 3)
−1 1 1 1 −1 1 1 1
𝑓(𝑡) = ⁡ 𝐿−1 {𝐹(𝑠)} = ⁡ 𝐿−1 { +⁡ +⁡ + }
6 𝑠 2 (𝑠 − 1) 2 (𝑠 − 2) 6 (𝑠 − 3)
103
−1 1 𝑡 1 2𝑡 1 3𝑡
𝑓(𝑡) = ⁡ 𝐿−1 {𝐹(𝑠)} = 𝐿−1 { + 𝑒 − 𝑒 + 𝑒 }
6 2 2 6

Example 25
Find inverse Laplace of the function
𝒔𝟐 − 𝟒𝒔 + 𝟕
𝒔𝟒 − 𝟒𝒔𝟑 + 𝟔𝒔𝟐 − 𝟒𝒔 + 𝟏
First we must test the limit of 𝐹(𝑠) as the following
𝒔𝟐 −𝟒𝒔+𝟕
lim 𝐹(𝑠) = lim = 0⁡then the inverse Laplace transform exists.
𝑠→∞ 𝑠→∞ 𝒔𝟒 −𝟒𝒔𝟑 +𝟔𝒔𝟐 −𝟒𝒔+𝟏

Using the previous case to calculate the inverse of Laplace transform as the following:
𝒔𝟐 − 𝟒𝒔 + 𝟕 𝒔𝟐 − 𝟒𝒔 + 𝟕
𝐹(𝑠) ⁡ = 𝟒 =
𝒔 − 𝟒𝒔𝟑 + 𝟔𝒔𝟐 − 𝟒𝒔 + 𝟏 (𝒔 − 𝟏)𝟒
𝑨 𝑩 𝒄 𝑫
=⁡ + ⁡ + ⁡ +
(𝒔 − 𝟏)𝟒 (𝒔 − 𝟏)𝟑 (𝒔 − 𝟏)𝟐 (𝒔 − 𝟏)
Solving this equation, one obtain
𝐴 = 4⁡, 𝐵 = −2, 𝐶 = ⁡1, 𝐷 = 0
𝟒 𝟐 𝟏
𝐹(𝑠) = − ⁡ + ⁡
(𝒔 − 𝟏)𝟒 (𝒔 − 𝟏)𝟑 (𝒔 − 𝟏)𝟐
4 2 1
𝑓(𝑡) = ⁡ 𝐿−1 {𝐹(𝑠)} = ⁡ 𝐿−1 { − ⁡ + ⁡ ⁡}
(𝑠 − 1)4 (𝑠 − 1)3 (𝑠 − 1)2
4 3! 2 1
𝑓(𝑡) = ⁡ 𝐿−1 {𝐹(𝑠)} = ⁡ 𝑒 𝑡 𝐿−1 { − ⁡ + }
3! (𝑠)4 (𝑠)3 (𝑠)2
2
𝑓(𝑡) = ⁡ 𝐿−1 {𝐹(𝑠)} = ⁡ 𝑒 𝑡 ( 𝑡 3 − ⁡ 𝑡 2 + 𝑡)
3

vi. Functions whose Derivatives has a known Inverse


In this case, the function 𝐹(𝑠) can be reducing to one of the standard forms by
𝑑𝐹(𝑠)
calculating = 𝐹 ′ (𝑠)𝑖𝑓⁡𝐿−1 {𝐹(𝑠)} = 𝑓(𝑡), 𝑡ℎ𝑒𝑛
𝑑𝑠

−1⁡ (𝑛) −1
𝑑 (𝑛)
𝐿 {𝐹 (𝑠)} = ⁡ 𝐿 { (𝑛) ⁡𝐹(𝑠)} = (−1)𝑛 𝑡 𝑛 𝑓(𝑡)
𝑑𝑠

104
Example 26
Find inverse laplace of the function
𝒔+𝟏
𝒍𝒏
𝒔−𝟏
First we must test the limit of 𝐹(𝑠) as the following
𝒔+𝟏
lim 𝐹(𝑠) = lim = 0⁡then the inverse Laplace transform exists.
𝑠→∞ 𝑠→∞ 𝒔−𝟏

Using the previous case to calculate the inverse Of Laplace transform as the following:
𝑠+1
𝐹(𝑠) ⁡ = ⁡𝑙𝑛 = ln(𝑠 + 1) − ln⁡(𝑠 − 1)
𝑠−1
1 1
𝐹′(𝑠) ⁡ = −⁡
𝑠+1 𝑠−1
1 1
𝐿−1 {𝐹′(𝑠)} = ⁡ 𝐿−1 { −⁡ } = ⁡ 𝑒 −𝑡 − ⁡ 𝑒 𝑡 ⁡⁡⁡⁡⁡⁡⁡⁡⁡⁡⁡⁡⁡⁡⁡⁡⁡⁡(𝑖)
𝑠+1 𝑠−1
𝑑 (𝑛)
 𝐿−1 {𝐹 (𝑛) (𝑠)} = ⁡ 𝐿−1 { 𝐹(𝑠)} = (−1)𝑛 𝑡 𝑛 𝑓(𝑡)
𝑑𝑠 (𝑛)

−1 (𝑛) −1
𝑑 (𝑛)
𝐿 {𝐹 (𝑠)} = ⁡ 𝐿 { (𝑛) 𝐹(𝑠)} = (−1)𝑡𝑓(𝑡) = −𝑡𝑓(𝑡)⁡⁡⁡⁡⁡⁡⁡⁡⁡⁡⁡⁡(𝑖𝑖)
𝑑𝑠
𝑒 𝑡 − ⁡ 𝑒 −𝑡
= 𝑓(𝑡) = ⁡
𝑡

vii. Functions whose Integral has a known Inverse


In this case, the function F(s) can be reducing to one of the standard forms by

calculating ∫𝑠 𝐹 (𝑠)𝑑𝑠. If 𝐿−1 {𝐹 (𝑠)} = 𝑓 (𝑡), 𝑡ℎ𝑒𝑛

𝑓(𝑡)
𝐿−1 {∫ 𝐹(𝑠)𝑑𝑠⁡} =
𝑡
𝑠

Example 27
Find inverse Laplace of the function
−𝟐𝒔
(𝒔𝟐 − 𝟏)𝟐
First we must test the limit of 𝐹(𝑠) as the following

105
−𝟐𝒔
lim 𝐹(𝑠) = lim = 0⁡then the inverse Laplace transform exists.
𝑠→∞ 𝑠→∞ (𝒔𝟐 −𝟏)𝟐

Using the previous case to calculate the inverse of Laplace transform as the following:
−2𝑠
𝐹(𝑠) ⁡ =
(𝑠 2 − 1)2
∞ ∞ ∞
−2𝑠
∫ 𝐹(𝑠)𝑑𝑠 = ⁡ ∫ 2 ⁡𝑑𝑠 = ⁡ ∫ −2𝑠(𝑠 2 + 1)−2 𝑑𝑠
(𝑠 − 1)2
𝑠 𝑠 𝑠

1 ∞ −1
=| | = ⁡
𝑠2 + 1 𝑠 𝑠2 + 1

𝑓(𝑡)
=> ⁡ 𝐿−1 {∫ 𝐹(𝑠)𝑑𝑠} =
𝑡
𝑠

−1 𝑓(𝑡)
𝐿−1 { 2 } = ⁡ −𝑠𝑖𝑛𝑡 =
𝑠 +1 𝑡
𝑓(𝑡) = ⁡ −𝑡⁡𝑠𝑖𝑛𝑡
viii. Functions divided by 𝒔
𝑭(𝒔)
If 𝑳−𝟏 { } = 𝑓(𝑡)
𝒔
𝑡
𝑭(𝒔)
𝑳−𝟏 { } = ⁡ ∫ 𝑓(𝑡)𝑑𝑡
𝒔
0

Thus division by 𝑠 (or multiplication by 1/𝑠) has the effect of integrating 𝑓(𝑡) from 0⁡𝑡𝑜⁡𝑡

Example 28
Find inverse Laplace of the function
𝟏
𝒔(𝒔𝟐 ⁡ + ⁡⁡𝟒)
First we must test the limit of 𝐹(𝑠) as the following
𝟏
lim 𝐹(𝑠) = lim = 0⁡then the inverse Laplace transform exists.
𝑠→∞ 𝑠→∞ 𝒔(𝒔𝟐 ⁡+⁡⁡𝟒)

Using the previous case to calculate the inverse of Laplace transform as the following:

106
𝟏
𝑭(𝒔) ⁡ =
(𝒔𝟐 ⁡ + ⁡⁡𝟒)
1
𝐿−1 {𝐹(𝑠)} = 𝑓(𝑡) = 𝑠𝑖𝑛2𝑡
2
1 𝑡 𝑡
𝐹(𝑠) ( 2 ) 1
𝐿−1 { } = ⁡ 𝐿−1 { 𝑠 ⁡ + ⁡⁡4 } = ⁡ ∫ 𝑓(𝑡)𝑑𝑡 = ⁡ ∫ ⁡𝑠𝑖𝑛2𝑡⁡𝑑𝑡
𝑠 𝑠 2
0 0

1
𝐹(𝑠) ( 2 ) 1
−1
𝐿 { −1
} =⁡𝐿 { 𝑠 ⁡ + ⁡⁡4 } = (1 − 𝑐𝑜𝑠2𝑡)
𝑠 𝑠 4

ix. Functions whose Inverse seems to be Periodic


In this case we can see the term (1 − 𝑒 −𝑝𝑠 ) in the denominator

Example 29
Find inverse Laplace of the function
𝟏
𝒔(𝟏 + ⁡ 𝒆−𝒔 )
First we must test the limit of 𝐹(𝑠) as the following
𝟏
lim 𝐹(𝑠) = lim = 0⁡then the inverse Laplace transform exists.
𝑠→∞ 𝑠→∞ 𝒔(𝟏+⁡𝒆−𝒔 )

Using the previous case to calculate the inverse of Laplace transform as the following:
𝟏
𝐹(𝑠) ⁡ =
𝒔(𝟏 + ⁡ 𝒆−𝒔 )
Multiply F(s) by the conjugate of the term (1 + 𝑒 −𝑠 ) which is equal (1 - 𝑒 −𝑠 )
(1⁡ − ⁡ 𝑒 −𝑠 )⁡ (1⁡ − ⁡ 𝑒 −𝑠 )⁡
𝐹(𝑠) ⁡ = =⁡
𝑠(1 + ⁡ 𝑒 −𝑠 )(1⁡ − ⁡ 𝑒 −𝑠 )⁡⁡ 𝑠(⁡1 − ⁡ 𝑒 −2𝑠 )
1 𝑒 −𝑠
(𝑠 − ⁡ 𝑠 )
𝐹(𝑠) ⁡ =
(1 − ⁡ 𝑒 −2𝑠 )
1 𝑒 −𝑠
 𝐿−1 { − ⁡ } = 𝑢(𝑡) − 𝑢(𝑡 − 1)
𝑠 𝑠

107
i.e. the function 𝑓(𝑡) is a periodic function with 𝑝𝑒𝑟𝑖𝑜𝑑⁡ = ⁡2, then
𝑓(𝑡) = 𝑓(𝑡 + 2) = 𝑢(𝑡) − 𝑢(𝑡 − 1)𝑎𝑛𝑑⁡⁡0 < 𝑡 < 2

x. Functions whose Inverse can be obtained using Convolution Theorem


Convolution Theorem
If 𝐹1 (𝑠)⁡𝑎𝑛𝑑⁡𝐹2 (𝑠) are the transforms of the functions 𝑓1 (𝑠)⁡𝑎𝑛𝑑⁡𝑓2 (s) respectively,
𝑡
i.e. 𝐿 {∫0 𝑓1 (𝑡)⁡} = 𝐹1 (𝑠)⁡⁡𝑎𝑛𝑑⁡⁡𝐿{𝑓2 (𝑡)} = ⁡ 𝐹2 (𝑠)𝑡ℎ𝑒𝑛
𝑡

𝐿 {∫ 𝑓1 (𝜏)𝑓2 (𝑡 − 𝜏) 𝑑𝜏} = ⁡ 𝐹1 (𝑠). 𝐹2 (𝑠)


0

The inverse of the convolution theorem is hold


𝑡

𝐿−1 {𝐹1 (𝑠). 𝐹2 (𝑠)} = ⁡ ∫ 𝑓1 (𝜏)𝑓2 (𝑡 − 𝜏)𝑑𝜏


0

where 𝑓1 (𝑡) = ⁡ 𝐿−1 {𝐹1 (𝑠)}𝑎𝑛𝑑⁡𝑓2 (𝑡) = ⁡ 𝐿−1 {𝐹2 (𝑠)}

Example 30
Find inverse Laplace of the function
𝟏
(𝒔 − 𝟏)(𝒔 − 𝟐)
First we must test the limit of 𝐹(𝑠) as the following
𝟏
lim 𝐹(𝑠) = lim = 0⁡then the inverse Laplace transform exists.
𝑠→∞ 𝑠→∞ (𝒔−𝟏)(𝒔−𝟐)

Using the previous case to calculate the inverse of Laplace transform as the following
1 1
𝐹1 (𝑠) = ⁡⁡⁡⁡⁡⁡⁡𝑎𝑛𝑑⁡⁡⁡⁡⁡⁡⁡⁡⁡⁡⁡⁡⁡𝐹2 (𝑠) =
𝑠−1 𝑠−2
1
𝑓1 (𝑡) = ⁡ 𝐿−1 {𝐹1 (𝑠)} = ⁡ 𝐿−1 { } = ⁡ 𝑒𝑡
(𝑠 − 1)
1
𝑓2 (𝑡) = ⁡ 𝐿−1 {𝐹2 (𝑠)} = ⁡ 𝐿−1 { } = ⁡ 𝑒 2𝑡
(𝑠 − 2)
108
𝑡
1
𝐿−1 { } = ⁡ ∫ 𝑓1 (𝜏)𝑓2 (𝑡 − 𝜏)𝑑𝜏
(𝑠 − 1)(𝑠 − 2)
0
𝑡

= ∫ 𝑒 𝜏 𝑒 2(𝑡−𝜏) 𝑑𝜏 = ⁡ 𝑒 2𝑡 − ⁡ 𝑒 𝑡 ⁡
0

Note that
This example can be solved by Partial Fractions method.

Example 31
Find inverse Laplace of the function
𝟏
𝒔𝟐 (𝒔 + 𝟏)𝟐
First one must test the limit of F(s) as the following
𝟏
lim 𝐹(𝑠) = lim = 0,⁡⁡⁡then the inverse Laplace transform exists.
𝑠→∞ 𝑠→∞ 𝒔𝟐 (𝒔+𝟏)𝟐

Using the previous case to calculate the inverse of Laplace transform as the following
1 1
𝐹1 (𝑠) = ⁡⁡⁡⁡⁡⁡⁡𝑎𝑛𝑑⁡⁡⁡⁡⁡⁡⁡⁡⁡⁡⁡⁡⁡𝐹2 (𝑠) =
𝑠2 (𝑠 + 1)2
1
𝑓1 (𝑡) = ⁡ 𝐿−1 {𝐹1 (𝑠)} = ⁡ 𝐿−1 { 2 } = ⁡𝑡
𝑠
1
𝑓2 (𝑡) = ⁡ 𝐿−1 {𝐹2 (𝑠)} = ⁡ 𝐿−1 { } = ⁡ 𝑡𝑒 −𝑡
(𝑠 + 1)2
𝑡
1
𝐿−1 { 2 } = ⁡ ∫ 𝑓1 (𝜏)𝑓2 (𝑡 − 𝜏)𝑑𝜏
𝑠 (𝑠 + 1)2
0
𝑡 𝑡

= ∫ 𝜏(𝑡 − 𝜏)𝑒 −𝑡+𝜏 ⁡𝑑𝜏 = ⁡ 𝑒 −𝑡 ∫(𝑡𝜏 − 𝜏 2 )𝑒 𝜏 𝑑𝜏


0 0
𝑡
= ⁡ 𝑒 −𝑡 |(𝑡𝜏 − ⁡ 𝜏 2 )𝑒 𝜏 − ∫(𝑡 − 2𝜏)𝑒 𝜏 𝑑𝜏|
0
= ⁡𝑡⁡ − 2⁡ + ⁡ (𝑡⁡ + 2)⁡𝑒 −𝑡

109
L.T. for Solving Linear ODE with Constant Coefficients

Laplace transform is useful in solving linear ordinary differential equations with constant
coefficients.
For example, suppose we wish to solve the second order linear differential equation
𝑑2𝑦 𝑑𝑦
+ 𝑎 + 𝑏 = 𝑓(𝑡)⁡⁡𝑜𝑟⁡⁡𝑦 ′′ + 𝑎𝑦 ′ + 𝑏 = 𝑓(𝑡)
𝑑𝑡 2 𝑑𝑡
where a and b are constants, subject to the initial or boundary conditions
𝑦(0) ⁡ = ⁡𝐴⁡⁡⁡⁡⁡⁡⁡⁡⁡⁡⁡⁡,⁡⁡⁡⁡⁡⁡⁡⁡⁡⁡⁡⁡⁡⁡⁡⁡𝑦′(0) ⁡ = 𝐵
where 𝐴⁡𝑎𝑛𝑑⁡𝐵 are given constants. On taking the Laplace transform of both sides of the
differential equation and using the initial conditions, we obtain an algebraic equation for
determination of 𝐿(𝑦(𝑡)) ⁡ = 𝑌(𝑠)
The required solution is then obtained by finding the inverse Laplace transform of 𝑌(𝑠).
The method is easily extended to higher order differential equations.
Using theorem of Laplace transform of derivatives, we have
𝐿(𝑦(𝑡)) ⁡ = ⁡𝑌(𝑠)
𝐿(𝑦′(𝑡)) ⁡ = ⁡𝑠⁡𝑌(𝑠) ⁡ − ⁡𝑦(0)
𝐿(𝑦′(𝑡)) ⁡ = ⁡𝑠⁡′𝑌(𝑠) − ⁡𝑠⁡𝑦(0) − 𝑦′(0)
generally
𝐿{𝑦 (𝑛) (𝑡)} = ⁡ 𝑠 𝑛 𝑌(𝑠) − ⁡ 𝑠 𝑛−1 𝑦(0) − ⋯ … … … . −⁡𝑦 (𝑛−1) (0)
where
𝑦(0), 𝑦′(0), . . . . . . . . . , 𝑦 (𝑛−1) (0) are initial conditions included within the transform of the
differential equation.

Example 32
Solve the differential equation 𝒚′′ + 𝟗𝒚 = 𝟎 with the initial conditions
𝒚(𝟎) = 𝟎,⁡⁡⁡⁡⁡⁡⁡⁡⁡⁡⁡⁡⁡⁡⁡⁡𝒚′(𝟎) ⁡ = ⁡𝟐
Taking the Laplace transform differential equation
𝐿{𝑦 ′′ } + 𝐿{9𝑦} = 0

110
𝑌(𝑠) − 𝑠𝑦(0) − 𝑦′(0) + 9𝑌(𝑠) ⁡ = ⁡0
Using the given conditions, one get
2 2 3
(𝑠 2 + 9⁡)𝑌(𝑠) = 2⁡⁡⁡ => ⁡⁡⁡⁡⁡𝑌(𝑠) = ⁡ = ( )
(𝑠 2 + 9) 3 𝑠 2 + 9
Taking inverse Laplace transform of both sides
𝑌(𝑡) ⁡ = ⁡2/3⁡𝑠𝑖𝑛⁡3𝑡 Check your result

Example 33
Solve the differential equation 𝒚′′ − 𝟑𝒚′ + 𝟐𝒚 = 𝟒𝒆𝟐𝒕 using Laplace transform method
with the initial conditions 𝒚(𝟎) ⁡ = ⁡ −𝟑⁡, 𝒚′(𝟎) ⁡ = 𝟓
Taking the Laplace transform of both sides of the differential equation
𝐿{𝑦′′} − 3𝐿{𝑦 ′ } + 2𝐿{𝑦} = 4𝐿{𝑒 2𝑡 }
4
(𝑠 2 𝑌(𝑠) − 𝑠𝑦(0) − 𝑦 ′ (0)) − 3(𝑠𝑌(𝑠) − 𝑦(0) + 2𝑌(𝑠)) =
𝑠−2
Using the given conditions, we have
4
(𝑠 2 − 𝑠(−3) − 5) − 3(𝑠𝑌(𝑠) − (−3) + 2𝑌(𝑠)) =
𝑠−2
4
(𝑠 2 − 3𝑠 + 2)𝑌(𝑠) + 3𝑠 − 14⁡ =
𝑠−2
4 14 − 3𝑠
𝑌(𝑠) ⁡ = 2 +⁡ 2
(𝑠 − 3𝑠 + 2)(𝑠 − 2) (𝑠 − 3𝑠 + 2)
3𝑠 2 + 20𝑠 − 24 −7 4 4
𝑌(𝑠) = = + ⁡ + ⁡
(𝑠 − 1)(𝑠 − 2)2 𝑠 − 1 𝑠 − 2 (𝑠 − 2)2

Taking inverse Laplace transform of both sides

𝑦(𝑡) = −7𝑒 𝑡 + 4𝑒 2𝑡 + 4𝑡𝑒 2𝑡 Check your result

111
Solved Problems:
Problem 1
Using the Laplace table to find the Laplace transform of
4 sin 3𝑡
a) 5 + 𝑡 3 +
𝑒 −2𝑡
1
b) 7𝑒 3𝑡 𝑠𝑖𝑛6𝑡 − 𝑡𝑐𝑜𝑠5𝑡
7

Answers
5 6 12
a) + + ⁡ (𝑠−2)2
𝑠 −𝑠 4 +9
42 𝑠 2 −⁡52
b) (𝑠−3)2 +36
−⁡
7(𝑠 2 +⁡52 )2

Problem 2
3 2 𝑠+2
Find 𝐿−1⁡ { 5 − 4} ⁡𝑎𝑛𝑑⁡⁡𝐿−1 { }
𝑠 𝑠 𝑠 2 +4

𝑡4 𝑡3
𝑓(𝑡) = − ⁡ ⁡⁡𝑎𝑛𝑑⁡𝑐𝑜𝑠2𝑡 + ⁡⁡𝑠𝑖𝑛2𝑡
8 3
Problem 3
𝑠+6 𝑠+5
Find 𝐿−1⁡ { } ⁡⁡𝑎𝑛𝑑⁡⁡⁡𝐿−1 { }
𝑠 2 +6𝑠+10 𝑠 2 +4𝑠+8

(𝑠 + 3) + 3
𝐿−1⁡ { } = ⁡ 𝑒 −3𝑡 ⁡𝑐𝑜𝑠𝑡 + 3𝑒 −3𝑡 𝑠𝑖𝑛𝑡
(𝑠 + 3)2 + 1

(𝑠 + 2) + 3 3 −2𝑡
𝐿−1⁡ { } = ⁡ 𝑒 −2𝑡
⁡𝑐𝑜𝑠2𝑡 + 𝑒 𝑠𝑖𝑛2𝑡
(𝑠 + 2)2 + 4 2
Problem 4
7𝑠+11
Find 𝐿−1⁡ { }
𝑠 2 +𝑠−6

7𝑠 + 11 2 5
𝑳−𝟏 { 2 } = ⁡ 𝐿−1 { +⁡ } = 2𝑒 −3𝑡 + 5𝑒 2𝑡
𝑠 +𝑠−6 𝑠+3 𝑠−2
Problem 5
Find the inverse Laplace transform for the followings:
𝟑𝒔+𝟐
a) 𝑭(𝒔) =
𝒔𝟑

112
𝟑⁡𝒔𝟐 +𝟐𝒔+𝟓
b) 𝑭(𝒔) = (𝒔+𝟏)(𝒔𝟐
+𝟏)

Answers
3𝑠+2 3 2
a) 𝐿−1 { } = ⁡ 𝐿−1 { 2 + ⁡ 3} = 3𝑡 + ⁡ 𝑡 2
𝑠3 𝑠 𝑠
3⁡𝑠 2 +2𝑠+5 3 2
b) 𝐿−1 {(𝑠+1)(𝑠2 } = ⁡ 𝐿−1 { +⁡ } = 3𝑒 −𝑡 + 2𝑠𝑖𝑛𝑡
+1) 𝑠+1 𝑠 2 +1

Problem 6
𝑑𝑦
solve + 𝑦 = 2𝑒 𝑡⁡⁡⁡⁡⁡⁡⁡⁡ ⁡⁡⁡⁡⁡⁡⁡⁡⁡𝑤𝑖𝑡ℎ⁡𝑦(0) = 2
𝑑𝑡

2
𝑠𝑌(𝑠) − 𝑦(0) + 𝑌(𝑠) =
𝑠−1
2𝑠 1 1
𝑌(𝑠) ⁡ = =⁡ +⁡
(𝑠 − 1)(𝑠 + 1) 𝑠−1 𝑠+1
𝑦(𝑡) ⁡ = ⁡ 𝑒 𝑡 + ⁡ 𝑒 −𝑡
Problem 7
𝑑𝑥
Solve − 𝑥 = 2 − ⁡ 𝑡 2 ⁡⁡⁡⁡⁡⁡⁡⁡⁡⁡⁡⁡⁡⁡⁡⁡𝑤𝑖𝑡ℎ⁡⁡𝑥(0) = 0
𝑑𝑡

2 2
𝑠𝑋(𝑠) − 𝑥(0) + ⁡𝑋(𝑠) = −
𝑠 𝑠3
2(𝑠 + 1) 1 1
𝑋(𝑠) = = ⁡2 ( − )
𝑠3 𝑠 𝑠3
𝑥(𝑡) ⁡ = ⁡2𝑡⁡ + 𝑡 2

Problem 8
𝑑 2 ⁡𝑥 𝑑𝑥
Solve +3 − 4𝑥 = 5𝑒 𝑡 ,⁡⁡⁡⁡⁡⁡⁡⁡⁡𝑥(0) = 0⁡, 𝑥 ′ (0) = 1
𝑑𝑡 2 𝑑𝑡

5
𝑠 2 𝑋(𝑠) − 𝑠𝑥(0) − 𝑥 ′ (0) + 3𝑠𝑋(𝑠) − 3𝑥(0) − 4𝑋(𝑠) = ⁡
𝑠−1
1
𝑋(𝑠) ⁡ =
(𝑠 − 1)2
𝑥(𝑡) ⁡ = 𝑡𝑒 𝑡

113
Exercises
𝑸𝟏 / Use the integral definition to find laplace transforms of the following functions;

𝑓(𝑡) = 𝑡 𝑓(𝑡) = ⁡ 𝑒 𝑎𝑡 𝑓(𝑡) = sinh 𝑡

𝑸𝟐 / Find laplace transforms of the following functions :


𝑓(𝑡) = sin 𝑎𝑡 𝑓(𝑡) = ⁡ 𝑒 2𝑡 sin(3𝑡 𝑓(𝑡) = ⁡ 𝑒 −𝑡 𝑠𝑖𝑛2 (𝑡) 𝑓(𝑡) = ⁡ 𝑡 2 sin(2𝑡)
+ 1)
𝑡 𝑡
𝑓(𝑡) 𝑓(𝑡) = ⁡ 𝑒 2𝑡 𝑡𝑐𝑜𝑠(2𝑡) ⁡1 − ⁡ 𝑒 −𝑡 1 − 𝑐𝑜𝑠𝑡
= ⁡ 𝑒 2𝑡 sin(2𝑡) 𝑓(𝑡) = ⁡ ∫ 𝑑𝑡⁡ 𝑓(𝑡) = ⁡ ∫ 𝑑𝑡
𝑡 𝑡
0 0

𝑸𝟑 / Graph the following periodic functions which are assumed to have a period and find
their laplace transforms :
1)⁡𝑓(𝑡) = 𝑡 2 ⁡,⁡⁡⁡0 < 𝑡 < 2𝜋⁡⁡⁡⁡⁡⁡⁡⁡⁡⁡⁡⁡⁡⁡⁡⁡⁡⁡⁡⁡⁡⁡⁡2)𝑓(𝑡) = 2𝜋 − 𝑡, 0 < 𝑡 < 2𝜋
0⁡⁡⁡⁡⁡⁡⁡⁡⁡⁡⁡⁡⁡⁡⁡⁡⁡0 < 𝑡 < 𝜋 𝑡⁡⁡⁡⁡⁡⁡⁡0 < 𝑡 < 1
3)⁡𝑓(𝑡) = ⁡ { ⁡⁡⁡⁡⁡⁡⁡⁡⁡⁡⁡⁡⁡4)⁡𝑓(𝑡) = ⁡ {
𝑡 − 𝜋⁡⁡⁡⁡⁡⁡⁡𝜋 < 𝑡 < 2𝜋 0⁡⁡⁡⁡⁡⁡⁡⁡1 < 𝑡 < 2

𝑸𝟒 / Find the inverse laplace transforms of the following functions:


𝑠 1 − 𝑠⁡ 6𝑠 − 4
𝐹(𝑠) = ⁡ 𝐹(𝑠) = ⁡ 𝐹(𝑠) = ⁡
(𝑠 + 1)2 (𝑠 2 − 2𝑠 + 2)2 𝑠 2 − 4𝑠 + 20
3𝑠 + 7 1 1
𝐹(𝑠) = ⁡ 𝐹(𝑠) ⁡ = (𝑠2 𝐹(𝑠) = ⁡
(𝑠 − 1)2 − 4 +1)2 𝑠 2 (𝑠 2 − 1)

𝑸𝟓 / Find the inverse laplace transforms of the following functions;


𝑠2 + 1 1
𝐹(𝑠) = ⁡𝑙𝑛 𝐹(𝑠) = 𝑙𝑛 (1 + )
(𝑠 − 1)2 𝑠2
1
𝐹(𝑠) ⁡ = cot −1 (𝑠 + 1) 𝐹(𝑠) = ⁡
(𝑠 − 4)2

114
𝑸𝟔 / Find the inverse laplace transforms of the following functions:

𝑒 −2𝑠 𝑒 −𝑠 𝑒 −𝑠
𝐹(𝑠) = ⁡ 4 𝐹(𝑠) = ⁡ 𝐹(𝑠) = 2
𝑠 𝑠−2 𝑠 +4
𝐹(𝑠) 𝑠
1 − ⁡ 𝑒 −2𝑠 1
𝐹(𝑠) = ⁡ 2 𝐹(𝑠) = ⁡
𝑠 + 16 = (𝑠 2 + 2𝑠 + 2⁡)2
𝑠(𝑠 2 ⁡ + ⁡⁡1)(𝑠 2 ⁡ − 1)

𝑸𝟕 / Solve the following ordinary differential equations (initial value problems);

1) 𝑦 ′′ − 3𝑦 ′ + 2𝑦 = 6⁡𝑒 −𝑡 ⁡𝑤𝑖𝑡ℎ⁡𝑡ℎ𝑒⁡𝐼. 𝐶𝑠. 𝑦(0) = 𝑦 ′ (0) = 3

2) 𝑦 ′′ + 2𝑦 ′ + 2𝑦 = 10𝑠𝑖𝑛2𝑡⁡𝑤𝑖𝑡ℎ⁡𝑡ℎ𝑒⁡𝐼. 𝐶𝑠. 𝑦(0) = −1⁡, 𝑦 ′ (0) = −3

3) 𝑦 ′′ − 2𝑦 ′ + 5𝑦 = 8𝑠𝑖𝑛𝑡 − 4𝑐𝑜𝑠𝑡⁡𝑤𝑖𝑡ℎ⁡𝐼. 𝐶𝑠. 𝑦(0) = 0⁡, 𝑦 ′ (0) = 1

4) 𝑦 ′′ + 2 = −2𝑠𝑖𝑛𝑡⁡⁡⁡⁡⁡𝑤𝑖𝑡ℎ⁡𝑡ℎ𝑒⁡𝐼. 𝐶𝑠⁡.⁡⁡𝑦(0) = 0⁡, 𝑦 ′ (0) = 1

5) 𝑦 ′′ + 4𝑦 = 4(𝑐𝑜𝑠2𝑡 − 𝑠𝑖𝑛𝑡)𝑤𝑖𝑡ℎ⁡𝑡ℎ𝑒⁡𝐼. 𝐶𝑠. 𝑦(0) = 1, 𝑦 ′ (0) = 3

6) 𝑦 ′′ + 2𝑦 ′ + 5𝑦 = ⁡ 𝑒 −𝑡 𝑠𝑖𝑛𝑡⁡𝑤𝑖𝑡ℎ⁡𝑡ℎ𝑒⁡𝐼. 𝐶𝑠. 𝑦(0) = 0, 𝑦 ′ (0) = 1

115
Additional Exercices

116
117
118
119
120
121
122
123
Chapter Three
Multiple Integral

objectives
This chapter is dedicated to assist the student to understand and to learn the
fundamental topics of multiple integrals and its applications. These topics include
(definition, types of integrals, importance, applications, Jacobeans). The main objectives
of this chapter are listed below to learn the students how to:
i) reduce triple integral to double integral and consequently to single finite integral.
ii) use the Jacobean of transformation to covert the integral from Cartesian
coordinates into polar coordinates.
iii) calculate surface and volume integrals.
iv) use multiple integrals in many different engineering applications such as: fluid
mechanics, solid mechanics, mechanics of materials, structure analysis… and many
other applications.

Introduction
Multiple integral is considered as a general form of finite integral for different regions in
plane or in space. The value of this integral can be calculated alternately. If the integral is
triple it should be converted into double integral and sequentially to single finite integral,
therefore this type of integral is called successive or sequential integral. In this type of
integral, the functions of real variables of different coordinates are integrated on lines, 2𝐷
surfaces, and 3𝐷 surfaces. In all cases the integrations are formulated and calculated by a
similar style. Multiple integral is divided into two categories; scalar multiple integral and
vector merged multiple integral, where the merged vector integral can be easily converted
into multiple scalar integral. Multiple integrals have great importance in numerous
applications in Engineering and scientific fields. These fields include; structure analysis,
fluid mechanics, solid mechanics, mechanics of materials, and many other fields and
miscellaneous applications.

124
1. Double Integral
In Cartesian coordinates on the 𝑥𝑦 plane, the rectangular element of area is
∆𝐴 = ⁡ ∆𝑥∆𝑦
Summing all such elements of area along a vertical strip, the area of the elementary vertical
slip is
𝜓 (𝑥 )

( ∑ ∆𝑦) ∆𝑥
𝑦=𝜙(𝑥)

Summing all the strips across the region 𝑅, the


total are of the region

𝑏 𝜓(𝑥)
𝐴⁡ ≈ ∑ ( ∑ ∆𝑦) ∆𝑥
𝑥=𝑎 𝑦=𝜙(𝑥)
( )
In the limit as the elements ∆𝑥 and ∆𝑦 shrink to zero, this sum becomes
𝑏 𝜓(𝑥)
𝐴=∫ ∫ 1⁡⁡⁡⁡𝑑𝑦⁡𝑑𝑥⁡
𝑥=𝑎⁡⁡ ⁡𝑦=𝜙(𝑥)

If the surface density 𝜎 within the region is a function of location, 𝜎⁡ = ⁡𝑓⁡(𝑥, 𝑦), then the
mass of the region is

125
𝑏 𝜓(𝑥)
𝐴=∫ (∫ 𝑓(𝑥, 𝑦)𝑑𝑦)⁡𝑑𝑥
𝑥=𝑎 𝑦=𝜙(𝑥)

(Thickness = unity)
The inner integral must be evaluated first.

Re-iteration (Horizontal Slip)


One may reverse the order of integration by
summing the elements of area ∆𝐴 horizontal
first, then adding the strips across the region
from bottom to top. This generates the
double integral for the total area of the region
𝑏 𝜓 (𝑦 )
𝐴=∫ (∫ 1𝑑𝑥⁡)⁡𝑑𝑦⁡⁡⁡
𝑦=𝑎 𝑥=𝜙(𝑦)

The mass becomes


𝑏 𝜓 (𝑦 )
𝑀=∫ (∫ 𝑓(𝑥, 𝑦)𝑑𝑥⁡)⁡𝑑𝑦⁡⁡⁡
𝑦=𝑎 𝑥=𝜙(𝑦)

Choose the orientation of elementary strips that generates the simpler double integration.
We will first briefly outline the definition of the do integral:

∫ ∫ 𝑓(𝑥, 𝑦)𝑑𝑥𝑑𝑦
𝐷

126
The function 𝑓(𝑥, 𝑦) is defined over a closed region 𝐷 of the⁡𝑥𝑦 plane. For example, 𝑦 =
±𝑥, 𝑥 = 2

Example 2

Evaluate 𝑰 = ∫ ∫ℝ(𝟔𝒙 + 𝟐𝒚𝟐 )𝒅𝑨 where 𝑹 is the region enclosed by the parabola 𝒙=
𝒚𝟐 and 𝒙 + ⁡𝒚 = ⁡𝟐

127
The upper boundary changes form at 𝑥⁡ = ⁡1. The left boundary is the same throughout 𝑅.
The right boundary is the same throughout 𝑅. Therefore, choose horizontal strips.

Horizontal Slip
1 2−𝑦
𝐼=∫ ∫ (6𝑥 + 2𝑦 2 )𝑑𝑥⁡𝑑𝑦
−2 𝑦 2

Vertical Slip
1 √𝑥
𝐼=∫ ∫ (6𝑥 + 2𝑦 2 )𝑑𝑦⁡𝑑𝑥
0 −√𝑥
4 2−𝑥
+∫ ∫ (6𝑥 + 2𝑦 2 )𝑑𝑦⁡𝑑𝑥
1 −√𝑥

128
4

129
5

130
Polar Double Integrals (Change of Variables: Jacobians)
You may recall when you learned to integrate using trig substitution, that when a
substitution was made, a change in the integrand was necessary. For example, if you say
let 𝑥⁡ = 𝑠𝑖𝑛𝜃, then 𝑑𝑥 is replaced by 𝑑𝑥⁡ = ⁡𝑐𝑜𝑠𝜃𝑑𝜃⁡𝑎𝑛𝑑⁡𝑐𝑜𝑠𝜃 becomes a factor of the
integrand. In general, if we change a single integral by putting 𝑥⁡ = ⁡𝑔(𝑢), then 𝑑𝑥⁡ =
⁡𝑔′⁡(𝑢)𝑑𝑢. Hence we can write
𝑏 𝑑
∫ 𝑓(𝑥)𝑑𝑥 = ⁡ ∫ 𝑓(𝑔(𝑢))𝑔′ (𝑢)𝑑𝑢
𝑎 𝑐

where 𝑎⁡ = ⁡𝑔(𝑐)⁡𝑎𝑛𝑑⁡𝑏⁡ = ⁡𝑔(𝑑). In double integrals, making such a change of variables


(which we have already done in polar and spherical coordinates) changes by having factor
introduced into the integrand and is called Jacobian. The formula for conversion looks like
this

∫ ∫ 𝒇(𝒙, 𝒚)𝒅𝑨 ⁡⁡⁡ = ⁡ ∫ ∫ 𝑓 (𝑔(𝑢, 𝑣), ℎ(𝑢, 𝑣))⁡𝐽⁡𝑑𝑢𝑑𝑣


ℝ 𝑆

𝜕𝑥 𝜕𝑦
𝜕(𝑥, 𝑦)
𝐽 =⁡ = ⁡ |𝜕𝑢 𝜕𝑢|
𝜕(𝑢, 𝑣) 𝜕𝑥 𝜕𝑦
𝜕𝑣 𝜕𝑣
Note that the region 𝑅 is converted to a region 𝑆. For example, the Jacobian of the
transformation from Cartesian to plane polar coordinates
In general, in plane polar coordinates, 𝑥⁡ = 𝑟𝑐𝑜𝑠𝜃, 𝑦 = 𝑟𝑠𝑖𝑛𝜃

𝜕𝑥 𝜕𝑦
𝐽 = ⌈ 𝜕𝑟 𝜕𝑟 ⌉ = 𝜕(𝑥. 𝑦) = |𝑐𝑜𝑠𝜃 −𝑟𝑠𝑖𝑛𝜃
| = ⁡⁡𝑟
𝜕𝑥 𝜕𝑦 𝜕(𝑟, 𝜃) 𝑠𝑖𝑛𝜃 𝑟𝑐𝑜𝑠𝜃
𝜕𝜃 𝜕𝜃
The element of area is therefore
𝑑𝐴⁡ = ⁡𝑑𝑥⁡𝑑𝑦⁡ = ⁡𝑟⁡𝑑𝑟⁡𝑑𝜃

𝛽 ℎ(𝜃)
∫ ∫ 𝒇(𝒙, 𝒚)𝒅𝑨 = ⁡ ∫ ∫ 𝑓(𝑟𝑐𝑜𝑠𝜃⁡, 𝑟𝑠𝑖𝑛𝜃)𝑟⁡𝑑𝑟⁡𝑑𝜃
𝛼 𝑔(𝜃)
𝑫
131
Example 6
By transforming into polar coordinates evaluate the integral

𝒂 √𝒂𝟐 −𝒙𝟐
∫ ∫ (𝒙𝟐 + ⁡ 𝒚𝟐 )𝒅𝒚𝒅𝒙
𝟎 𝟎

Solution
The region is shown
𝑥 = 𝑟𝑐𝑜𝑠𝜃⁡, 𝑦 = ⁡𝑟𝑠𝑖𝑛𝜃
𝜕𝑥 𝜕𝑦
𝜕(𝑥. 𝑦) 𝜕𝑟 ⌉ = |𝑐𝑜𝑠𝜃 −𝑟𝑠𝑖𝑛𝜃
𝐽= = ⁡ ⌈ 𝜕𝑟 |
𝜕(𝑟, 𝜃) 𝜕𝑥 𝜕𝑦 𝑠𝑖𝑛𝜃 𝑟𝑐𝑜𝑠𝜃
𝜕𝜃 𝜕𝜃
= 𝑟(𝑐𝑜𝑠 2 𝜃 + ⁡ 𝑠𝑖𝑛2 𝜃) = 𝑟

Then 𝑑𝑥⁡𝑑𝑦⁡ = ⁡𝑟⁡𝑑𝑟⁡𝑑𝜃


𝑎 √𝑎2 −𝑥 2 𝜋/2 𝑎
2 2
∫ ∫ 𝑥 + ⁡ 𝑦 𝑑𝑦𝑑𝑥 = ⁡ ∫ ∫ 𝑟 2 ⁡𝑟⁡𝑑𝑟𝑑𝜃
0 0 𝜃=0 𝑟=0
𝜋/2 4 𝜋/2 4
𝑟 𝑎 𝑎
=∫ | 𝑑𝜃 = ⁡ ∫ 𝑑𝜃
𝜃=0 4 0 𝜃=0 4

𝜋𝑎4
=
8

132
7

133
8

Properties of Double Integral


1. ∫ ∫𝐷 𝐾⁡𝑓(𝑥, 𝑦)𝑑𝑥⁡𝑑𝑦 = 𝐾 ∫ ∫𝐷 𝑓(𝑥, 𝑦)𝑑𝑥𝑑𝑦 (𝐹𝑜𝑟⁡𝑎𝑛𝑦⁡𝑐𝑜𝑛𝑠𝑡𝑎𝑛𝑡⁡𝐾)

2. ∫ ∫𝐷(𝑓(𝑥, 𝑦) ± 𝑔(𝑥, 𝑦))𝑑𝑥𝑑𝑦 = ⁡ ∫ ∫𝐷 𝑓(𝑥, 𝑦)𝑑𝑥𝑑𝑦 ± ⁡ ∫ ∫𝐷 𝑔(𝑥, 𝑦)𝑑𝑥𝑑𝑦

3. ∫ ∫𝐷 𝑓(𝑥, 𝑦)𝑑𝑥𝑑𝑦 = ∫ ∫𝐷 𝑓(𝑥, 𝑦)𝑑𝑥𝑑𝑦⁡ ±⁡ ∫ ∫𝐷 𝑓(𝑥, 𝑦)𝑑𝑥𝑑𝑦


1 2

Where the domain 𝐷 is divided into two domains 𝐷1 ⁡𝑎𝑛𝑑⁡𝐷2 without common interior
points, and a function 𝑓(𝑥, 𝑦) is continuous at all points of 𝐷

134
Evaluation Double Integral
If the function 𝑓(𝑥, 𝑦) of two variables continuous on the domain 𝐷, then by successive
integration with respect to each variable one can evaluate certain double integrals using
elementary calculus.
Theory I
If 𝑓(𝑥, 𝑦) is continuous on rectangular region 𝑅:
𝑎1 ⁡ ≤ ⁡𝑥⁡ ≤ ⁡ 𝑎2 ,⁡⁡⁡⁡⁡⁡𝑏1 ⁡ ≤ ⁡𝑦⁡ ≤ ⁡ 𝑏2 ⁡⁡ then
𝑏2 𝑎2 𝑎2 𝑏2
∫ ∫ 𝑓(𝑥, 𝑦)𝑑𝑥𝑑𝑦 = ⁡ ∫ ∫ 𝑓(𝑥, 𝑦)𝑑𝑥𝑑𝑦⁡ = ⁡ ∫ ∫ 𝑓(𝑥, 𝑦)𝑑𝑥𝑑𝑦⁡
𝑏1 𝑎1 𝑎1 𝑏1
𝐷

Example 9
Calculate the double integral ∫ ∫𝑫 𝒇(𝒙, 𝒚)𝒅𝒙𝒅𝒚⁡for 𝒇(𝒙, 𝒚) ⁡ = ⁡𝟏⁡ − 𝟔𝒙𝒚⁡and 𝑫 id
bounded by 𝟎⁡ ≤ ⁡𝒙⁡ ≤ ⁡𝟐⁡, −𝟏 ≤ ⁡𝒚⁡ ≤ ⁡𝟏

∫ ∫ 𝑓(𝑥, 𝑦)𝑑𝑥𝑑𝑦
𝐷
1 2

= ⁡ ∫ ( ∫(1 − 6𝑥𝑦)𝑑𝑥 ) 𝑑𝑦
𝑦=−1 𝑥=0

1
𝑥=2
= ∫ [𝑥⁡ − ⁡3𝑥 2 𝑦] 𝑑𝑦
𝑥=0
𝑦=−1

= ∫ (2 − 12𝑦)𝑑𝑦
𝑦=−1
𝑦=1
= ⁡ [2𝑦⁡ − ⁡6𝑦 2 ]
𝑦 = −1
= ⁡ −4⁡–⁡(−2⁡ − 6) ⁡ = 4
Or
𝑥=2 𝑦⁡=1

∫ ∫ 𝑓(𝑥, 𝑦)𝑑𝑥𝑑𝑦 ⁡ = ⁡ ∫ ( ∫ (1⁡ − ⁡6𝑥𝑦)𝑑𝑦)⁡𝑑𝑥⁡


𝐷 𝑥=0 𝑦⁡=−1
135
𝑥=2

= ∫ [𝑦⁡ − ⁡3𝑥𝑦 2 ] 𝑑𝑥
𝑥=0
2
𝑥=2
= ∫ 2𝑑𝑥⁡ = ⁡ [2𝑥]
𝑥=0
𝑥=0

= ⁡4⁡– ⁡0⁡ = ⁡4

Theory II
Let 𝑓(𝑥, 𝑦) be continuous on a region 𝑅:
1. if 𝑅 is defined by 𝑦1 (𝑥) ≤ ⁡𝑦⁡ ≤ ⁡ 𝑦2 (𝑥),⁡⁡⁡⁡𝑎1 ⁡ ≤ ⁡𝑥⁡ ≤ ⁡ 𝑎2 ⁡ with 𝑦1 (𝑥)and 𝑦2 (𝑥)⁡ are
continuous on ⁡[⁡𝑎1 ⁡, 𝑎2 ]then
𝑥⁡=⁡𝑎2 𝑦⁡=⁡𝑦2 (𝑥)

∫ ∫ 𝑓(𝑥, 𝑦)𝑑𝑥𝑑𝑦 = ⁡ ∫ ∫ 𝑓(𝑥, 𝑦)⁡𝑑𝑦𝑑𝑥⁡


𝐷 𝑥⁡=⁡𝑎1 𝑦=⁡𝑦1 (𝑥)

2. if 𝑅 is defined by 𝑏1 ⁡ ≤ ⁡𝑦⁡ ≤ ⁡ 𝑏2 ⁡,⁡⁡⁡⁡⁡⁡𝑥1 (𝑦) ≤ ⁡𝑥⁡ ≤ ⁡ 𝑥2 (𝑦) with 𝑥1 (𝑦)⁡𝑎𝑛𝑑⁡𝑥2 (𝑦)⁡ are
continuous on [𝑏1 ⁡, 𝑏2 ]⁡then
𝑦⁡=⁡𝑏2 𝑥⁡=⁡𝑥2 (𝑦)

∫ ∫ 𝑓(𝑥, 𝑦)𝑑𝑥𝑑𝑦 = ⁡ ∫ ∫ 𝑓(𝑥, 𝑦)⁡𝑑𝑥𝑑𝑦⁡


𝐷 𝑦⁡=⁡𝑏1 𝑥=⁡𝑥1 (𝑦)

Example 10
Calculate the double integral ∫ ∫𝑫 𝒇(𝒙, 𝒚)𝒅𝒙𝒅𝒚 for 𝒇(𝒙, 𝒚) ⁡ = ⁡𝟑⁡ − 𝒙⁡ − 𝒚 and 𝑫 is
bounded 𝟎⁡ ≤ ⁡𝒙⁡ ≤ ⁡𝟏 , 𝟎⁡ ≤ ⁡𝒚⁡ ≤ ⁡𝒙
Solution
𝑥=1
𝑦=𝑥
∫ ∫ 𝑓(𝑥, 𝑦)𝑑𝑥𝑑𝑦 ⁡ = ⁡ ∫ (∫ (3⁡ − ⁡𝑥⁡ − ⁡𝑦)⁡𝑑𝑦⁡)⁡𝑑𝑥
𝑦=0⁡
𝐷 𝑥=0

136
𝑥=1
𝑦2
= ∫ (3𝑦⁡ − 𝑥𝑦⁡ − ⁡ )⁡𝑑𝑥
2
𝑥=0
𝑥=1
3⁡𝑥 2
= ∫ (3𝑥⁡ − ⁡ )⁡𝑑𝑥
2
𝑥=0

3𝑥 2 𝑥3
= ⁡⁡ − ⁡⁡ ⁡ = ⁡1
2 2

Applications of Double Integral


The double integrals have a variety of applications in Engineering sciences and physics.
Here, we will mention some of its applications:

i. Area between two curves


The area between two curves (say domain 𝐷) in
the 𝑥⁡𝑦 plane is equal (numerically) to the
volume of the cylinder of unit altitude with ′𝐷′ as
a base i.e. 𝑓(𝑥, 𝑦) = 1 then
Vertical Slip

𝐴𝑟𝑒𝑎 = ∫ ∫ 𝑑𝐴 ⁡ = ⁡ ∫ ∫ 𝑑𝑦𝑑𝑥
𝐷 𝐷

Horizontal Slip

𝐴𝑟𝑒𝑎 = ∫ ∫ 𝑑𝐴 ⁡ = ⁡ ∫ ∫ 𝑑𝑥𝑑𝑦
𝐷 𝐷

11

137
12

138
ii. Mass and centre of mass

if the function 𝑓(𝑥, 𝑦) interpreted as the local density per unit area, then Mass

𝑚 = ∫ ∫ 𝑑𝑥𝑑𝑦
𝐷

and the centre of the mass (centre of gravity) (𝑥, 𝑦) of a thin plate is given by

𝑚⁡𝑥 = ∫ ∫ 𝑥⁡𝑓(𝑥, 𝑦)𝑑𝑥𝑑𝑦


𝐷

and

𝑚⁡𝑦 = ⁡ ∫ ∫ 𝑦⁡𝑓(𝑥, 𝑦)𝑑𝑦𝑑𝑥


𝐷

where (𝑥, 𝑦) is the coordinated of the Center of the mass

Example 13
Evaluate the mass of a thin plate bounded by the straight
lines
𝒚⁡ = ⁡𝒙⁡,⁡⁡⁡⁡𝒚 = 𝟎⁡,⁡⁡⁡⁡𝒙⁡ = ⁡𝟏⁡𝒂𝒏𝒅⁡𝒙⁡ = ⁡𝟒 given that its
mass density (𝒙𝟐 ⁡ + ⁡ 𝒚𝟐 )

The region 𝐷 can be described by the inequalities 1 ≤ 𝑥 ≤ 4, 𝑎𝑛𝑑⁡0 ≤ 𝑦 ≤ ⁡⁡𝑥⁡using


vertical strip (we first integrate with respect to 𝑦 and then with respect to 𝑥).
Thus, the integral becomes
139
𝑦=𝑥
𝑥=4
𝑚 = ∫ ∫(𝑥 3 + ⁡ 𝑦 2 )𝑑𝑥𝑑𝑦 = ⁡ ∫ ( ∫ (𝑥 3 + ⁡ 𝑦 2 )𝑑𝑦) 𝑑𝑥
𝑥=1
𝐷 𝑦=0

𝑥=4
4
𝑥3
=∫ (𝑥 + ⁡ ) 𝑑𝑥 = 225.85⁡𝑢𝑛𝑖𝑡⁡𝑜𝑓⁡𝑚𝑎𝑠𝑠
𝑥=1 3

Note that, here we cannot use the horizontal strip because of change of boundaries as
shown in the figure

The region 𝐷 can be described by the inequalities


𝑦⁡ ≤ 𝑥⁡⁡ ≤ 4⁡𝑎𝑛𝑑⁡0⁡ ≤ 𝑦⁡ ≤ 𝑥⁡ using the strip number (I) while using strip number (II)
Region 𝐷 can be described by the inequalities
1⁡ ≤ 𝑥⁡⁡ ≤ 4⁡𝑎𝑛𝑑⁡0⁡ ≤ 𝑦⁡ ≤ 𝑥

we can observe that the boundaries of 𝑥 in strip I is different from the boundaries of x in
strip II. But we can divide the region 𝐷 into two regions (triangle and rectangle) but in this
case, the double integral equal the integral in two regions. Moreover, the boundaries are
𝑦⁡ ≤ 𝑥⁡⁡ ≤ 4⁡𝑎𝑛𝑑⁡1 ≤ 𝑦⁡ ≤ 4⁡⁡𝑖𝑛⁡𝑡ℎ𝑒 triangle 1⁡ ≤ 𝑥⁡⁡ ≤ 4⁡𝑎𝑛𝑑⁡0⁡ ≤ 𝑦⁡ ≤ 1⁡ in the
rectangle

𝑚 = ∫ ∫(𝑥 3 + ⁡ 𝑦 2 )𝑑𝑥𝑑𝑦 =⁡
𝐷

= ⁡𝑖𝑛𝑡𝑒𝑔𝑟𝑎𝑡𝑖𝑜𝑛⁡𝑖𝑛⁡𝑡𝑟𝑖𝑎𝑛𝑔𝑙𝑒⁡
+ ⁡𝑖𝑛𝑡𝑒𝑔𝑟𝑎𝑡𝑖𝑜𝑛⁡𝑖𝑛⁡𝑟𝑒𝑐𝑡𝑎𝑛𝑔𝑙𝑒⁡
= ⁡225.85⁡⁡𝑢𝑛𝑖𝑡⁡𝑜𝑓⁡𝑚𝑎𝑠𝑠

Example 14

Find the Center of the mass of a thin plate bounded by the parabola 𝒚 = 𝟔𝒙 − 𝒙𝟐 and
the straight line 𝒚⁡ = ⁡𝒙, given that it has a mass density 𝒇(𝒙, 𝒚) ⁡ = ⁡𝒑(𝒙, 𝒚)⁡=1.

140
𝑚⁡ = ∫ ∫ ⁡𝑓(𝑥, 𝑦)𝑑𝑥𝑑𝑦 =⁡ ∫ ∫(1)⁡𝑑𝑥𝑑𝑦
𝐷 𝐷

5 6𝑥−𝑥 2 5
125
𝑚 = ⁡ ∫ 𝑑𝑥 ∫ 𝑑𝑦 = ⁡ ∫(5𝑥 − 𝑥 2 )𝑑𝑥 =
6
0 𝑥 0

5 6𝑥−𝑥 2

𝑚⁡𝑥 = ∫ ∫ 𝑥⁡𝑓(𝑥, 𝑦)𝑑𝑥𝑑𝑦 = ⁡ ∫ 𝑥𝑑𝑥⁡ ∫ 𝑑𝑦


0 𝑥
𝐷

125 125 1
⁡⁡𝑥 = = ➔⁡⁡𝑥 =
6 12 6
5 6𝑥−𝑥 2

𝑚⁡𝑦 = ⁡ ∫ ∫ 𝑦⁡𝑓(𝑥, 𝑦)𝑑𝑦𝑑𝑥 =⁡ ∫ 𝑑𝑥⁡ ∫ 𝑦𝑑𝑦


𝐷 0 𝑥

125 625 25
⁡⁡𝑦 = = ➔⁡⁡𝑥 =
6 6 8

iii. Moment of Inertia


The moment of inertia of a particle with respect
to any axis ′𝐴′ is the product of its mass ′𝑚′ and
the square distance ′𝑟′ of the point ′𝑝′ from the
axis ′𝐴′, see the shown figure

𝐼⁡ = ⁡𝑚⁡. 𝑟 2

In the same way, we define the moment of inertia


about the 𝑥⁡𝑎𝑥𝑖𝑠 of a mass distributed with
density 𝑓(𝑥, 𝑦) ⁡ = ⁡𝑝(𝑥, 𝑦) through a region 𝐷 by:
(see the figure)

141
𝐼𝑥 = ∫ ∫ 𝑦 2 ⁡𝑓(𝑥, 𝑦)𝑑𝑦𝑑𝑥
𝐷

𝐼𝑦 = ∫ ∫ 𝑥 2 ⁡𝑓(𝑥, 𝑦)𝑑𝑥𝑑𝑦
𝐷

also the polar moment of inertia about the origin


‘𝑂’ is given by.

𝐼0 = ∫ ∫ 𝑟 2 ⁡𝑓(𝑥, 𝑦)𝑑𝑥𝑑𝑦 = ∫ ∫(𝑥 2 + ⁡ 𝑦 2 )𝑓(𝑥, 𝑦)𝑑𝑥𝑑𝑦 = ⁡ 𝐼𝑥 + ⁡ 𝐼𝑦


𝐷 𝐷

Example 15
Find the moment of inertia of a mass bounded by
the first of parabola 𝒚𝟐 = ⁡𝟏⁡ − ⁡𝒙, 𝒑𝒐𝒔𝒊𝒕𝒊𝒗𝒆⁡𝒙 −
⁡𝒂𝒙𝒊𝒔⁡𝒂𝒏𝒅⁡𝒕𝒉𝒆⁡𝒑𝒐𝒔𝒊𝒕𝒊𝒗𝒆⁡𝒚 − ⁡𝒂𝒙𝒊𝒔 about:
a) x-axis
b) y-axis
c) the origin
Consider 𝑓(𝑥, 𝑦) = ⁡𝜌(𝑥, 𝑦) = ⁡1 then

𝐼𝑥 = ∫ ∫ 𝑦 2 ⁡𝑓(𝑥, 𝑦)𝑑𝑦𝑑𝑥 =⁡⁡ ∫ ∫ 𝑦 2 ⁡𝑑𝑦𝑑𝑥


𝐷 𝐷

1 √1−𝑥 1
1 3
𝐼𝑥 = ∫ 𝑑𝑥⁡ ∫ 𝑦 2 ⁡𝑑𝑦 = ⁡∫(1 − 𝑥)2 𝑑𝑥
3
0 0 0
2
= ⁡⁡𝑢𝑛𝑖𝑡⁡𝑜𝑓⁡𝑚𝑜𝑚𝑒𝑛𝑡
15

𝐼𝑦 = ∫ ∫ 𝑥 2 ⁡𝑓(𝑥, 𝑦)𝑑𝑥𝑑𝑦 = ∫ ∫ 𝑥 2 ⁡𝑑𝑥𝑑𝑦


𝐷 𝐷

1 √1−𝑥 1
16
𝐼𝑦 = ∫ 𝑑𝑥⁡ ∫ 𝑥 2 ⁡𝑑𝑦 = ⁡ ∫ 𝑥 2 √1 − 𝑥𝑑𝑥 = ⁡𝑢𝑛𝑖𝑡⁡𝑜𝑓⁡𝑚𝑜𝑚𝑒𝑛𝑡
105
0 0 0

142
2 16 2
𝐼0 ⁡ = ⁡ 𝐼𝑥 + ⁡ 𝐼𝑦 = + = ⁡⁡𝑢𝑛𝑖𝑡⁡𝑜𝑓⁡𝑚𝑜𝑚𝑒𝑛𝑡
15 105 7

2. Volume Integral (Triple Integral)


The volume integrals have numerous applications
such as, mass, centre of mass (centroid), volume of a
region, moment of inertia and electrostatic potential.
We will first briefly outline the definition of the triple
integral

𝑉 = ∭ 𝑓(𝑥, 𝑦, 𝑧)𝑑𝑥⁡𝑑𝑦⁡𝑑𝑧

The function 𝑓(𝑥, 𝑦, 𝑧) is defined over a-closed region


𝑉 of the three-dimensional space. It is assumed that 𝑉
is bounded. We divide the region 𝑉 into 𝑛 sub
regions ∆𝑉1 , ∆𝑉2 , ∆𝑉3 , … … ..

Let (𝑥𝑘 . 𝑦𝑘 . 𝑧𝑘 ) be an arbitrary point in the sub region


and , 𝑓(𝑥𝑘 . 𝑦𝑘 . 𝑧𝑘 ) the value of 𝑓 at that point. We
form the sum
𝑁

𝐹 = ∑ 𝑓(𝑥𝑘 , 𝑦𝑘 ⁡, 𝑧𝑘 )∆𝑣𝑘
𝑘=1
𝑁

= ∑ 𝑓(𝑥𝑘 , 𝑦𝑘 ⁡, 𝑧𝑘 )∆𝑥∆𝑦∆𝑧
𝑘=1

there are some conditions on 𝑉 which we will not discuss here Suppose that the function
𝑓(𝑥𝑘 , 𝑦𝑘 ⁡, 𝑧𝑘 )⁡is continuous throughout V and on its boundary, then the sum above has a
limit as ∆𝑉1 , ∆𝑉2 , ∆𝑉3 , … … .. approaches zero

This limit is called the Riemann triple integral of


𝑓(𝑥𝑘 , 𝑦𝑘 ⁡, 𝑧𝑘 )𝑜𝑣𝑒𝑟⁡𝑣
𝑛
∭ 𝑓(𝑥, 𝑦, 𝑧)𝑑𝑥⁡𝑑𝑦⁡𝑑𝑧 = lim ∑ 𝑓(𝑥𝑘 , 𝑦𝑘 ⁡, 𝑧𝑘 )∆𝑥∆𝑦∆𝑧
𝑁→⁡∞ 𝑘=1

143
Example 16

Evaluate the volume integral ∫ ∫𝑽 ∫ 𝒙𝟐 ⁡𝒚⁡𝒅𝒗 where 𝑽 is the closed region bounded by the
coordinate planes 𝒂𝒏𝒅⁡𝒙 = 𝟏⁡,⁡⁡⁡⁡⁡𝒚 = 𝟏, 𝒂𝒏𝒅⁡𝒛 = 𝟏
𝑉 is a cube bounded by the planes
𝑥 = 0, 𝑥 = 1, 𝑦 = 0,
𝑦 = 1, 𝑧 = 0, 𝑧 = 1.
Thus, we integrate
𝑥 = 0⁡𝑡𝑜⁡𝑥⁡ = 1, 𝑦 = 0⁡⁡𝑡𝑜⁡𝑦⁡ = 1, 𝑧 = 0⁡𝑡𝑜⁡𝑧
= 1⁡𝑡ℎ𝑒𝑛,

𝑥=1 𝑦=1 𝑧=1

∫ ∫ ∫ 𝑥 2 𝑦⁡𝑑𝑣 = ⁡ ∫ ∫ ∫ 𝑥 2 𝑦⁡𝑑𝑧⁡𝑑𝑦⁡𝑑𝑥
𝑥=0 𝑦=0 𝑧=0
𝑉

𝑥=1 𝑦=1

= ∫ ∫ 𝑥 2 𝑦⁡𝑑𝑦⁡𝑑𝑥
𝑥=0 𝑦=0

𝑥=1
1 2 1
= ∫ 𝑥 𝑑𝑥 = ⁡⁡𝑢𝑛𝑖𝑡⁡𝑜𝑓⁡𝑣𝑜𝑙𝑢𝑚𝑒
2 6
𝑥=0

Example 17
Evaluate the volume integral
∫ ∫ ∫(𝒙𝒛 + 𝒚𝒛)𝒅𝒗
𝑽

where 𝑽 is the closed region bounded by the


coordinate planes and 𝒙 = 𝟐, 𝒚 = 𝟑, 𝒂𝒏𝒅⁡𝒛 =
−𝒙 + 𝟒.

144
𝒚=𝟑 𝒙=𝟐 𝒛=−𝒙+𝟒

∫ ∫ ∫(𝒙𝒛 + 𝒚𝒛)𝒅𝒗 = ∫ ∫ ∫ ⁡(𝒙𝒛 + 𝒚𝒛)𝒅𝒛⁡𝒅𝒚⁡𝒅𝒙⁡⁡


𝒚=𝟎 𝒙=𝟎 𝒛=𝟎
𝑽
𝑦=3 𝑥=2
𝑧2
= ∫ ∫ [(𝑥 + 𝑦) ] 𝑑𝑥⁡𝑑𝑦
2
𝑦=0 𝑥=0

𝑦=3
22 28
= ∫ ( + ⁡ ⁡𝑦⁡) 𝑑𝑦 = 64⁡⁡𝑢𝑛𝑖𝑡⁡𝑜𝑓⁡𝑣𝑜𝑙𝑢𝑚𝑒
3 3
𝑦=0

Example 18
Evaluate the triple integral of function (𝟏 − ⁡𝒙𝒚) on the domain bounded by: 𝒙⁡ = ⁡𝒚⁡ =
⁡𝒛⁡ = 𝟎⁡⁡, 𝒙⁡ + 𝒚⁡ + 𝒛 = 𝟏

𝒙=𝟏 𝒚=𝟏−𝒙 𝒛=𝟏−𝒙−𝒚

∫ ∫ ∫(𝟏 − 𝒙𝒚⁡)𝒅𝒗 = ∫ ∫ ∫ ⁡(𝟏 − 𝒙𝒚)𝒅𝒛⁡𝒅𝒚⁡𝒅𝒙⁡⁡


𝒙=𝟎 𝒚=𝟎 𝒛=𝟎
𝑽

𝑥=1 𝑦=1−𝑥

= ∫ ∫ (1 − 𝑥𝑦)(1 − 𝑥 − 𝑦)𝑑𝑦⁡𝑑𝑥
𝑥=0 𝑦=0

𝑥=1
𝑦 2 𝑥𝑦 2 𝑥 2 𝑦 2
= ∫ ⌈𝑦 − 𝑥𝑦 − ⁡ − ⁡ +
2 2 2
𝑥=0
𝑥⁡𝑦 3 𝑦 = 1 − 𝑥
+ ⌉
3 𝑦 = 0⁡
19
= ⁡𝑢𝑛𝑖𝑡⁡𝑜𝑓⁡𝑣𝑜𝑙𝑢𝑚𝑒
120

145
19

20

146
147
Exercises
𝑸𝟏 / Evaluate the following integrals
𝑎 𝑏 𝜋
2 4𝑐𝑜𝑠𝜃
1)⁡⁡⁡∫ ∫ 𝑥𝑦(𝑥 − 𝑦)𝑑𝑥⁡𝑑𝑦
2)⁡⁡⁡⁡⁡⁡⁡⁡∫ ∫ 𝑟 3 𝑑𝜃𝑑𝑟
0 0
0 0
∞ 1 𝜋 𝜋/2
2
3)⁡⁡⁡⁡⁡∫ ∫ 𝑥𝑦⁡𝑒 −𝑥 𝑑𝑦⁡𝑑𝑥 4)⁡⁡⁡⁡⁡⁡∫ ∫ 𝑐𝑜𝑠𝑦⁡𝑒 𝑥+𝑠𝑖𝑛𝑦⁡ 𝑑𝑥⁡𝑑𝑦⁡
0 0 0 0

𝑸𝟐 / Evaluate the integral ∫ ∫𝑫(𝒙𝟐 + ⁡ 𝒚𝟐 )𝒅𝒙𝒅𝒚⁡𝒘𝒉𝒆𝒓𝒆⁡𝑫⁡ ia the triangle bounded by the


lines: 𝒙⁡ = ⁡𝟏, 𝒚⁡ = ⁡𝒙, 𝒚⁡ = ⁡𝟎

𝑸𝟑 / Evaluate the integral ∫ ∫𝑫 𝒚⁡𝒍𝒏⁡𝒙⁡𝒅𝒙𝒅𝒚⁡𝒘𝒉𝒆𝒓𝒆⁡𝑫⁡ is the triangle bounded by the


lines: 𝒙𝒚 = 𝟏, 𝒚 = √𝒙, 𝒙 = 𝟐

𝑸𝟒 / Evaluate the integral ∫ ∫𝑫(𝒙 + 𝒚 + 𝟏)𝒅𝒙𝒅𝒚⁡𝒘𝒉𝒆𝒓𝒆⁡𝑫⁡ ia the triangle bounded by


the lines: 𝒚 = ⁡ 𝒙𝟐 ⁡, 𝒚⁡ = ⁡ −𝒙⁡, 𝒚⁡ = ⁡𝟐

𝑸𝟓 / Find the coordinates of the center of gravity of the domain bounded by 𝑦 2 = 4𝑥 +


4⁡⁡⁡𝑎𝑛𝑑⁡⁡⁡⁡⁡⁡𝑦 2 = ⁡ −2𝑥 + 4

𝑸𝟔 / Find the polar moment of intertia of the area bounded by the lines:
𝑥 𝑦
1. ⁡ + ⁡⁡ ⁡ = 1⁡,⁡⁡⁡𝑥 = 0⁡⁡⁡,⁡⁡⁡𝑎𝑛𝑑⁡𝑦 = 0
𝑎 𝑏
2. 𝑦 = 2√𝑥⁡,⁡⁡⁡⁡⁡⁡⁡𝑥 + 𝑦 = 3⁡, 𝑎𝑛𝑑⁡⁡⁡⁡𝑦 = 0

𝑸𝟕 / Evaluate the following integrals:


2 4𝑥 2 𝑧 5 𝑥 𝑧

1)⁡⁡⁡⁡⁡⁡⁡⁡∫ ∫ ∫ 𝑥⁡𝑑𝑦⁡𝑑𝑧⁡𝑑𝑥 2)⁡⁡⁡⁡⁡⁡⁡⁡⁡∫ ∫ ∫ 𝑥𝑦𝑧⁡⁡𝑑𝑦⁡𝑑𝑧⁡𝑑𝑥⁡


0 0 0 0 0 0

148
𝑸𝟖 / Evaluate the following integrals

∫ ∫ ∫(𝟐𝒙 − 𝒚 − 𝒛⁡)𝒅𝒙⁡𝒅𝒚⁡𝒅𝒛
𝑫

(𝒙⁡, 𝒚⁡, 𝒛)⁡⁡⁡⁡⁡⁡⁡⁡⁡⁡⁡⁡⁡⁡⁡⁡⁡⁡⁡⁡⁡⁡⁡⁡⁡⁡𝟎 ≤ 𝒙⁡ ≤ 𝟏


Where 𝑫 = {
𝟎⁡ ≤ 𝒚⁡ ≤ ⁡ 𝒙𝟐 ⁡⁡⁡⁡⁡⁡⁡⁡⁡⁡⁡⁡𝟎⁡ ≤ 𝒛⁡ ≤ 𝒙 + 𝒚

𝑸𝟗 / Find the volume of the pyramid bounded by:


𝒙 𝒚 𝒛
𝒙 = 𝟎⁡, 𝒚⁡ = ⁡𝟎⁡, 𝒛⁡ = ⁡𝟎,⁡ and +⁡ +⁡ = 𝟏
𝒂 𝒃 𝒄

𝑸𝟏𝟎 / Calculate the volume of the body bounded by the surfaces:

𝒛⁡ = ⁡𝟒⁡–⁡𝒙𝟐 ⁡, 𝟐𝒙⁡ + ⁡𝒚⁡ = ⁡𝟒⁡⁡, 𝒙⁡ = ⁡𝟎⁡, 𝒚⁡ = ⁡𝟎⁡, 𝒛⁡ = ⁡𝟎

𝑸𝟏𝟏 / Calculate the triple of the function on the domain bounded by the lines:
𝒛⁡ = ⁡𝒙𝒚⁡⁡, 𝒚⁡ = ⁡𝒙⁡, 𝒙⁡ = ⁡𝟏⁡, 𝒙⁡ = ⁡𝒚⁡ = ⁡𝒛⁡ = ⁡𝟎

Additional Exercises

149
150
Chapter 4
Infinite Sequences and Series

Objectives
This chapter is dedicated to assist the student to understand and to learn the basic topics
of infinite sequences and series and their applications. These topics include (importance,
definitions, sequences, series, series and summation notation, convergence and
divergence tests of series, power series). The main objectives of this chapter are listed
below to learn the students how to:
i) use infinite sequences and series in mathematical modelling and structure.
ii) use infinite sequences and series to prove or to reaches results related with other
sciences.
iii) use sequences and series in many fields and different engineering applications such
as: automatic control, structure analysis, system analysis, signal analysis,
electromagnetic fields, fluid mechanics, solid mechanics, mechanics of materials,
and many other fields and applications.

Importance and definition


The Infinite sequences are considered an introduction to the infinite series. Sequences
are considered as a main and general form, where the infinite series are based on form,
concepts, and properties of sequences. Otherwise, the computations of the infinite
sequences and series and the understanding of its concepts play an important role in
constructing mathematical models and systems analysis. Sequences and series have many
miscellaneous applications which are used to prove or to reaches results related with other
branches of sciences. In this chapter some basic definitions, tests for convergence, and
some examples are discussed and reported. Finally, the main types of series are included
with illustrative examples. Series have great importance in numerous applications in
Engineering and scientific fields. These fields include; fluid mechanics, solid mechanics,
electrical Engineering, commerce and economics, … and many other fields and
miscellaneous applications.

151
4.1 Basic Definitions
A sequence is a function that calculates an ordered list. Instead of using the function 𝑦 =
𝑓(𝑥), the form 𝑢𝑛 = 𝑓(𝑛) is customary used to express the sequence, where the natural
number 𝑛 is used instead of the real variable x. The elements in the range of a sequence
𝑢1 , 𝑢2 , 𝑢3 … are called the terms of the sequence. The elements of both the domain and
the range of a sequence are expressed in ordered form. The first term is determined by
putting 𝑛 = 1, the second term is determined by putting 𝑛 = 2, and so on. The general
term or the nth term of the sequence is written 𝑢𝑛 .

Infinite sequence

An infinite sequence has the set all-natural numbers as its domain which is written
{1, 2, 3, … 𝑛}, where 𝑛 is a natural number. For examples the set of natural numbers
multiples {2, 4, 6, 8 … }⁡is infinite, but the sequence of days in June {1, 2, 3 … ,30} is finite.
An infinite sequence is a function whose domain is the set of positive integers which is

denoted by the formula u n n =1 = u1, u 2 , u 3 ,u n ,  .

Example 1

 1   1 1 
The sequences  
u n n =1 =   = 1, , , ,
 n n =1  2 3 

  n  1 2 3 
 
u n n =1 =  =  , ,  ,
 n + 1n =1  2 3 4 

u n n =1 = 3n − 5n =1 = −2, 1, 4, ,

u n n =1 = (−1)n + (n + 1)n =1 = −3, −2, −5,


Particular cases of sequences


I. Arithmetic sequences
A sequence in which the difference between any two successive terms d is constant. 𝑑 =
⁡⁡𝑢𝑛+1 ⁡ − 𝑢𝑛 . For example, the sequence 6, 9, 12, 15 … is arithmetic, where the common
difference 𝑑 = 3.
152
nth term of an arithmetic sequence
𝑎𝑛 = 𝑢1 + ⁡ (𝑛 − 1)⁡𝑑
Sum of the first n terms of an arithmetic series
For any arithmetic sequence of first term 𝑢1 and common difference 𝑑, the sum of the first
n
𝑛 terms is given by the relation S n = (u1 + u n )
2
For example, the sum of six term of the sequence {2, 4, 6, 8, … } is calculated as
n 6
S n = (u1 + u n ) = (2 + 12) = 42
2 2
Geometric sequence
A sequence in which the ratio between any two successive terms 𝑟 is constant.
u
i.e., the common ratio r = n +1 ,⁡𝑟 > 0. For example, the sequence 2, 4, 6, 8 … is geometric
un
of common ratio 2.

nth term of a geometric sequence


𝑢𝑛 = 𝑢1 ⁡𝑟 𝑛−1
for example, the six term of the geometric sequence {2,4,6,8,32,64, … } is computed
𝑢𝑛 = 𝑢1 ⁡𝑟 𝑛−1 = ⁡2⁡. 26−1 ⁡ = ⁡2. 25 = ⁡2. 32⁡ = 64

Infinite geometric sequence


If the common ratio −1 < 𝑟 < 1, then the sum of this sequence is finite and the sequence
is said to be convergence. This limit can be calculated from the formula
u
S  = 1 , where − 1  r  1
1− r
For examples the geometric sequence  
1 1 1 1
1, , , ,..., n −1 ,... has common ratio 𝑟 = 1/2,
 2 4 8 2 
so that it diverges.
If the common ratio 𝑟1, the sequence diverges and the sum of the sequence has no limit.
 n

For examples the geometric sequence 3,9,27,81,...,3 ,... has common ratio 𝑟 = 3, so
that it diverges.

Bounded sequence
A sequence uu is said to be bounded if there exist two positive real numbers 𝑃⁡𝑎𝑛𝑑⁡𝑄 in
such a way that 𝑃⁡⁡𝑢𝑛 ⁡⁡𝑄 for all values of 𝑛.

153
Example 2

If the sequence 
3 5 7 
 , , ,... is bounded between the two numbers 𝑎⁡𝑎𝑛𝑑⁡𝑏, find the
2 4 6 
general term 𝑢𝑛 , then calculate each number.
The general term is written

  2n + 1  3 5 7 
 
u n n =1 =  =  , , ,...
 2n n =1  2 4 6 
2n + 1 3
when 𝑛 = 1  a= =
2n 2
2n + 1
when 𝑛→  b = Lim =1
n → 2n
 2n + 1  3
i.e., 1  
 2n  2
Example 3
Show that the sequence {2, 4, 6, … } is unbounded.
𝑢𝑛 = 2𝑛
2 2n 
i.e., the sequence is unbounded.
Convergent and divergent sequences
The limit
Lim u n = a , a  0
n →
If 𝑎𝑅, the sequence is convergent.
For example, considering the sequence
 1
n   1 n 
u n = 2 +  −  which has the limit Lim 2 +  −   = 2
 2 n →
  2  
then this sequence is convergent.

converges to (1) since Lim 


n n 
Also, the sequence   =1 ,
n +1 n →  n + 1

Otherwise, the sequence is said to be divergent if Lim u n =


n →
For example, the sequence un=(2+3n) diverges which has the limit Lim u n = .
n →

154
4.2 Series and summation notation
The sum of a sequence is called a series and is expressed by using summation notation.
n
The finite series is written S n = u1 + u 2 + u 3 + ... + u n =  u k where the letter 𝑘 is called the
k =1
index of summation.

Otherwise, the infinite series is the sum of an infinite sequence 𝑢𝑛 , which takes the form

 u n = u 0 + u1 + u 2 + ... + u n + ... .
n =0
Convergence and divergence of series
If the series defines the N-th partial sum of a convergent sequence, the series is said to be
convergent where 𝑆𝑛 = ⁡ 𝑢0 + 𝑢1 + 𝑢2 + ⋯ + 𝑢𝑛 . Otherwise, the series is said to be
divergent if the sequence diverges. Also, a series is said to be absolutely convergent

if  un converges and conditionally convergent if it does converge but not absolutely (if
n =0
the series contains only positive terms terms).

Particular series
Geometric series: It is the sum of partial terms of a geometric sequence. Considering the
series, 𝑎 + 𝑎𝑟 + 𝑎𝑟 2 + 𝑎𝑟 3 + ⋯ + 𝑎𝑟 𝑛 , where 𝑎 is the first term and 𝑟 is the common
ratio. The sum of the first 𝑛 terms of this series is written
a (1 − r n )
Sn =
1− r
a (1 − r 2 ) a (1 − r )(1 + r)
For example: S 2 = = = a + ar ,
1− r 1− r
a (1 − r 4 )
S4 = = a (1 + r + r 2 + r 3 ) = a + ar + ar 2 + ar 3
1− r
There are different cases according to the value of the common ratio r.

Case 1
a
If the common ratio r  1 the series converges to where 𝑟 𝑛 ⁡→0⁡, 𝑛→ and
1− r
a
Lim S n = and the series converges.
n → 1− r
155
Case 2
If the common ratio r  1 the series diverges where; 𝑟 𝑛 ⁡→0⁡, 𝑛→ , Lim S n =  , and
n →
the series diverges. If r=1, the series takes the form Sn=an, then
Lim S n =  , and the series diverges.
n →
Case 3
o when n is even
𝐼𝑓⁡𝑟 = −1, the series takes the form {𝑎 − 𝑎 + 𝑎 − 𝑎 + ⋯ }, S n =  ,
a when n is odd
Sn has no limit, and the series diverges. This means that, the geometric series is convergent
if r  1 , and it diverges if r  1 .
Example 4

Show that the series  


1 1 1
 + + + ... is a geometric convergent series.
2 4 8 
1
The series is a geometric series with 𝑎 = 1/2⁡𝑎𝑛𝑑⁡𝑟 = 1/2, u n = n .
2

1 1/ 2
then S n =  n = = 1 , and the series is convergent.
n =1 2 1 − (1/ 2)
Example 5

Show that the series  


4 8 16
 + + + ... is a geometric convergent series.
 3 9 27 
n
2
The series is a geometric series with 𝑎 = 4/3, 𝑟 = 2/3 < 1, u n = 2   ,
3

1 1/ 2
then S n =  = = 1 , and the series is convergent.
n =1 2 n
1 − (1/ 2)
Example 6

Show that the series  


9 27
3 + + + ... is a geometric divergent series.
 2 4 
n
2
The series is a geometric series with 𝑎 = 3, 𝑟 = 3/2 > 1, u n = 2   ,
3
 n
then S n =  2   =  , and the series is divergent.
3
n =1  2 
2. P- series

156

1 1 1 1
Sn =  p
= 1 + p + p + p + ...
n =1 n 2 3 4
P-series has three cases for convergence and divergence:
Case 1: If |𝑃| > ⁡1 the series converges,
Case 2: If |𝑃| < ⁡1 the series diverges,
Case 3: If |𝑃| = 1 the series diverges, and is called harmonic.

1 1 1
For example, the series S n =  = 1 + + + ... is harmonic and divergent.
n =1 n 2 3
It is to be noted that the geometric series and P-series are used in the comparison test as
shown later.
Necessary condition for convergence of a series
For any convergent series, its nth term approaches zero as n tends to infinity,
i.e., Lim u n = 0 . This condition is necessary but it is not sufficient for convergence.
n →

Example 7

1
The series S n =  is harmonic and Lim u n = 0 , but the series diverges because it is P-
n →
n =1 n
series with 𝑝 = 1. (proof is needed)

4.3 Convergence and divergence tests of series


i. The comparison tests
There are two comparison tests; the comparison test, and the limit comparison test.
The two tests are valid for some series, where one test of them is only valid for some
another series.

Comparison test:
Letting ∑ 𝑢𝑛 and ∑ 𝑣𝑛 be two series of positive terms, then
* If series ∑ 𝑣𝑛 converges and ∑ 𝑢𝑛 ≤ 𝑣𝑛 for all 𝑛 being sufficient large, then series∑ 𝑢𝑛
also converges.
** If series ∑ 𝑣𝑛 diverges and ∑ 𝑢𝑛 ≥ 𝑣𝑛 for all 𝑛 being sufficient large, then series ∑ 𝑢𝑛
also diverges.

Example 8

n
Using the comparison test show that the series S n =  converges.
n =1 2n + 3n + 2
3

157
 
n 1
Since the series  3 =  2 is the p-series of 𝑝 = 2, then this series converges.
n =1 2n n =1 2n
n n
by comparison  , then series Sn converges.
2n 3 + 3n + 2 2n 3
Example 9

Ln (n )
Using the comparison test show that the series S n =  n
diverges.
n =2

1
Since the series n is the p-series of p=1, then this series diverges.
n =2
Ln (n ) 1
For n2 by comparison test  , then series Sn diverges.
n n
Limit comparison test:
un
Letting ∑ 𝑢𝑛 and ∑ 𝑣𝑛 be two series of positive terms. If Lim = c , where c R+, then
n → v n

either both series converge or both of them diverge.

Example 10

n
Show that the comparison test is not valid to test the series S n =  ,
n =1 2 n 3
− 3n + 2
use the limit comparison test to show that the series converges.
1  1
Since the series  is the p-series of 𝑝 = 2, then this series converges, otherwise the
2 n =1 n 2
comparison test is not valid because of the existence of the negative sign in the
denominator of the series, then limit comparison test is utilized as follows
n
un 2 n 3
− 3n + 2 2n 3
Lim = Lim = Lim 3 =1 0 ,
n → v n n → 1 n → 2n − 3n + 2
2n 2
So, by using limit comparison test, the series Sn converges.

Example 11

Ln (n )
Show that the limit comparison test is not valid to test the series S n =  n
n =2
for convergence or divergence.

158

1
Since the series n is the p-series of 𝑝 = 1, then this series diverges, then
n =2
Ln (n )
Lim n = Lim Ln (n ) =  (infinite, the test fails).
n → 1 n →
n
ii. The ratio test
u n +1
For a series Sn with positive terms, if S n = Lim = k , 𝑆𝑛 has three cases:
n → u n

If 𝑘 < 1 , the series 𝑆𝑛 converges.


If 𝑘 > 1 , the series 𝑆𝑛 converges.
If 𝑘 = 1 , the test is invalid, and another test is needed.

Example 12

2n − 1
Using the ratio test show that the series S n =  n
converges.
n =1 2
2n − 1
The nth term is written u n = ,
2n
2(n + 1) − 1 2n + 1
The (𝑛 + 1)𝑡ℎ term is written u n +1 = = n +1
2n +1 2
Using the ratio test
u n +1 2n + 1 2 n 1 2n + 1 1
Lim = Lim n +1 . = Lim = 1
n → u n n → 2 2n − 1 2 2n − 1 2
n →
Then, the series 𝑆𝑛 converges.

Example 13

n!
Using the ratio test show that the series S n =  n
diverges.
n =1 3
n!
The nth term is written u n = ,
3n
(n + 1)!
The (𝑛 + 1)𝑡ℎ term is written u n +1 =
3n +1
Using the ratio test

159
u n +1 (n + 1)! 3n 1
Lim = Lim n +1 . = Lim (n + 1) =   1
n → u n n → 3 n! 3
n →
Then, the series 𝑆𝑛 diverges.

iii. The 𝒏𝒕𝒉 root test (Cauchy's test)


For a series 𝑆𝑛 with positive terms, if S n = Lim n u n = k , 𝑆𝑛 has three cases:
n →
If 𝑘 < 1 , the series 𝑆𝑛 converges.
If 𝑘 > 1 , the series 𝑆𝑛 diverges.
If 𝑘 = 1 , the test is invalid, and another test is needed.

Example 14
Using Cauchy's test, test the series Sn for convergence
2 2
1 2 3
S n = +   +   + ...
3 5 7
n
 n 
The 𝑛 term u n = 
𝑡ℎ

 2n + 1 
Using Cauchy's test
n
 n  n 1
Lim n = Lim 
nu n
 = Lim =  1,
n → n →  2n + 1  n → 2n + 1 2

i.e., the series is convergent.

Example 15
Using Cauchy's test, test the series 𝑆𝑛 for divergence
2 2
1 4 9
S n = +   +   + ...
3 5 7
n
 n2 
The 𝑛𝑡ℎ term u n =  
 2n + 1 
Using Cauchy's test
n
 n2  n2
Lim u n = Lim 
n n  = Lim =   1,
n → n →
 2 n + 1  n → 2n + 1

i.e., the series is divergent.

160
Example 16
Show that Cauchy's test is invalid to test the series 𝑆𝑛 for divergence
2 2
2 4 6
S n = +   +   + ...
3 5 7
n
 2n 
The 𝑛 term u n = 
𝑡ℎ

 2n + 1 
Using Cauchy's test
n
 2n  2n
Lim n = Lim 
nu n
 = Lim = 1,
n → n →  2n + 1  n → 2n + 1

i.e., the test is invalid.

iv. Raabi's test


 S 
For a series 𝑆𝑛 with positive terms, if S n = Lim n 1 − n +1  = k , 𝑆𝑛 has three cases:
n →  Sn 
If 𝑘 < 1 , the series 𝑆𝑛 diverges.
If 𝑘 > 1 , the series 𝑆𝑛 converges.
If 𝑘 = 1 , the test is invalid, and another test is needed.

Example 17
1 2 3
Determine the general term of the series S n = + + + ... , then show that the ratio test
2 3 4
fails, and use Raabi's test to prove that the series diverges.
n
The 𝑛𝑡ℎ term is written u n = ,
n +1
n +1
The (𝑛 + 1)𝑡ℎ term is written u n +1 =
n +2
Using the ratio test
u n +1 n +1 n +1 n 2 + 2n + 1
Lim = Lim . = Lim = 1 (the test fails)
n → u n n → n + 2 n n → n 2 + 2n

Using Raabi's test


 u    n + 1 n + 1 
Lim n 1 − n +1  = Lim n 1 −  . 
n →  u n  n →   n + 2 n  

161
 n 2 + 2n + 1 
= Lim n 1 −  = 0 1
n →
 n 2
+ 2 n 
i.e., the series diverges.

Example 18
Use the ratio test and Raabi's test to test the series 𝑆𝑛 for convergence where,

(2n − 1)!
Sn = 
n =1 (2n + 1)(2n !)
(2n − 1)!
The 𝑛𝑡ℎ term is written u n = ,
(2n + 1)(2n !)
(2n + 1)!
The (𝑛 + 1)𝑡ℎ term is written u n +1 =
(2n + 3)(2n + 2)!
Using the ratio test
u (2n + 1)!(2n + 1)(2n !)
Lim n +1 = Lim
n → u n n → (2n + 3)(2n + 2)!(2n − 1)!

2n 2 + 1
= Lim 2 =1
n → 2n + 5n + 3

i.e., the test fails


Using Raabi's test
 u   2n 2 + n 
Lim n 1 − n +1  = Lim n 1 − 2 
n →  u n  n →  2n + 5n + 3 
 4n 2 + 3n 
= Lim  2  = 2 1
n → 2n + 5n + 3
 
i.e., the series converges.

v. The integral test


Letting 𝑆𝑛 be positive and decreasing series of general term 𝑢𝑛 , then the test depends on

the improper integral  u n dn .
1
If the integral converges, the series also converges, otherwise if the integral diverges, the
series also diverges.
Example 21
1 1
Use the integral test to test the series S n = 1 + + + ... for divergence.
2 3
162
 
1 
 u n dn =  n dn = − Ln (n ) 1 =
1 1
the integral is infinite, and the series diverges

Example 19
1 1
Use the integral test to test the series S n = 1 + + + ... for convergence.
4 9
  
1 1
 u n dn =  n 2 dn = − n 1 = 1
1 1
the integral is finite, and the series converges

Example 20
1 1 1
Use the integral test to test the series S n = + 2 4 + 3 9 + ... for convergence.
e e e
  
−n 2 1 1
dn = −  (−2n )e − n dn = − ,
2
 u n dn =  ne 21 2e
1 1
the integral is finite, and the series diverges.

6. Alternating (oscillating) series


Consider the series of alternating sign of terms alternately which takes the form
𝑆𝑛 = 𝑢1 + 𝑢2 + 𝑢3 + 𝑢4 + ⋯ 𝑢𝑛
where 𝑢1 + 𝑢2 + 𝑢3 + 𝑢4 + ⋯ are positive terms. To test this alternating series for
convergence or divergence Leibniz theorem is used.
Leibniz theorem

This theorem stats that, the alternating series S n =  (−1)n +1u n is supposed to be
n =1
convergent if it satisfies the two conditions:
u n +1  u n and Lim u n = 0 ,
n →
otherwise the series is divergent.

Example 21
1 1
Test the series S n = 1 − + − ... for convergence.
2 3

163
 
1 1 1
S n = 1 − + − ... =  u n =  (−1)n +1
2 3 n =1 n =1 n
1 1
In this series un = and u n +1 = , then
n n +1
1
u n +1  u n and Lim u n = Lim = 0 ,
n → n → n
The series satisfies the two conditions, so it converges.

Example 22
3 4
Test the series S n = 2 − + − ... for convergence.
2 3
 
1 1 1
S n = 1 − + − ... =  u n =  (−1)n +1
2 3 n =1 n =1 n
1 1
In this series and u n +1 =
un = , then
n n +1
n
u n +1  u n and Lim u n = Lim =1 0,
n → n → n + 1
The second condition does not satisfy and the series diverges.

4.4 Convergence of power series


The general form of the power series takes the form 𝑎0 + 𝑎1 𝑥 + 𝑎2 𝑥 2 + ⋯ + 𝑎𝑛 ⁡𝑥 𝑛 + ⋯
where: the coefficients: 𝑎0 , 𝑎1 , 𝑎2 , … 𝑎𝑛 are all real numbers.
Determination of convergence interval
Using the ratio test gives
u n +1 a x n +1 a
Lim = Lim n +1 n = Lim n +1 x = kx
n → u n n → a x n → an
n

1 1
The series converges 𝑖𝑓⁡|𝑘𝑥| < 1⁡⁡𝑜𝑟⁡⁡|𝑥| < ⁡1/𝑘⁡⁡𝑜𝑟⁡⁡⁡ − x  .
k k
Example 22
Determine the interval of convergence of the series
1 2 1 3 
x n −1
S n = 1 + x + x + x + ... = 
2! 3! n =1 ( n − 1)!

x n −1 xn
un = and u n +1 =
(n − 1)! n!
Using the ratio test
164
u n +1 x n (n − 1)!
Lim = Lim . = Lim 1 x = 0  1
n → u n n → n ! x n −1 n → n

i.e., the series converges for all values of 𝑥𝑅.

Example 23
Determine the interval of convergence of the series

(x − 2) (x − 2)2 (x − 2)3 (x − 2) n
Sn = + + + ... = 
1 2 3 n =1 n
(x − 2) n (x − 2)n +1
un = and u n +1 =
n n +1
Using the ratio test
u n +1 n n
Lim n +1 = Lim . +
. = Lim (x − 2) = x − 2
n → u n n → (x − 2) n 1
(x − 2) n n → n + 1

Determination of the convergence interval


|𝑥 − 2| < 1⁡⁡⁡⁡⁡ − 1 < ⁡𝑥 − 2 < 1⁡⁡⁡⁡⁡1⁡ < 𝑥⁡ < ⁡3

1
For 𝑥 = 3, the series takes the form S n =  ,
n =1 n
which is the divergent harmonic series.

(−1)n
For 𝑥 = 1, the series takes the form S n = 
n =1 n
which is the convergent alternating series.
Then the interval of convergence becomes 1⁡⁡𝑥⁡ < ⁡3.

Exercise 4
Infinite Sequences and Series

165
The boundaries of a sequence

1. If the sequence 
3 5 7 
 , , ,... is bounded between the two numbers a and b, find the
2 4 6 
general term un, then calculate each number.

2. If the sequence 
3 5 7 
 , , ,... is bounded between the two numbers a and b, find the
2 4 6 
general term un, then calculate each number.

3. If the sequence 1, , ,... is bounded between the two numbers a and b, find the
3 5
 4 9 
general term un, then calculate each number.

Convergence and divergence tests of series


1 3 5
4. Test the series + + ... for convergence.
2 4 8
1 1 1
5. Test the series 1 + + + + ... for convergence.
1 2 1 2  3 1 2  3  4
4 6 6
1 2 3 4
6. Test the series +   +   +   + ... for convergence.
3 5 7 9
1 1 1
7. Test the series 1 + + + + ... for convergence.
1 2 1 2  3 1 2  3  4
4 6 6
1 2 3 4
8. Test the series +   +   +   + ... for convergence.
9 5 7 9
1 2 3
9. Test the series + + + ... for convergence.
2 3 4
Ln (n )
10. Test the series  n 3 + 1 for convergence.
3 4 5
11. Test the series 2 − + − + ... for convergence.
2 3 4
1 4 27
12. Test the series + + + ... for convergence.
3 25 343
166
1 2 3
13. Test the series + + + ... for convergence.
2Ln 2 5Ln 5 10Ln10

The interval of convergence of power series


9 2
14. Find the interval of convergence of the series 3x + x + 9 x 3 + ...
2
x3 x4
15. Find the interval of convergence of the series x + x + 2
+ + ...
2 6

x2 x3
16. Find the interval of convergence of the series x + + + ...
4 9

x2 x3
17. Find the interval of convergence of the series 1 + x + + + ...
2! 3!

Chapter 5
Fourier Series

167
Objectives
This chapter is dedicated to assist the student to understand and to learn the
fundamental topics of Fourier series and its applications. These topics include (importance
and definition, periodic functions, Fourier series and its coefficients, Dirichlet conditions,
general form of Fourier series, Fourier series for symmetric functions). The main objectives
of this chapter are listed below to learn the students how to:
i) expand any periodic or non-periodic function to Fourier series.
ii) expand Fourier series for symmetric functions.
iii) determine some series by using Fourier series such as: fluid mechanics, solid
mechanics, mechanics of materials, structure analysis, and many other
applications.

Importance and definition


Taylor and Maclaurin series are commonly used to expand power series functions, these
expansions are only used for continuous functions of n continuous derivatives in the
convergence interval. Maclaurin series is written
x / x2 xn n
f (x ) = f (0) + f (0) + f //
(0) + ... + f (0) + ...
1! 2! n!
This chapter presents Fourier series which is used to represent series of continues and
discontinuous periodic functions which cannot be represented by Taylor or Maclaurin
series. Fourier series is expressed in trigonometric functions (sins and cosines) which
appear in different fields and applications and in many Engineering systems. Fourier series
are considered one of the most important tools for solving ordinary differential equations,
partial differential equations, and boundary value problems. Fourier theory is divided into
three main parts: Fourier series, Fourier integrals, and Fourier transforms which is related
with Laplace transform and many other applications.

1. Periodic functions
A real function 𝑓(𝑥) is said to be periodic of period P if for all 𝑥,⁡⁡⁡𝑓(𝑥) = 𝑓(𝑥 + 𝑃), where
is a positive constant. The least value of 𝑃 > 0 is called the period of 𝑓(𝑥).
168
Example 1
The trigonometric function 𝑠𝑖𝑛⁡𝑥 has periods 2, 4, 6, …,
Since, 𝑠𝑖𝑛⁡(𝑥 + 2) = 𝑠𝑖𝑛⁡(𝑥 + 4) = 𝑠𝑖𝑛⁡(𝑥 + 6) = ⋯ = 𝑠𝑖𝑛⁡𝑥 …. However, 2⁡is the
least period or simply the period of 𝑠𝑖𝑛⁡𝑥.
Example 2
The period of the function 𝑠𝑖𝑛⁡𝑛𝑥 or 𝑐𝑜𝑠⁡𝑛𝑥 where 𝑛 is any positive integer is 2/𝑛.
Example 3
The period of the function 𝑡𝑎𝑛⁡𝑛𝑥 where 𝑛 is any positive integer is /𝑛.
Example 4

Example 5

x 0 x a
f (x ) = 
k a  x b

169
The graph shows two branches function, the function 𝑓(𝑥) has two-line equations, the
period 𝑃 of the function is shown in figure. All these functions can be expanded by means
of Fourier series.

2. Fourier series
Letting 𝑓(𝑥) be defined in the interval (−𝐿, 𝐿) and outside of this interval by the formula
𝑓(𝑥 + 2𝐿) ⁡ = 𝑓(𝑥), assuming that 𝑓(𝑥) is periodic function of period 𝑃 = 2𝐿. Fourier
series or Fourier series is written
ao   n n 
F (x ) = +   an cos x + bn sin x (1)
2 n =1 L L 
where: 𝑎0 , 𝑎𝑛 , 𝑏𝑛 are called Fourier coefficients. These coefficients are calculated from
Fourier series.

3. Some needed integrals


To determine the values of these coefficients some finite integrals are needed to be
defined as follows:

n n
L L
L
 L
sin x dx = −
n
cox
L −L
=0 (2)
−L

n n
L L
L 2L
 cos L x dx = n sin L −L
=
n
sin n = 0, n = 1,2,3,... (3)
−L

Particular case when 2L=2, (1) and (2) becomes


 2

 sin nx dx =  sin nx dx =0 , (4)


− 0

 2

 cos nx dx =  cos nx dx =0 (5)


− 0

If 𝑚⁡𝑎𝑛𝑑⁡𝑛 are positive integers, then

m x n x
L

 sin L
.cox
L
dx = 0 for all m=n or mn (6)
−L

170
L
m x n x
L
m x n x 0 m  n
 cos
L
cox
L
dx =  sin
L
sin
L
dx = 
L m = n
(7)
−L −L

It should be noted that if 𝑚 = 𝑛 = 0 the value of integral (6) equals zero, and the value of
integral (7) equals 2𝐿.
As a particular case 𝑖𝑓⁡2𝐿 = 2⁡⁡𝐿 = , this gives
 2

 sin mx cox nx dx =  sin mx cox nx dx =0 for all m=n or mn (8)


− 0

L L
0 m  n
 cos mx cox nx dx =  sin mx sin nx dx = 
L m = n
(9)
−L −L

4. Calculating Fourier series coefficients

ao   n n 
Letting f (x ) = +   an cos x + bn sin x  be defined in the interval (-L, L).
2 n =1 L L 
Calculating the coefficient a0
Integrating both sides with respect to x from x=-L and x=L, one get

n x n x
L L L
ao
 f ( x )dx =  2 dx +   (an cos L + bn sin L ) dx
−L −L − L n =1

n x n x
L L
Since,  sin dx =  cos dx =0 , then
−L
L −L
L
L L
ao
f (x )dx =  2
dx = La0
−L −L

L
1
a0 =  f (x )dx (10)
L −L

putting 𝐿 = ⁡, gives


 2
1 1
a0 =
 f (x )dx =
 f (x )dx (11)
− 0

Calculating the coefficient an

171
n x
Multiplying both sides of equation (1) by cos and integrating the equation from 𝑥 =
L
−𝐿⁡𝑡𝑜⁡𝑥 = 𝐿, on get

m x m x
L L
a
 f (x )cos L dx =  2o cos L dx +
−L −L


m x n x m x n x
L
 
+    a n cos L
cos
L
+ bn sin
L
cos
L
 dx

− L n =1

Letting 𝑚 = 𝑛 gives

n x
L
1
an =  f (x )cos dx , n = 0,1,2,... (12 )
L −L L

If 𝑇 = 2𝐿 = 2⁡⁡⁡𝐿 =  gives
 2
1 n x 1 n x
an =  f (x )cos dx = f (x )cos dx , n = 0,1, 2,... (13)
 − L  0
L

As a particular case of an putting n=0, gives


 2
1 1
a0 =
 f (x )dx =
 f (x )dx
− 0

Calculating the coefficient 𝑏𝑛


n x
Using similar way multiplying both sides of equation (1) by sin and integrating from
L
𝑥 = −𝐿⁡𝑡𝑜⁡𝑥 = 𝐿, on get

n x
L
1
bn =  f (x )sin dx , n = 0,1,2,... (14)
L −L L

If 𝑇 = 2𝐿 = 2  L= gives
 2
1 n x 1 n x
bn =  f (x )sin dx =  f (x )sin dx , n = 0,1,2,... (15)
 − L  0
L

5. Dirichlet conditions of Fourier series


Assuming function 𝑓(𝑥) satisfies the following three conditions:
i. 𝑓(𝑥) is defined and single valued at finite 𝑖𝑛𝑡𝑒𝑟𝑣𝑎𝑙⁡(−, )⁡𝑜𝑟⁡(0, 2).

172
ii. 𝑓(𝑥) is periodic outside its period 𝑃 = 2.
iii.⁡𝑓(𝑥) 𝑎𝑛𝑑⁡𝑓′(𝑥) are sectionally continuous along the function interval.
By verifying these three conditions Fourier series converges with the coefficients
𝑎0 , 𝑎𝑛 , 𝑏𝑛 to the followings:
‫ )𝑥(𝑓⁡⁡ـــ‬if x is a connecting point.
f (x + 0) + f (x − 0)
‫ـــ‬ if x is a disconnecting point, where
2
f (x + 0) = Lim f (x +  ),   0,
 →0+

f (x − 0) = Lim f (x −  ),   0.
 →0 −

i.e., 𝑓(𝑥 + 0)⁡𝑎𝑛𝑑⁡𝑓(𝑥 − 0) are the right and left end points of the 𝑓(𝑥) at the
disconnected point. The three conditions of Dirichlet imposed on the 𝑓(𝑥) are sufficient
but not necessary, and often they are satisfied in Engineering applications. Recently, there
is no necessary and sufficient conditions for convergence, and continuity of 𝑓(𝑥) does not
guarantee alone series convergence.

6. General form of Fourier series and its coefficients


The followings are Fourier series and the integral forms of its coefficients for a periodic
function 𝑓(𝑥) of a simplest period 𝑃 = 2𝐿.

ao   n n 
F (x ) = +   an cos x + bn sin x
2 n =1 L L 
L
1
L −L
a0 = f (x )dx

n x
L
1
an =  f (x )cos dx , n = 0,1,2,...
L −L L

n x
L
1
bn =  f (x )sin dx , n = 0,1,2,...
L −L L

Example 6
173
Using Fourier expansion for the function 𝑓(𝑥) = 𝑥, 0 < 𝑥 < 2𝜋


sin nx 1
show that = (  − x ) where 0 < 𝑥 < 2𝜋.⁡
n =1
n 2

Calculating Fourier coefficients


2 2
1 1
ao =
 f (x ) dx =
 x dx = 2
0 0

2 2
1 n x 1
an =
  f (x )cos
L
dx =
  x cos nx dx = 0
0 0

2 2
1 n x 1 2
bn =
  f (x )sin
L
dx =
  x sin nx dx = −
n
0 0

ao  
sin nx
F (x ) = +  (an cos nx + bn sin nx ) =  − 2 
2 n =1 n =1 n


sin nx 1
Since 𝑓(𝑥) = 𝑥, then  n
= ( − x ) , 0 < 𝑥 < 2
2
n =1

Example 7
Using Fourier series for the function
0 − 5  x  0
f (x) = 
3 0  x  5

174

1 2n − 1 
Show that  2n − 1 sin 5
x = , then define the function 𝑓(𝑥) at the disconnecting
4
n =1

points: 𝑥 = −5, 𝑥 = 0, 𝑥 = 5 to converge the series in the interval −5 ≤ 𝑥 ≤ 5.


The function is periodic and sectionally continuous of period 10⁡(𝐿 = 5).
Determination of Fourier coefficients

n n
5 5
1 1
an =  f (x )cos x dx =  3cos x dx
5 −5 5 50 5

n
5 5
3 3
=
50 cos
5
x .dx = 0,...n  0 ,  ao =  dx = 3, n = 0
50
,

1 5 n 3 5 n 3 5 n 5
bn =  f ( x )sin x .dx =  sin x .dx = − cos x
5 −5 5 50 5 5 n 5 0

3 5 n 5 3(1 − cos n )
= − cos x = , n 0
5 n 5 0 n

The corresponding Fourier series is written


ao   n n 
F (x ) = +   an cos x + bn sin x
2 n =1 L L 

3  3(1 − cos n ) n
F (x ) = +  sin x
2 n =1 n 5
3 6  x 1 3x 1 5x 
= +  sin + sin + sin + ...
2  5 3 5 5 5 

3 6  1 2n − 1
= +  sin
2  n =1 2n − 1 5

175

1 2n − 1 
 sin =
n =1 2 n − 1 5 4

Since 𝑓(𝑥) satisfies Dirichlet conditions, the series converges to 𝑓(𝑥) at all connecting and
disconnecting points: 𝑥 = −5, 𝑥 = 0, 𝑥 = 5 where the series converges to

(3 + 0) 3
= the function can be rewritten as follows
2 2

3 / 2, x = −5
 0, −5  x  0

F (x ) = 3 / 2, x =0
 3, 0x 5

3 / 2, x =5

i.e., the series 𝐹(𝑥) converges to function 𝑓(𝑥) in the interval −5 ≤ ⁡𝑥 ≤ ⁡5.
Example 8
 0 0 x 
Using Fourier series for the function f (x ) = 
1   x  2


sin(2n − 1) 
show that  x = − ,   x  2
n =1 2n − 1 4

Determination of Fourier coefficients


2L 2
1 1
a0 =
L  f (x ) dx = 0 +
  dx =1
0 
2
n x
2L
1 1
an =
L f (x )cos
L
dx =
  cos nx dx =0
L 

176
L 2
1 1
bn =  f (x )sin nx dx =  sin nx dx
L −L  
 −2
−1  n  , n is odd
= 1 − ( −1) =  n 
n  
 0, n is even

The corresponding Fourier series takes the form


ao 
F (x ) = +  bn sin nx
2 n =1
1 2 1 1 
= −  sin x + sin 3x + sin 5x + ... 
2  3 5 

1 2  sin(2n − 1)
= −  x
2  n =1 2n − 1

sin(2n − 1) 
  x = − ,   x  2
n =1 2n − 1 4
It is to be noted that the coefficient 𝑎𝑛 = 0, in example (8) as a result of the finite integral
from 𝑥 = ⁡𝑡𝑜⁡𝑥 = 2⁡𝑤ℎ𝑒𝑟𝑒⁡𝑓(𝑥) = 1, without any other reason.

7. Fourier series for even and odd functions

The symmetry of functions has two cases: even symmetric function where the vertical axis
is the coordinate of symmetry. Even functions satisfy the relation 𝑓(𝑥) =
𝑓(−𝑥)⁡𝑓𝑜𝑟⁡𝑎𝑙𝑙⁡𝑥, some examples of even function are: 𝑐𝑜𝑠⁡𝑥, 𝑐𝑜𝑠ℎ⁡𝑥, 𝑥 2 . Otherwise, odd
symmetric function where the origin is the point of symmetry (let it be the origin). Odd
functions satisfy the relation 𝑔(𝑥) = −𝑔(−𝑥)⁡𝑓𝑜𝑟⁡𝑎𝑙𝑙⁡𝑥, some examples of odd functions
are:⁡⁡𝑠𝑖𝑛⁡𝑥, 𝑠𝑖𝑛ℎ⁡𝑥, 𝑥 2 + 𝑥. Fourier series depends on the symmetry of the expanded
function.
Fourier series for even symmetric functions
Fourier series is written
a 
F (x ) = o +  an cos nx + bn sin nx (16)
2 n =1

177
Since 𝑓(𝑥) is even symmetric function of period 2, letting 𝑥 = −𝑥 gives
a 
F (−x ) = o +  (an cos(−nx ) + bn sin(−nx )
2 n =1

a 
= o +  an cos nx − bn sin nx (17)
2 n =1

Since the condition of the even symmetric function is 𝐹(𝑥) = 𝐹(−𝑥), equations (16) and
(17) should be equated, this result

 bn sin nx = 0
n =1
Since 𝑛𝑥⁡0, 𝑡ℎ𝑒𝑛⁡𝑏𝑛 = 0 for all values of 𝑛, and Fourier series includes only cosines and
takes the form
ao 
F (x ) = +  an cos nx (18)
2 n =1
and in this case the two coefficients of this function are written

2
an =
 f (x )cos nx dx , (19)
0


2
 0
a0 = f (x ) dx (20)

Example 8

Using Fourier series for the function 𝑓(𝑥) = │𝑥│, −𝜋 < 𝑥 < 𝜋 show that

cos(2n − 1)   
 x =  −x , 0  x  
n =1 (2n − 1) 4 2 
2

−x −  x  0
𝑓(𝑥) may take the form f (x ) = 
 x 0 x 

178
The function is periodic, continuous, and even symmetric, then 𝑏𝑛 = 0, and the
coefficients 𝑎0 and 𝑎𝑛 are determined as follows:

 
2x2
a0 =  xdx = = ,
0 2 0

2
an =
  x cos nx dx
0

Integrating by parts gives


2 x 
1 
an =  sin x −  sin nxdx 
 n n 0


2 cos nx 2 
= = 2
(−1)n − 1
 n2 0 n

In this case Fourier series takes the reduced cosines form


a0 
F (x ) = +  an cos nx
2 n =1
 4  cos x cos3x cos5x 
= −  2 + + + ... 
2  1 32 52 


 cos(2n − 1)

4
= − x
2 n =1 (2 n − 1) 2

In this example the Fourier function 𝐹(𝑥) can be defined at all connecting points and
intervals to make 𝐹(𝑥) converge to the function 𝑓(𝑥) as follows

179
 −x , −  x  0

F (x ) =  0, x =0
 x, 0 x 


cos(2n − 1)
To determine the series  (2 n − 1) 2
x , 0  x   , let
n =1


 cos(2n − 1)

4
x = − x , gives
2 n =1 (2 n − 1) 2


cos(2n − 1)   
 x =  −x , 0  x  
n =1 (2n − 1) 4 2 
2

Fourier series for odd symmetric functions


Using the condition of the odd symmetric function 𝑓(𝑥) = −𝑓(−𝑥) and substituting in
equation (16) gives
a 
−F (−x ) = − o +  −an cos(nx ) + bn sin(nx ) (21)
2 n =1

equating equation (21) to equation (16) gives


ao 
+  a cos nx = 0
2 n =1 n

So that, a0=an=0 for all values of n, and Fourier series includes only sins and could takes the
form

F (x ) =  bn sin nx
n =1

and in this case the coefficient bn is written



2
bn =
 f (x )sin nx dx (23)
0

Example 9
− , −  x  0
Use Fourier series for the function f (x ) = 
 , 0  x 
180
sin 3x sin 5x 
to show that sin x + + + ... = , 0  x  
3 5 4

The function is odd and continuous with period 2, then Fourier coefficients are:
𝑎0 = 𝑎𝑛 = 0, and
2 2  2
bn =   sin nxdx = −  cos nx  = − (−1)n − 1
0 n 0 n
Substituting in Fourier expansion gives
 
F (x ) =  b n sin nx =  1 − (−1) n  sin nx
2
 
n =1 n =1 n

If 𝑛 is an odd number the series takes the form

 sin 3x sin 5x 
F (x ) = 4  sin x + + + ... 
 3 5 
In this example the function 𝐹(𝑥) can be defined for Fourier series at all connecting and
disconnecting points to approximate the function 𝐹(𝑥)⁡𝑡𝑜⁡𝑓(𝑥).

 0, x = −
− , −  x  0

F (x ) =  0, x =0
 , 0 x 

 0, x =

Letting 𝐹(𝑥) = , gives

181
sin 3x sin 5x 
sin x + + + ... = , 0  x  
3 5 4

Notifications:
1. Define the type of the function (periodic, continuous, sectionally continuous).
2. Determine the period of any periodic function to be expanded.
3. For odd function let a0=an=0 without any additional calculations, and for even function
let bn=0 without any additional calculations.
4. In case of asymmetric function all Fourier coefficients must be determined.
5. Use Dirichlet conditions to calculate the values of Fourier function at the disconnecting
points along the intervals of the range of the function.
6. Check the validity of Fourier series at some distinct points.

182
Exercise 5
Fourier Series

For the following functions, define all properties and illustrate each graph, then deduce
Fourier series for each periodic function where 𝑓(𝑥) = 𝑓(𝑥 + 𝑃), 𝑃 = 2𝐿.

0 for 0x 2
1. f (x ) = 
1 for 2x 4

2. f (x ) = x for 0  x  2

−x 2 −  x  0
 for
3. f (x ) = 
 2 0 x 
x for

4. Deduce Fourier series of the function

 1 for −  x  0
f (x ) = 
x for 0 x 

2 1 1 1
then, show that = 2
+ 2 + 2 + ...
8 1 3 5

x2
5. Show that Fourier series of the function f (x ) = in the interval -<x< 
4
 2  (−1)n
takes the form F (x ) = + cos nx
12 n =1 n 2

183
‫‪References‬‬

‫‪• M.D. Raisighania: Advanced Differential Equations. S.‬‬


‫‪Chand and Company Ltd., india 1991.‬‬
‫‪• E.D. Rainville and P. Bediant: Elementary Differential‬‬
‫‪Equations, McMillan Pub. Co., New York, 1980.‬‬
‫‪• M. Rao: Ordinary Differential Equations, John Wiley‬‬
‫‪and Sons, N.Y. 1989.‬‬
‫‪• S. Rao: Ordinary Differential Equations, John Wiley and‬‬
‫‪Sons, N.Y. 1989.‬‬

‫المعادالت التفاضلية‪ :‬ريتشارد برنسون (سلسلة سشوم) الدار الدولية‬ ‫•‬


‫لالستثمارات التقافية‪ ,‬ترجمة د‪ /.‬حسن العويضى‪ ,‬د‪ /.‬عبدالوهاب عباس‬
‫(‪ )2001‬القاهرة‪.‬‬
‫نظريات المعادالت التفاضلية‪ ,‬د‪ /.‬رحمة عبدالكريم‪ ,‬مطبوعات الملك سعود‪,‬‬ ‫•‬
‫‪ 1408‬هجرية‪.‬‬
‫نظرية وسائل المعادالت الفاضلية (سلسلة سشوم) فرانك أيرز‪ ,‬الدار الدولية‬ ‫•‬
‫لالستثمارات الثقافية‪ 1997 .‬ميالدية‪.‬‬
‫المعادالت التفاضلية العادية‪ ,‬الجزء الثانى‪ :‬د‪ /.‬حسن العويضى‪ ,‬د‪ /.‬عبدالوهاب‬ ‫•‬
‫عباس‪ ,‬د‪ /.‬سناء على زارع‪ ,‬دار الرشد‪ 2005 ,‬ميالدية‪.‬‬
‫الرياضيات المتقدمة للمهندسين والعلميين‪ ،‬تأليف موراى شبيجل‪ ،‬ترجمة كل‬ ‫•‬
‫من‪ :‬أ‪.‬د سعد كامل مسعود و أد‪ .‬إدوارد ميخائيل إبراهيم‪ ،‬ملخصات سشوم‪.‬‬
‫تحليالت فورى‪ ،‬سلسلة سشوم‪.1974 ،‬‬ ‫•‬
‫مبادئ التحليل الرياضى‪ ،‬تأليف أ‪.‬د مصطفى الجندى‪ ،‬سلسلة مجموعة كتب‬ ‫•‬
‫الرياضيات‪ ،‬دار الراتب الجامعية‪ ،‬مصر ‪.1995‬‬
‫‪184‬‬

You might also like